Problemas de Tratamiento Digital de Señales [2 ed.]
 9788415834205;8415834209

  • Commentary
  • Noitaenola’s library
  • 0 0 0
  • Like this paper and download? You can publish your own PDF file online for free in a few minutes! Sign Up
File loading please wait...
Citation preview

UAH

TEXTOS UNIVERSITARIOS TECNOLOGÍA

           Problemas de Tratamiento Digital de Señales (2ª edición)

Manuel Blanco Velasco Fernando Cruz Roldán Roberto Jiménez Martínez José Sáez Landete

Problemas de Tratamiento Digital de Señales (2ª edición)

Problemas de Tratamiento Digital de Señales (2ª edición)

SERVICIO DE PUBLICACIONES

Manuel Blanco Velasco Fernando Cruz Roldán Roberto Jiménez Martínez José Sáez Landete

El contenido de este libro no podrá ser reproducido, ni total ni parcialmente, sin el previo permiso escrito del editor. Todos los derechos reservados. © Universidad de Alcalá Servicio de Publicaciones Plaza de San Diego, s/n 28801 Alcalá de Henares www.uah.es ISBN: 978-84-15834-20-5 Depósito Legal: M-26092-2013 Impresión y encuadernación: Imprenta de la UAH Impreso en España

Índice general Prólogo

iii

1. Conceptos Básicos y Tipos de Sistemas

1

2. Muestreo y Reconstrucción de Señales

41

3. La Transformada Discreta de Fourier

71

4. Filtros Digitales 91 4.1. Diseño de Filtros Digitales IIR . . . . . . . . . . . . . . . . . . . . . . . . . 91 4.2. Diseño de Filtros Digitales FIR . . . . . . . . . . . . . . . . . . . . . . . . . 116 4.3. Estructuras de Filtros Digitales . . . . . . . . . . . . . . . . . . . . . . . . . 139 5. Sistemas de Tasa Múltiple

167

6. Análisis Espectral

203

Bibliografía

217

Prólogo El objetivo general del libro consiste en presentar una colección de problemas propuestos con solución que sirva de apoyo a cualquier asignatura cuya temática esté relacionada con el Tratamiento Digital de Señales. La mayor parte del libro se centra en sistemas discretos lineales y temporalmente invariantes, aunque también se incluyen bloques lineales que no satisfacen la propiedad de invarianza, y que tienen gran utilidad en comunicaciones y en diversos sistemas de codificación de señales de audio, imágenes y biomédicas. Con este libro, se pretende adquirir y afianzar los conocimientos y habilidades siguientes: Conocer las ventajas e inconvenientes más relevantes que presenta el tratamiento digital de señales frente al procesado analógico. Saber las características más importantes de los sistemas paso-todo, de fase mínima, de fase lineal. Dominar las técnicas de muestreo y reconstrucción de señales de tiempo continuo, así como el uso de técnicas de tratamiento digital para procesar señales de tiempo continuo. Conocer la DFT, sus aplicaciones y algoritmos rápidos de cálculo. Conocer las ventajas e inconvenientes más relevantes que ofrecen los filtros de respuesta al impulso de duración infinita (IIR) frente a los de duración finita (FIR). Elegir las técnicas de diseño -etapa de aproximación- más adecuadas a las especificaciones iniciales de partida. Representar el filtro digital a través de diferentes estructuras, conociendo algunos de los problemas que acarrea en las mismas el uso de aritmética de precisión finita. Conocer los bloques básicos que componen los sistemas multitasa de muestreo y los efectos que producen en la señal de tiempo discreto. Conocer las técnicas de estimación espectral y sus propiedades fundamentales. Se propone la organización de capítulos expuesta en la tabla 1. La estructura del programa se ha realizado atendiendo a la experiencia de los autores en la impartición de asignaturas con estos contenidos y similares. Este libro complementa otro conjunto de libros de texto con contenidos teóricos en los que se explican los conceptos y las técnicas necesarias para desarrollar cada uno de los problemas propuestos. En la bibliografía se muestra una colección de libros variada e interesante, sin embargo, desglosados por temas, se sugieren los libros indicados en la tabla 1. Algunas referencias incorporan programas en Matlab, que sirven como complemento a las partes teórica y práctica de la asignatura. Para facilitar el estudio de la asignatura, también se recomiendan algunas de las interfaces gráficas de usuario (GUI) de demostración que incorpora Matlab. Además de las anteriores,

Prólogo.

iv Tabla 1 – Organización del libro.

Capítulo I Capítulo II Capítulo III Capítulo IV Capítulo V Capítulo VI

Temario Conceptos Básicos y Tipos de Sistemas Muestreo y Reconstrucción de Señales La Transformada Discreta de Fourier Filtros Digitales Sistemas de Tasa Múltiple Análisis Espectral

Bibliografía recomendada [1, 2] [1, 2] [1, 3] [1, 2, 3] [1, 4, 5]. [1, 3, 6, 7]

resultan de gran utilidad las GUIs o applets desarrolladas en el Center for Signal and Image Processing [8], del Georgia Institute of Technology, la coordinada por el profesor Andreas Spanias de la Arizona State University [9], también [10] de la John Hopkins University, bastante completa, y por último, la elaborada por C. A. Nyack [11]. La notación utilizada en el libro es diferente a la utilizada en algunos libros de la bibliografía. Se emplean corchetes para indicar las dependencias de variables independientes discretas y paréntesis para las dependencias de variables independientes continuas. A su vez, la variable ω representa la pulsación para el dominio transformado de Fourier de señales de tiempo continuo, mientras que Ω representa la pulsación correspondiente al dominio transformado de Fourier de señales de tiempo discreto.

Capítulo 1

Conceptos Básicos y Tipos de Sistemas Problema 1.1 Del sistema de la figura 1.1 se conoce que es LTI, de orden dos, de respuesta al impulso real y finita (h[n] 6= 0 sólo para 0 ≤ n ≤ 2), y que presenta un cero en c1 = e−j·2/3 . A dicho 2 4π sistema se le introduce como entrada la secuencia x [n] = 12 · ej· 3 ·n + 41 · ej· 3 ·n .

x[n]

y[n] h[n] Figura 1.1

Se pide: a) Razonar si la secuencia de entrada es periódica y, en caso afirmativo, indicar el valor del periodo de dicha señal. b) Dibujar el diagrama de polos y ceros del sistema dado. Sabiendo que H (Ω)|Ω=0 = 1, obtener su función del sistema. c) Dibujar de manera aproximada el módulo de su respuesta en frecuencia. d) Razonar si la secuencia de salida y[n] es periódica y, en caso afirmativo, indicar el valor del periodo. Solución

a) No es periódica.    2 2 b) La función sistema es: H (z) = 2,335 1 − e−j 3 z −1 1 − ej 3 z −1 .

Conceptos Básicos y Tipos de Sistemas.

2

1 Parte Imaginaria

e j 2/3 0,5 2

0 −0,5

e -j 2/3 −1 −1

−0,5

0 0,5 Parte Real

1

c) El módulo de la respuesta en frecuencia:

10 8

|H((Ω)|

6 4 2 0 0

1

2

3

4

5

6

Ω (rad) d) La salida es periódica con periodo N = 3. Problema 1.2 Al sistema LTI real, causal y estable de la figura 1.2 se le introduce como señal de entrada la secuencia x1 [n] = ejΩ0 ·n , con Ω0 ∈ R+ .

x[n]

y[n] h[n] Figura 1.2

Se pide: a) Empleando la expresión de la suma de convolución, calcular la secuencia de salida y1 [n] a la anterior exponencial. b) Expresar la respuesta y2 [n] del sistema cuando la entrada es la secuencia x2 [n] = ejΩ0 ·n · u [n] como la suma de dos términos: y2 [n] = yper [n] + ytra [n] ,

3 donde yper [n] = y1 [n]. c) Demostrar que al ser el sistema LTI estable, la respuesta ytra [n] está acotada en amplitud, es decir, |ytra [n] | < B < ∞, ∀n, B ∈ R+ . Solución  a) y1 [n] = ejΩ0 n H ejΩ0 .

b) y2 [n] = y1 [n] − c) |ytra [n]| ≤

∞ P

∞ P

h [l] ejΩ0 (n−l) .

l=n+1

l=n+1

|h [l]| ≤

∞ P

l=−∞

|h [l]| < K < ∞.

Problema 1.3 A un sistema LTI causal, estable y real se le introduce una secuencia de entrada x[n] real y causal cuya parte imaginaria de su transformada de Fourier es XI (Ω) = −sen (Ω) . La secuencia de salida y[n] presenta un espectro cuya parte real viene dada por la expresión YR (Ω) = 1 − cos (2Ω) . Se pide: a) Sabiendo que x [0] = 1, obtener la secuencia de entrada x [n]. b) Obtener la secuencia de salida y [n]. c) Obtener las respuestas en frecuencia y al impulso del sistema LTI dado. Datos: Considere que una secuencia x [n] causal se puede expresar como: x [n] = 2 · xe [n] · u [n] − xe [0] · δ [n] , x [n] = 2 · xo [n] · u [n] + x [0] · δ [n] , donde xe [n] y xo [n] son, respectivamente, la parte par e impar de la secuencia. Solución a) x [n] = δ [n] + δ [n − 1]. b) y [n] = δ [n] − δ [n − 2]. c) H (Ω) = 1 − e−jΩ , h [n] = δ [n] − δ [n − 1].

Conceptos Básicos y Tipos de Sistemas.

4 Problema 1.4 Dada la siguiente señal:

X (Ω) = R (Ω) + δ (Ω) + δ (Ω − π/2) + δ (Ω + π/2) , para − π < Ω < π, con X (Ω) periódica de periodo 2π y R (Ω) =



2 0 ≤ |Ω| ≤ π/4, 0 π/4 < |Ω| ≤ π,

se pide: a) Representar X (Ω) para −2π < Ω < 2π. b) Obtener x [n]. Expresar dicha secuencia como suma de dos términos de la forma x [n] = xp [n] + xap [n] , donde xp [n] contiene en su totalidad la parte periódica de la señal x [n], y xap [n] el resto. c) Representar la secuencia xp [n] e indicar su periodo fundamental. d) Dado el sistema LTI causal, caracterizado por la siguiente ecuación en diferencias de coeficientes constantes y reales: y [n] = x [n] +

3 X

k=1

bk x [n − k] + 0,9y [n − 1] − 0,81y [n − 2] + 0,729y [n − 3] ,

obtener el valor de los coeficientes bk para que se satisfaga que y [n] = h [n] ∗ x [n] = h [n] ∗ xap [n] , donde h [n] es la respuesta al impulso que caracteriza al sistema. e) Sabiendo que el sistema anterior presenta un polo en p1 = 0,9j , representar su diagrama de polos y ceros completo, así como el módulo de la respuesta en frecuencia del mismo de manera aproximada. Solución a) El espectro de la señal X(Ω) se muestra en la siguiente figura:

X(Ω) 2

−π −π/2 −π/4

b) Los términos que componen la señal son:

π/4 π/2

π

π  1 1 + cos ·n , 2π π 2 2 sen (n · π/4) xap [n] = . πn xp [n] =



5 c) El periodo es N = 4.

x p [n] 3/2π N=4 1/2π -2 -4 -3

2 -1

3

1

4

n

-1/2π d) b1 = −1, b2 = 1, b3 = −1. e) Los polos se encuentran situados en: p1 = 0,9j, p2 = −0,9j y p3 = 0,9. El diagrama de polos y ceros y su respuesta en frecuencia es: 1,2

p1

1

0,5

0,8 0

|H(Ω)|

Parte Imaginaria

1

p3

0,6 0,4

−0,5

p2

0,2

−1 −1

−0,5

0

0,5

0

1

−π

Parte Real

−π/2

0

π/2

π

Ω(rad)

Problema 1.5 El esquema de la figura 1.3 está compuesto por tres sistemas LTI, se emplea para procesar la señal de entrada x[n].

y1[n]

S1

y[n]

x[n]

S3

+ y3[n]

S2

y2[n] Figura 1.3

Acerca del conjunto se conocen los siguientes datos:

Conceptos Básicos y Tipos de Sistemas.

6 La relación entrada-salida del S1 es: y1 [n] =

3 5

∞ P

k=0

 3 k 4

x [n − k].

Al inyectar x [n] = δ[n] − 34 δ [n − 1] como señal de entrada del esquema general, se n obtiene y3 [n] = − 21 u [n] a la entrada del sistema S3 . El sistema S3 es real, causal, de 2º orden y con todos sus polos en el origen. Se pide: a) Demostrar que el sistema S1 es LTI y calcular su respuesta impulsiva y su función del sistema. b) Determinar la respuesta impulsiva h2 [n] del sistema S2 . c) Se ha comprobado que la salida del sistema global cuando se aplica la señal x [n] =  n cos (πn) + cos 7π 4 n es y[n] = α (−1) , α ∈ R. Determinar la función del sistema S3 , sabiendo que H3 (Ω)|Ω=0 = 0,2929. Solución a) El sistema es lineal e invariante.  n 3 3 u [n] h1 [n] = 5 4 3 3 1 , |z| > H1 (z) = 5 1 − 43 z −1 4 b) h2 [n] = c) H3 (z) =

2 5 1 2

 −1 n 2



u [n].

1 − ej

7π 4

z −1



1 − e−j

7π 4

 z −1 .

Problema 1.6 El diagrama de bloques de la figura 1.4 está pensado para procesar una señal real x [n] ∈ R. Se sabe que la señal de salida y[n] se puede obtener mediante un único sistema LTI.

Sistema LTI H( )

1

x[n]

y[n] 2

e

2

-J( / N

e

J(

/ N

Figura 1.4

Se pide: a) Determinar razonadamente la respuesta impulsiva heq [n] del sistema LTI equivalente al de la figura que permite obtener la misma respuesta y[n] cuando se aplica idéntica entrada x[n].

7 b) Determinar y representar el sistema que permite recuperar la señal de entrada x[n] a partir de la señal de salida y[n] mediante un diagrama de bloques en el que se especifiquen cada una de las funciones que realiza. Solución π

a) heq [n] = ej 2 n

sen( π 2 n) . πn

b) x[n] = 2 · Re {y[n]}:

y[n]

Re{·}

2

x[n]

Problema 1.7 La ventana de duración finita wT (t) de la figura 1.5 se emplea para transmisión de datos en comunicaciones de banda ancha y está definida matemáticamente como:     π t  −TT R /2 ≤ t ≤ TT R /2,  sen2 2 0,5 + TT R ,  wT (t) = TT R /2 ≤ t ≤ T − TT R /2,    1,    sen2 π 0,5 − (t−T ) , T − TT R /2 ≤ t ≤ T + TT R /2. 2

TT R

Figura 1.5

Si los parámetros relativos a la duración de la ventana son T = 4 µs y TT R = 100 ns, y se muestrea la señal a un régimen de 20 · 106 muestras por segundo, se pide: a) Obtener la expresión de la ventana discretizada wT [n] = wT (nTs ).

b) Obtener la transformada de Fourier de wT [n]. Solución a) La ventana es:

b) La transformada de Fourier: WT (Ω) =

 0,5, n = 0,    1, 1 ≤ n ≤ 79, wT [n] =  0,5, n = 80,   0, resto. sen(39,5 · Ω) −j·40·Ω 1 1 −j·80·Ω ·e + + ·e sen(0,5 · Ω) 2 2

Conceptos Básicos y Tipos de Sistemas.

8

Problema 1.8 De una función C(z) = H(z) · H ∗ ( z1∗ ) se conoce que presenta un cero y un polo en los siguientes puntos del plano z: Cero: c1 = 0,5 + 0,5j, Polo: p1 = −0,5 − 0,5j. Se pide: a) Completar el resto de polos y ceros de la función C(z) de orden mínimo que cumple los datos del enunciado. Considere a partir de ahora que, además de las condiciones anteriores, se cumple que la transformada z inversa de C(z) es una secuencia real. Se pide: b) Añadir los polos y ceros necesarios para obtener la función C(z) de orden mínimo que cumple lo indicado. c) Encontrar todas las posibles funciones H(z) correspondientes a sistemas causales, estables, de respuesta al impulso real que tengan la misma C(z) obtenida en el apartado anterior. Solución a) Falta un cero y un polo, respectivamente: c2 = 1 + j p2 = −1 − j b) Los ceros y polos son, respectivamente: c3 = 0,5 − 0,5j c4 = 1 − j p3 = −0,5 + 0,5j p4 = −1 + j c) Los posibles sistemas son:   1 − c1 · z −1 1 − c3 · z −1 H1 (z) = k1 , (1 − p1 · z −1 ) (1 − p3 · z −1 )

1 |z| > √ 2

  1 − c2 · z −1 1 − c4 · z −1 , H2 (z) = k2 (1 − p1 · z −1 ) (1 − p3 · z −1 )

1 |z| > √ 2

Problema 1.9 De una función C(z) = H(z) · H ∗ ( z1∗ ) se conoce que presenta ceros y polos en los siguientes puntos del plano z: Polos: zp1 = 21 , zp2 = − 21 + 21 j. Ceros: zc1 = −2, zc2 =

1 2

+ 21 j.

Sabiendo que los coeficientes de la transformada inversa de la función C(z) son reales:

9 a) Dibujar el diagrama de polos y ceros de la función C(z) de orden mínimo que cumple las condiciones del enunciado. b) Dibujar el diagrama de polos y ceros asociado a H(z), sabiendo que se trata de un sistema LTI causal de fase mínima. c) Dibujar el diagrama de polos y ceros de orden mínimo de todos los posibles sistemas reales LTI causales y estables cuyo módulo de la respuesta en frecuencia es el mismo que el de H(z) del apartado anterior. d) La siguiente figura representa el módulo y la fase de uno de los sistemas Hi (z) obtenidos en los apartados b y c. Sin realizar ningún cálculo, razone a cual o cuales de ellos corresponde.

Magnitud (dB)

20 10 0 −10

0

0,2 0,4 0,6 0,8 Frecuencia Normalizada ( ×π rad/muestra)

1

0,2 0,4 0,6 0,8 Frecuencia Normalizada ( ×π rad/muestra)

1

Fase (grados)

0

−200

−400

0

e) ¿Existe algún otro sistema real LTI causal y estable, distinto de los ya obtenidos, que tenga el mismo módulo de la respuesta en frecuencia que los anteriores? En caso afirmativo ponga un ejemplo. ¿Podría ser este sistema de fase mínima? Razone la respuesta.

Conceptos Básicos y Tipos de Sistemas.

10 Solución a) El diagrama de polos y ceros es:

C(z) 2 1,5 Parte Imaginaria

1 0,5 0 −0,5 −1 −1,5 −2 −2

−1

0 Parte Real

1

2

b) El diagrama de polos y ceros es:

H(z) 1 0,8 Parte Imaginaria

0,6 0,4 0,2 0

−0,2 −0,4 −0,6 −0,8 −1 −1

−0,5

0 0,5 Parte Real

1

11 c) Los diagramas de polos y ceros son: H (z)

H (z)

2

1

1,5

1 0,8

1

Parte Imaginaria

Parte Imaginaria

0,6 0,4 0,2 0 −0,2 −0,4 −0,6

0,5 0 −0,5 −1

−0,8 −1,5

−1 −1

−0,5

0

0,5

−2

1

−1,5

−1

−0,5

0

0,5

1

Parte Real

Parte Real

H 3(z) 1,5

Parte Imaginaria

1 0,5 0 −0,5 −1 −1,5 −2

−1,5

−1

−0,5

0

0,5

1

Parte Real d) Corresponde al sistema H1 (z). e) Basta con multiplicar por un sistema paso todo causal cualquiera de las funciones Hi (z) obtenidas anteriormente. Por ello, existen infinitas posibilidades. Sin embargo, al introducir el sistema paso todo ceros ubicados fuera de la circunferencia de radio unidad, ninguna de las nuevas funciones resultantes de igual módulo cumpliría la propiedad de fase mínima.

Conceptos Básicos y Tipos de Sistemas.

12

Problema 1.10 De un sistema H(z) lineal, invariante y estable se conoce que satisface la condición C(z) = H(z) · H ∗ ( z1∗ ), y que dicha función C(z) presenta un polo en p1 = −0,5 y un cero en c1 = 0,5 − 0,5j. Se pide: a) Dibujar el diagrama de polos y ceros del sistema completo C(z), sabiendo que es el de orden mínimo que cumple las condiciones anteriores. b) Obtener las diferentes funciones H(z) que darán lugar a C(z). Discutir para cada una de las funciones obtenidas si se correspondería con un sistema causal. c) Si C(z) se descompone como producto de dos funciones del siguiente modo: C(z) = Hmin (z) · H1 (z), encontrar Hmin (z), sabiendo que caracteriza a un sistema de fase mínima estable con el mismo módulo de la respuesta en frecuencia (salvo una constante) y de igual orden que C(z). Dibujar el diagrama de polos y ceros y el módulo de la respuesta en frecuencia del sistema H1 (z). Solución a) Es necesario incluir: c2 = 1 − j p2 = −2

C(z)

Parte Imaginaria

1 0,5 0

p2

p1 c1

−0,5

c2

−1 −2

−1,5

−1

−0,5

0

0,5

Parte Real b)

 1 − c1 · z −1 Ha (z) = k1 , |z| > |p1 | (1 − p1 · z −1 )  1 − c2 · z −1 , |z| > |p1 | Hb (z) = k2 (1 − p1 · z −1 )  1 − c1 · z −1 Hc (z) = k3 , |z| < |p2 | (1 − p2 · z −1 )

1

13  1 − c2 · z −1 , |z| < |p2 | Hd (z) = k4 (1 − p2 · z −1 )

donde Ha (z) es causal y estable, Hb (z) es causal y estable, Hc (z) es no causal y estable y Hd (z) es no causal y estable. 2

(1−c1 ·z−1 ) c) Hm´ın (z) = k (1−p ·z−1 )2 . 1

H 1(z)

Parte Imaginaria

1

0,5

0

p2

c3

−0,5

p3

c2

−1 −2

−1,5

−1

−0,5 0 Parte Real

0,5

1

1 0,9 0,8

|H1(Ω )|

0,7 0,6 0,5 0,4 0,3 0,2 0,1 0 0

0,5

1

1,5 2 2,5 Ω (rad/muestra)

3

3,5

Conceptos Básicos y Tipos de Sistemas.

14

Problema 1.11 Dado un sistema FIR causal, de orden dos, con coeficientes reales y con un cero en c1 = 0,25 − 0,25j, se pide: a) Encontrar su función del sistema (H1 (z)), incluida la constante de ganancia, sabiendo que H1 (Ω)|Ω=0 = 1. b) De todos los sistemas que tienen el mismo módulo de la respuesta en frecuencia que H1 (z), ¿qué propiedad satisface el anterior sistema con respecto a la energía parcial acumulada? Razone la respuesta. c) Encontrar H2 (z), incluida la constante de ganancia, del sistema causal de retardo de energía máximo que tiene el mismo módulo de la respuesta en frecuencia que H1 (z). d) Razone la estabilidad de ambos sistemas. Solución

a) H1 (z) =

8 5

  · 1 − (0,25 + 0,25j) z −1 · 1 − (0,25 − 0,25j) z −1 .

b) Como es un sistema de fase mínima, tiene la propiedad de retardo de energía mínimo. c) H2 (z) =

1 5

  · 1 − (2 + 2j) z −1 · 1 − (2 − 2j) z −1 .

d) Ambos sistemas son estables pues tienen todos los polos están en el origen. Problema 1.12 Dado un sistema estable cuya función del sistema viene dada por H (z) = 2,4414 ·

0,4096 − 1,3988z −1 + 2,3885z −2 − 2,1856z −3 + z −4 1 − 2,1856z −1 + 2,3885z −2 − 1,3988z −3 + 0,4096z −4

Se pide: a) Representar el módulo de la respuesta en frecuencia. b) El sistema anterior presenta un polo en p1 = 0,8 · ejπ/3 . Las dos siguientes figuras representan la función de fase de dos sistemas estables, y una de ellas se corresponde con la del sistema indicado anteriormente. Deducir razonadamente de cuál de las dos se trata. Sistema (a)

Sistema (b)

0

0.3

−2 0.2

Radianes

Radianes

−4 −6 −8

0.1 0

−10 −0.1

−12 −14 0

0.5

1

1.5

Pulsación

2

2.5

3

−0.2 0

0.5

1

1.5

Pulsación

2

2.5

3

15 c) Dibujar el diagrama de polos y ceros completo sabiendo que el sistema también presenta un polo en p2 = 0,8 · e−jπ/6 . Solución A(z −1 ) a) La función del sistema dada, con coeficientes reales, satisface que H (z) = k · z −N A(z) . Por tanto es un sistema paso todo. Evaluando dicha función en un punto de la circunferencia de radio unidad, por ejemplo, H (z)|z=1 = 2,4414, se obtiene el resto de valores:

3 2.5

Amplitud

2 1.5 1 0.5 0 0

0.5

1

1.5

2

2.5

3

Pulsación

b) Observando la función del sistema, se aprecia que es un sistema paso-todo de orden cuatro de coeficientes reales. Con los datos del enunciado, se conoce que hay dos polos en el interior de la circunferencia de radio unidad, con lo que al menos debe haber dos ceros fuera de la circunferencia de radio unidad. La fase del sistema (b) se corresponde con un sistema que tendría todos los ceros en el interior de la circunferencia de radio unidad, con lo que no podría ser el paso-todo. La del sistema (a) sí satisface las anteriores condiciones, y es además monótona decreciente, la cual daría lugar a un retardo de grupo positivo. Por tanto, la opción correcta es la del sistema (a). c)

c =1/conj(p )

1

1

1

c =1/conj(p )

Parte Imaginaria

3

p

0.5

1

3

p =conj(p ) 3

2

0 p

2

−0.5

c =1/conj(p ) 2

2

p =conj(p ) 4

1

−1

c =1/conj(p ) 4

−1

−0.5

0

Parte Real

0.5

4

1

1.5

Conceptos Básicos y Tipos de Sistemas.

16

Problema 1.13 De un sistema LTI se conoce que su función H(z) es de coeficientes reales y tiene dos polos en los siguientes puntos del plano z: 2π 3

p1 = 2 · e j

, p2 =

1 . 2

Además, se ha comprobado que la respuesta del sistema es nula cuando a la entrada se introduce cualquiera de las dos secuencias siguientes: n  π  2 n , x2 [n] = − x1 [n] = cos 4 3 Se pide: a) Representar el diagrama de polos y ceros de la función H(z) de orden mínimo indicando el valor de los mismos. Determinar también su expresión algebraica y comentar razonadamente si el sistema es causal y estable. b) Determinar el diagrama polo-cero y la expresión algebraica de todas las funciones causales y estables que tienen el mismo módulo de la respuesta en frecuencia que H(z). c) Expresar los sistemas obtenidos en el apartado anterior que tengan ceros en el exterior de la circunferencia de radio unidad como el producto de un filtro sin ceros en el exterior de la circunferencia de radio unidad, y de un sistema paso-todo. Solución a) El diagrama de polos y ceros es:

H(z) 1,5

Parte Imaginaria

1 0,5 0 −0,5 −1 −1,5 −2

−1,5

−1

−0,5

0

0,5

1

1,5

2

Parte Real La función sistema es:    π π 1 + 32 z −1 1 − ej 4 z −1 1 − e−j 4 z −1 1  , < |z| < 2 H (z) = k  2π 2π 1 −1 2 1 − 2ej 3 z −1 1 − 2e−j 3 z −1 1 − 2z

17 El sistema es no causal y estable. b) Los diagramas de polos y ceros son:

H1(z)

Parte Imaginaria

1

0,5

0

−0,5

−1 −1

−0,5

0

0,5

1

Parte Real

H2(z)

Parte Imaginaria

1 0,5 0 −0,5 −1

−1,5

−1

−0,5

0

0,5

1

Parte Real

Las funciones sistema son:    π π 1 + 23 z −1 · 1 − ej 4 z −1 · 1 − e−j 4 z −1 1    , |z| > H1 (z) = k1 ·   2π 2π 1 −1 1 1 2 1 − 2z · 1 − 2 · ej 3 z −1 · 1 − 2 · e−j 3 z −1    π π 1 + 23 z −1 · 1 − ej 4 z −1 · 1 − e−j 4 z −1 1    , |z| >  H2 (z) = k2 ·  2π 2π 1 1 1 −1 2 j −j −1 −1 · 1− 2 ·e 3 z · 1− 2 ·e 3 z 1 − 2z

Conceptos Básicos y Tipos de Sistemas.

18 c)

H2 (z) = H0 (z) · Hap (z)    π π 1 + 32 z −1 · 1 − ej 4 z −1 · 1 − e−j 4 z −1    H0 (z) = k0 ·   2π 2π 1 − 12 z −1 · 1 − 21 · ej 3 z −1 · 1 − 21 · e−j 3 z −1  z −1 + 23  1 + 23 z −1

Hap (z) = k3

Problema 1.14 Para cada uno de los diagramas de polos y ceros siguientes, se pide indicar razonadamente si se pueden corresponder con:

a) Una función del tipo C(z) = H(z) · H ∗ ( z1∗ ).

b) Un sistema de fase mínima.

c) Un sistema paso todo.

d) Un filtro de fase lineal. En este caso, indicar el tipo de filtro de que se trata.

e) Ninguna de las anteriores.

Figura 1

Figura 2 2

1

Parte Imaginaria

Parte Imaginaria

2

2

0

−1

−2

1

0

2

6

−1

−2

−2

−1

0 1 Parte Real

2

−2,5 −2 −1,5 −1 −0,5 0 0,5 1 1,5 2 2,5 Parte Real

19

Figura 4

Figura 3 2 Parte Imaginaria

Parte Imaginaria

2

1

0

ROC: |z|>0,8

1 0,8

0

−1

−1

−2

−2

−2,5 −2 −1,5 −1 −0,5 0 0,5 1 1,5 2 2,5

−2,5 −2 −1,5 −1 −0,5 0 0,5 1 1,5 2 2,5 Parte Real

Parte Real

Figura 5

Parte Imaginaria

2

1

0

2

−1

−2 −2,5 −2 −1,5 −1 −0,5 0 0,5 1 1,5 2 2,5 Parte Real

Solución a) La figura 3. b) La figura 5. c) Ninguno. d) La figura 2, tipo I. e) La figura 1.

Conceptos Básicos y Tipos de Sistemas.

20

Problema 1.15 En las siguientes figuras se representan algunas características (diagramas de polos y ceros, respuestas en frecuencias o retardos de grupo) correspondientes a seis sistemas reales, causales y estables. Se pide, en función de dicha características, indicar razonadamente si cada uno de los mismos se pueden corresponder con sistemas de fase lineal, paso-todo, de fase mínima, o bien son funciones del tipo C (z) = H (z) · H ∗ (1/z ∗). Sistema 1

2,5

Sistema 2 1

2

0,8 0,6

1

Parte Imaginaria

Parte Imaginaria

1,5

0,5 0 −0,5

0,4 0,2 0

−0,2 −0,4

−1 −1,5

−0,6

−2

−0,8 −1

−2,5 −2

−1

0 1 Parte Real

2

3

−1

0 0, 5 Parte Real

1

Retardo de grupo del sistema 3

4

Retardo de grupo del sistema 4

2, 8 2, 6 Retardo de Grupo (muestras)

Retardo de Grupo (muestras)

−0,5

4

4

0

0,1 0,2 0,3 0,4 0,5 0,6 0,7 0,8 0,9 Frecuencia Normalizada ( ×π rad/muestras)

0,4

2, 4 2, 2 2 1, 8 1, 6 1, 4 1, 2 1

1

0

0,1 0,2 0,3 0,4 0,5 0,6 0,7 0,8 0,9 Frecuencia normalizada ( ×π rad/sample) Sistema 6

2, 5

Fase de la respuesta en frecuencia del sistema 5

1

2

0,2 1, 5

Parte Imaginaria

Fase (rad)

0 −0.2 −0,4 −0,6 −0,8

0, 5

8

0 −0,5 −1

−1

−1,5

−1,2 −1,4 0

1

−2

0,5

1 1,5 2 2,5 Pulsación (rad/muestra)

3

3,5

−2,5 −2

−1

0

Parte Real

1

2

3

21 Solución Considerando las figuras anteriores: El Sistema 1 es paso-todo, pues por cada cero ci existe un polo en una posición pi = 1/c∗i . El Sistema 2 es de fase mínima, pues todos los ceros ci y los polos pi están dentro de la circunferencia de radio unidad. El Sistema 3 es de fase lineal, ya que el retardo de grupo τg = − dφ(ω) es constante dω (φ (ω) es la función de fase). El Sistema 4 podría ser paso-todo, pues la fase es monótona decreciente, y por consiguiente, el retardo de grupo es siempre positivo. El Sistema 5 es de fase mínima, pues al ser el sistema causal y estable, y al tener una diferencia de fase de 0 a π de 0 radianes, tiene todos sus ceros en el interior de la circunferencia de radio unidad (los polos ya están en el interior por las características del sistema). El Sistema 6 es de fase lineal, pues los polos pi están todos en el origen y por cada cero ci existe otro cero cj en una posición cj = 1/ci . Es de tipo I, pues no existen ceros en z = 1 ni en z = −1. Problema 1.16 Obtener la función del sistema H(z), de un filtro FIR de fase lineal tipo √ II, de coeficientes reales y de orden mínimo cuya función del sistema se anule para z1 = 1 + 3j y z2 = 1. Solución

H (z) =

 −1 2

−1



1+z 1−z   1 −j π −1 3 × 1− e z 2

1 − 2e

jπ 3

z

−1



1 − 2e

−j π 3

z

−1



  1 j π −1 3 1− e z 2

Conceptos Básicos y Tipos de Sistemas.

22

Problema 1.17 La respuesta al impulso de la figura caracteriza a un filtro FIR. Indique razonadamente si la fase de la respuesta en frecuencia es lineal y el tipo de filtro del que se trata, así como la n respuesta del sistema a las señales de entrada x1 [n] = 1, ∀n y x2 [n] = (−1) , ∀n.

2 1.5 1

h[n]

0.5 0 −0.5 −1 −1.5 −2

0

2

4

6

8

10

12

n (índice de muestra) Solución Es un sistema de fase lineal tipo III. Las respuestas a las entradas x1 [n] y x2 [n] son, respectivamente, y1 [n] = 0 e y2 [n] = 0. Problema 1.18 Dado un filtro FIR de coeficientes reales y de longitud N , que cumple que h[n] 6= 0 para 0 ≤ n ≤ (N − 1) y h[n] = 0 fuera de ese intervalo. El sistema es de fase lineal si su respuesta al impulso satisface que h[n] = ±h[N − 1 − n] para 0 ≤ n ≤ (N − 1). Se pide: a) Deducir de la condición anterior la siguiente igualdad: H(z) = ±z −(N −1) · H(z −1 ) b) Indicar razonadamente cómo están distribuidos los polos y los ceros si el filtro cumple que h[n] = −h[N − 1 − n], y N es un número par. c) Indicar razonadamente si el sistema del apartado anterior puede ser un filtro paso-todo. d) Indicar razonadamente si el filtro del apartado b) es, de todos los que tienen el mismo módulo de la respuesta en frecuencia, el sistema de fase mínima. Solución b) Es un sistema FIR tipo IV, por lo tanto tiene al menos un cero en z = 1. Si aparecen más ceros en este punto, tienen que tener multiplicidad impar. El resto se distribuye en grupos de 4, por cada cero estará su recíproco y los conjugados de ambos, (z0 , z0−1 , z0∗ , (z0−1 )∗ ). Si existen ceros en el eje real y son diferentes de +1 y −1, se distribuyen por parejas con sus recíprocos. Si los ceros están en la circunferencia de radio

23 unidad y son diferentes de +1 y −1, se distribuyen por parejas con sus conjugados. Si aparecen ceros en z = −1, tienen que tener multiplicidad par. En cuanto a los polos, el sistema tiene N − 1 polos en el origen. c) No es paso todo, ya que tiene al menos un cero en z = 1. d) En general no es de fase mínima, ya que tiene ceros tanto en el interior como en el exterior de la circunferencia de radio unidad. Sin embargo, puede ser de fase mínima si todos los ceros están en la circunferencia unidad. Problema 1.19 La función del sistema de un filtro presenta dos ceros zi y dos polos pi en los siguientes puntos del plano z: z1

=

−0,9,

p1

=

0,9,

z2 = −1 − j, 1 1 p2 = − j. 2 2

a) Obtenga los restantes polos y ceros y la función sistema, de manera que el filtro H1 (z) tenga una respuesta al impulso real, sea un sistema estable de orden 3, y cumpla que H1 (z)|z=1 = 1. ¿Es el sistema de fase mínima? Razone la respuesta. b) Encuentre todas las funciones Hl (z) que tengan el mismo módulo de la respuesta en frecuencia –salvo una constante que no es necesario calcular– y se correspondan con sistemas causales de respuesta al impulso real. Represente para todos los sistemas obtenidos el diagrama de polos y ceros. c) A todos los filtros obtenidos se les introduce como señal de entrada una secuencia real v[n], obteniendose a las salidas las señales yj [n]. Sea wij [n] la energia parcial acumulada en el instante m de cada una de las secuencias de salida: wyj [m] =

m X

yj2 [n] .

n=0

Seleccione, de entre todos los filtros obtenidos en el apartado (a), el sistema Hk (z) que presente la mayor energía acumulada en el instante m, es decir, que satisface que: wyk [m] ≥ wyj [m] ∀k 6= j, ∀m. Obtenga la constante de ganancia para que se satisfaga que Hk (z)|z=1 = 1. d) Obtenga la función del sistema HA (z) de mínimo orden posible, que multiplicada por la del filtro obtenido en el apartado anterior: H(z) = HA (z) · Hk (z) proporcione como resultado la función del sistema H(z) de un filtro FIR de fase lineal.

Conceptos Básicos y Tipos de Sistemas.

24 Solución

a) El cero y polo que faltan son, respectivamente: z3 = −1 + j p3 =

1 1 + j 2 2

La función sistema es:    1 1 + 0,9z −1 1 − (−1 − j) z −1 1 − (−1 + j) z −1   , |z| > 0,9   H1 (z) = 190 (1 − 0,9z −1 ) 1 − 12 − 21 j z −1 1 − 21 + 12 j z −1

No es de fase mínima.

b) Hay que añadir los ceros y polos: 1 −1 1 z4 = −1 2 + 2 j, z5 = 2 − 2 j, z6 = 1 p4 = 1 + j, p5 = 1 − j, p6 = 0,9

−1 0,9

Las funciones sistema y los diagramas de polos y ceros son:

   1 − z1 · z −1 1 − z4 · z −1 1 − z5 · z −1 , |z| > 0,9 H2 (z) = k2 (1 − p1 · z −1 ) (1 − p2 · z −1 ) (1 − p3 · z −1 )

1

Parte Imaginaria

z

4

p

3

0,5 z

p

1

1

0 z

5

p

2

−0,5

−1 −1

−0,5

0 0,5 Parte Real

1

   1 − z1 · z −1 1 − z2 · z −1 1 − z3 · z −1 , |z| > 0,9 H3 (z) = k3 (1 − p1 · z −1 ) (1 − p2 · z −1 ) (1 − p3 · z −1 )

25 1

Parte Imaginaria

z

3

p

3

0,5

z

p

1

1

0 p

2

−0,5 z

2

−1 −1,5

−1

−0,5

0

0,5

1

1,5

Parte Real

   1 − z4 · z −1 1 − z5 · z −1 1 − z6 · z −1 H4 (z) = k4 , |z| > 0,9 (1 − p1 · z −1 ) (1 − p2 · z −1 ) (1 − p3 · z −1 ) 1

Parte Imaginaria

z

p

4

3

0,5 z

p

6

1

0 z

p

5

2

−0,5

−1 −1,5

−1

−0,5

0

0,5

1

1,5

Parte Real

H5 (z) = k5

   1 − z2 · z −1 1 − z3 · z −1 1 − z6 · z −1 , |z| > 0,9 (1 − p1 · z −1 ) (1 − p2 · z −1 ) (1 − p3 · z −1 )

1

Parte Imaginaria

z

3

p

3

0,5 z

p

6

1

0 p

2

−0,5 z

2

−1 −1,5

−1

−0,5

0 Parte Real

0,5

1

1,5

c) El sistema de fase mínima: H2 (z) =

   1 − z1 · z −1 1 − z4 · z −1 1 − z5 · z −1 1 · , |z| > 0,9 95 (1 − p1 · z −1 ) (1 − p2 · z −1 ) (1 − p3 · z −1 )

Conceptos Básicos y Tipos de Sistemas.

26 d) HA (z) =

  1 − z3 · z −1 1 − z6 · z −1    × 1 − p1 · z −1 1 − p2 · z −1 1 − p3 · z −1 1 − z2 · z −1



Problema 1.20 De una función R(z) = H(z) · H(z −1 ) se conoce que presenta un cero y un polo en los siguientes puntos del plano z: Cero: c1 = 0,5 + 0,5j, Polo: p1 = −0,5 − 0,5j. Se pide: a) Dibujar el diagrama de polos y ceros completo de la función R(z) de orden mínimo que cumple los datos del enunciado. Considere a partir de ahora que, además de las condiciones anteriores, se cumple que la transformada z inversa de R(z) es una secuencia real. Se pide: b) Añadir los polos y ceros necesarios para obtener la función R(z) de orden mínimo que cumple lo indicado. c) Encontrar todas las posibles funciones H(z) correspondientes a sistemas causales y estables que tengan la misma R(z) obtenida en el apartado anterior. d) Indicar de todas las funciones obtenidas en el apartado anterior que tienen el mismo módulo, cual es la que menor retardo de grupo presenta. e) Indicar de todas las funciones obtenidas en el apartado c), aquellas que se correspondan con sistemas paso todo. Razone la respuesta. f) Indicar de todas las funciones obtenidas en el apartado c), aquellas que se correspondan con sistemas causales de fase lineal. Razone la respuesta. Solución

a) Es necesario añadir los siguientes ceros y polos: c2 = 1 − j p2 = −1 + j

27 R(z) p2

1

Parte Imaginaria

0,8

c1

0,6 0,4 0,2 0

−0,2 p1

−0,4 −0,6 −0,8

c2

−1 −1

−0,5

b)

0 0,5 Parte Real

c3 = 0,5 − 0,5j,

p3 = −0,5 + 0,5j,

1

c4 = 1 + j, p4 = −1 − j.

c) Los diferentes sistemas son:   1 − c1 · z −1 1 − c3 · z −1 , (1 − p1 · z −1 ) (1 − p3 · z −1 )   1 − c1 · z −1 1 − c4 · z −1 , H2 (z) = k2 (1 − p1 · z −1 ) (1 − p3 · z −1 )   1 − c2 · z −1 1 − c4 · z −1 H3 (z) = k3 , (1 − p1 · z −1 ) (1 − p3 · z −1 )   1 − c2 · z −1 1 − c3 · z −1 , H4 (z) = k4 (1 − p1 · z −1 ) (1 − p3 · z −1 ) H1 (z) = k1

1 |z| > √ 2 1 |z| > √ 2 1 |z| > √ 2 1 |z| > √ 2

d) H1 (z). e) Ninguno. f) Ninguno. Problema 1.21 Un filtro H1 (z) de fase lineal tipo II, causal, y de respuesta al impulso real, presenta los siguientes ceros en puntos del plano z: c1 = 1, c2 = 0,5 + 0,5j Se pide: a) Completar el resto de polos y ceros de la función H1 (z) de orden mínimo que cumple los datos del enunciado. b) Dibujar el diagrama de polos y ceros de la función C(z) = H1 (z) · H1∗ ( z1∗ ). c) Encontrar todas las funciones Hl (z) que tengan el mismo módulo de la respuesta en frecuencia que H1 (z), salvo una constante que no es necesario calcular, y se correspondan con sistemas reales y causales. Dibujar para todos los sistemas obtenidos el diagrama de polos y ceros.

Conceptos Básicos y Tipos de Sistemas.

28

d) Descomponer el sistema H1 (z) en dos subsistemas, uno paso todo y otro de fase mínima: H1 (z) = Hap (z) · Hmin (z). Dibujar para cada uno de ellos el diagrama de polos y ceros.

Solución

a) El resto de ceros y polos son: c3 = 1 − j, c4 = −1, c5 = 0,5 − 0,5j, c6 = 1 + j, c7 = 1.

b) El diagrama de polos y ceros es:

1

c6

2

Parte Imaginaria

0,8 0,6

c2 2

0,4 0,2

−0,2

c1

7

2 c4

0

−0,4

4

c5 2

−0,6 −0,8 c3

−1 −1

−0,5

0 Parte Real

0,5

2 1

con 7 polos en el infinito.

c) Los diferentes sistemas son: 2  2 2 1 − c4 z −1 1 − c2 z −1 1 − c5 z −1 H2 (z) = k2 1 − c1 z −1 2  2 2 1 − c4 z −1 1 − c3 z −1 1 − c6 z −1 H3 (z) = k3 1 − c1 z −1

29 H (z) 3

1

1

0,8

0,8

0,4 0,2

c4

0

2

7

c1

−0,2 −0,4

2

c5

−0,6

0,2 0 −0,2 −0,6

−1

−1 0

c1 2

7 c4

−0,4 −0,8

−0,5

0,5

c3 2 −1

1

−0,5

Parte Real

0 0,5 Parte Real

1

d) Los sistemas son:

  1 − c6 z −1 1 − c3 z −1 Hap (z) = kap (1 − c2 z −1 ) (1 − c5 z −1 ) 2 2  2 1 − c5 z −1 1 − c4 z −1 1 − c2 z −1 Hm´ın (z) = km´ın 1 − c1 z −1

El diagrama de polos y ceros: Hap(z)

Hmin(z)

1

1

c6

0,8 0,6

c2

Parte Imaginaria

Parte Imaginaria

0,8 0,6 0,4 0,2 0 −0,2 −0,4

c5

−0,6

c2

0,4 0,2

c4

0

2

7

2

c1

−0,2 −0,4

c5

−0,6

2

−0,8

−0,8

c3

−1 −1

−0,5

0

Parte Real

0,5

2

0,4

−0,8 −1

c6

0,6

c2 2

0,6

Parte Imaginaria

Parte Imaginaria

H2(z)

−1

1

−1

−0,5

0

Parte Real

0,5

1

Conceptos Básicos y Tipos de Sistemas.

30

Problema 1.22 Un sistema LTI presenta una función de sistema tal que: H(z) = 1 + a · z −1 + b · z −2 + c · z −3 , a, b, c ∈ R Se sabe que la salida de dicho sistema ante una entrada: π   π  2√5 √ n + · sen n x [n] = 6 2 · cos 3 5 2

es:

y [n] = 2 · sen

π 2

n + φ0

a) Obtener todos los posibles valores de a, b y c.



b) Indicar razonadamente si el (los) filtro(s) obtenido(s) es (son) de fase lineal o no. En caso afirmativo indicar el tipo correspondiente. Solución

a) Una posibilidad es: a = −3, b = 3, c = −2 , o bien, a = 1, b = −1, c = 2. b) Ninguno es de fase lineal. Problema 1.23 Un sistema de tiempo discreto LTI causal tiene la siguiente función sistema:   1 − 41 z −1 1 + 4z −2  H (z) = . 1 − 41 z −2

Se pide:

a) Encontrar las expresiones para un sistema de fase mínima H1,m´ın (z) y un sistema de fase lineal Hlin (z) que cumplan: H (z) = H1,m´ın (z) Hlin (z) Representar el diagrama de polos y ceros de los tres sistemas. b) Encontrar las expresiones para un sistema de fase mínima H2,m´ın (z) y un sistema pasotodo Hap (z) que cumplan: H (z) = H2,m´ın (z) Hap (z) Representar el diagrama de polos y ceros de H2,m´ın (z) y Hap (z). c) Deducir razonadamente el valor que tiene que tomar α para que αn h [n] sea real y corresponda a un sistema de fase mínima.

Solución a) El diagrama de polos y ceros de los tres sistemas es:

31

H(z)

Parte Imaginaria

2 1 0

−1 −2 −3

−2

−1

0 1 Parte Real

2

3

H1,min(z)

Hlin(z) 1

Parte Imaginaria

Parte Imaginaria

2 1 4

0 −1 −2 −3

−2

−1

0

1

2

0.5

3

0 −0.5 −1 −1,5

3

−1

−0,5

Parte Real

0

0,5

1

1,5

Parte Real

b) El diagrama de polos y ceros es:

H2,min(z)

Hap(z) 1

Parte Imaginaria

Parte Imaginaria

2 1 0 −1 −2 −3

0,5 0

−0,5

−2

−1

0

1

2

−1 −1,5

3

−1

−0,5

0

0,5

Parte Real

Parte Real c) El valor de α debe valer: α ∈ R,

|α|


1

4/3 e jπ/4

Parte Imaginaria

0,8 0,6 0,4

3/4e j

0,2

π/4

2

0 −0,2

3/4e -jπ/4

−0,4 −0,6

-jπ/4

−0,8

4/3 e

−1 −1

− 0,5

Parte Real

0,5

1

c) h3 [n] = δ [n − 2] − 2 · δ [n − 4] + δ [n − 6] d) HT (z)

Retardo de grupo de HT (z)

1

Parte Imaginaria

π/4

0,6 0,4

3/4e j

π/4

0,2 0

2

2

8

−0,2 3/4e -j

−0,4

π/4

−0,6 −0,8

4/3e

-jπ/4

−1

4,06 Retardo de grupo (muestras)

4/3e j

0,8

4,04 4,02 4 3,98 3,96 3,94 3,92 3,9

−1

−0,5

0 0,5 Parte Real

1

0,1 0,2 0,3 0,4 0,5 0,6 0,7 0,8 0,9 1 Frecuencia Normalizada( ×π rad/muestra)

Problema 1.26 Un sistema causal de fase lineal Tipo IV de respuesta al impulso real presenta un cero en c1 = −0,5 + 0,5 · j. Se pide: a) Obtener todos los polos y ceros del sistema de orden mínimo que cumple los requisitos anteriores. b) Dibujar el diagrama de polos y ceros de dicho sistema. c) Dibujar el retardo de grupo. d) Añadir el menor número de ceros y polos posibles para que el sistema siga siendo causal y se convierta en un filtro de fase lineal: Tipo I. Tipo II. Tipo III.

Conceptos Básicos y Tipos de Sistemas.

36

Para cada uno de los anteriores, dibujar el diagrama de polos y ceros y el retardo de grupo. Solución a) El resto de ceros y polos son los siguientes Ceros → c2 = −0,5 − 0,5j, c3 = −1 + j, c4 = −1 − j, c5 = 1. Polos → p1 = 0 (orden 5). b) El diagrama de polos y ceros del sistema tipo IV es: Sistema Tipo IV c

Parte Imaginaria

1

3

c1

0.5

p 0

c5

5

1

c2

−0.5

c4

−1 −1

−0.5

0

0.5

1

Parte Real

c) El retardo de grupo del sistema tipo IV es: Sistema Dado (Tipo IV)

Retardo de grupo (muestras)

3.5

3

2.5

2

1.5 0

0.2

0.4

0.6

0.8

1

Frecuencia Normalizada (×π rad/muestra)

d) Los polos y ceros que hay que añadir se describen a continuación: Para el sistema Tipo I, se añade un cero c6 = 1, y otro polo en el origen. Para el sistema Tipo II, se añade un cero c6 = 1 y otro en c7 = −1, y dos polos en el origen. Para el sistema Tipo III, se añade un cero c6 = −1, y otro polo en el origen. Los diagramas de polos y ceros de cada uno de los sistemas se representan a continuación:

37 Sistema Tipo II

Sistema Tipo I c

c

1

0.5

p1

0

c5, c6

6

2

c

2

−0.5

−1

c1

0.5 c7

0

p1

0

0.5

c5, c6

2

c4

−1

−0.5

7

c2

−0.5

c4

−1

c3

1

3

Parte Imaginaria

Parte Imaginaria

1

1

−1

−0.5

0

0.5

1

Parte Real

Parte Real Sistema Tipo III c

Parte Imaginaria

1

3

c

1

0.5

p1

c6

0

c

6

5

c

2

−0.5

c4

−1 −1

−0.5

0

0.5

1

Parte Real

Por último, los retardos de grupo son los siguientes: Sistema Tipo I

Sistema Tipo II

Retardo de Grupo (muestras)

4.5

3.5

3

2.5

2 0

0.2

0.4

0.6

0.8

1

4

3.5

3

2.5 0

Frecuencia Normalizada (×π rad/muestra)

0.2

0.4

Sistema Tipo III

3.5

3

2.5

2 0

0.2

0.6

0.8

Frecuencia Normalizada (×π rad/muestra)

4

Retardo de Grupo (muestras)

Retardo de Grupo (muestras)

4

0.4

0.6

0.8

Frecuencia Normalizada (×π rad/muestra)

1

1

Conceptos Básicos y Tipos de Sistemas.

38

Problema 1.27 Dado un sistema FIR causal, de orden dos, con coeficientes reales y con un cero en c1 = −1/3 − 1/3 · j, se pide: a) Encontrar su función del sistema (H1 (z)), incluida la constante de ganancia, sabiendo que H1 (Ω)|Ω=0 = 1. b) Encontrar todas las Hi (z) de aquellos sistemas causales que tienen el mismo módulo de la respuesta en frecuencia que H1 (z). Calcule la constante de ganancia únicamente para aquellos Hi (z) que tengan coeficientes reales. c) De todos los sistemas obtenidos en el apartado anterior, ¿qué propiedad satisface H1 (z) con respecto a la energía parcial acumulada? Razone la respuesta. d) Partiendo de H1 (z), añada el menor número de polos y de ceros para obtener un nuevo sistema Hf l (z) que se corresponda con un sistema de fase lineal tipo IV (simetría HA). e) Dibuje el retardo de grupo de Hf l (z). Solución a) H1 (z) = b)

  9 · 1 − (−1/3 − 1/3 · j) z −1 · 1 − (−1/3 + 1/3 · j) z −1 17

  H2 (z) = k2 · 1 − (−1/3 − 1/3 · j) z −1 · 1 − (−3/2 + 3/2 · j) z −1   H3 (z) = k3 · 1 − (−3/2 − 3/2 · j) z −1 · 1 − (−1/3 + 1/3 · j) z −1

H4 (z) =

  2 · 1 − (−3/2 − 3/2 · j) z −1 · 1 − (−3/2 + 3/2 · j) z −1 17

c) Como es un sistema de fase mínima, tiene la propiedad de retardo de energía mínimo. d)

  9 Hf l (z) = 17 · 1 − (−1/3 − 1/3 · j) z −1 · 1 − (−1/3 + 1/3 · j) z −1    × 1 − (−3/2 − 3/2 · j) z −1 · 1 − (−3/2 + 3/2 · j) z −1 · 1 − z −1

e) El retardo de grupo es el siguiente:

Retardo de grupo (muestras)

3.5

3

2.5

2

1.5 0

0.2

0.4

0.6

0.8

Frecuencia Normalizada (×π rad/muestra)

1

39 Problema 1.28 Dado un filtro FIR causal, de longitud L, de fase lineal tipo III, se pide demostrar que sus funciones de amplitud A (Ω) y de fase θ (Ω) vienen dadas por las expresiones siguientes: (L−3)/2

X

A (Ω) = 2

n=0

   L−1 −n , h [n] sen Ω 2

π θ (Ω) = − 2



L−1 2



· Ω,

donde h [n] representa la respuesta al impulso de dicho sistema. Solución La respuesta en frecuencia del sistema se puede expresar del siguiente modo: H (Ω) =

L−1 X n=0

(L−3)/2

h [n] · e

−jΩn

X

=

n=0

h [n] · e

−jΩn

 L − 1 −jΩ L−1 2 + ·e +h 2 

L−1 X

n=(L+1)/2

h [n] · e−jΩn .

Al ser el sistema de fase lineal tipo III, cumple las siguientes dos condiciones: h [n] = −h [L − 1 − n] y h [(L − 1)/2] = 0. Por tanto, se tiene: (L−3)/2

X

H (Ω) =

n=0

h [n] · e−jΩn −

L−1 X

h [L − 1 − n] · e−jΩn .

n=(L+1)/2

Si en el segundo sumatorio se realiza el cambio de variable p = L − 1 − n, se obtiene: (L−3)/2

H (Ω) =

X

n=0

(L−3)/2

h [n] · e−jΩn −

X p=0

h [p] · e−jΩ(L−1−p) ,

expresión que es equivalente a: (L−3)/2

H (Ω) = j2e

−jΩ (L−1) 2

·

X

n=0

h [n] ·

ejΩ

(L−1) 2

e−jΩn − e−jΩ 2j

(L−1) 2

ejΩn

!

.

Claramente, esta última permite mostrar la respuesta en frecuencia como se indica en el enunciado del problema: (L−3)/2

H (Ω) = 2

X

n=0

   (L−1) π (L − 1) h [n] · sen Ω −n · ej ( 2 − 2 Ω) = A (Ω) · ejθ(Ω) . 2

Capítulo 2

Muestreo y Reconstrucción de Señales Problema 2.1 Dado el sistema de la figura:

x(t)

y(t)

xm(t) Sistema

x

p(t) Se pide: a) Obtener la expresión de P (ω) y representar dicha función para −3,5 ≤ ω ≤ 3,5. b) Representar Xm (ω) para −2 ≤ ω ≤ 2. c) Obtener y representar la respuesta en frecuencia de un sistema LTI que obtenga a su salida la señal de entrada, es decir, y (t) = x (t). P Datos: x (t) = cos (1, 5 · t), p (t) = ∞ n=−∞ q (t − 2πn)

q(t)

π

−π/4

π/4

t

Muestreo y Reconstrucción de Señales.

42 Solución a) P (ω) = 2π

∞ P

k=−∞

sen(kπ/4) ·δ (ω k

− k).

P(ω) π2/2 1,41π

1,41π

π

π

0,47π

0,47π

...

-3

-2

-1

1

2

... ω

3

b) Xm (ω) :

Xm(ω) 1,20π

1,20π

1,02π

1,02π

...

... -1,5

c) H (ω) =



-0,5

0,5

1,5

0,9793, ωc1 < |ω| < ωc2 , , 0, resto

donde: 0,5 < ωc1 < 1,5 y 1,5 < ωc2 < 2,5.

H(ω)

0,97

−ωc2

−ωc1

0,97

ωc1

ωc2 ω

ω

43 Problema 2.2 El proceso de reconstrucción de una señal de tiempo continuo a partir de una de tiempo discreto se puede dividir en dos etapas, tal y como se representa en el diagrama de bloques de la figura 2.1. El primer bloque convierte las muestras discretas en una suma de deltas de Dirac ponderadas por el valor de dichas muestras, y el segundo bloque es el filtro de reconstrucción.

x[n]

n

xr (t)

t

hr (t)

xc (t)

Ts Figura 2.1

Considere que la secuencia de entrada al sistema de la figura es la siguiente: ( |n| (1/2) , −2 ≤ n ≤ 2, x [n] = 0, resto, Se pide: a) Obtener la expresión de xc (t) en función de Ts , x [n] y hr (t). b) Representar la secuencia de salida resultante xc1 (t) cuando la función hr1 (t) es la respuesta al impulso correspondiente al filtro ideal, es decir, Hr1 (ω) =



Ts , |ω| < π/Ts , 0, resto.

c) Representar la secuencia de salida resultante xc2 (t) cuando la función hr2 (t) es la respuesta al impulso correspondiente al sistema interpolador “zero-order-hold”: hr2 (t) =



1, 0 ≤ t ≤ Ts , 0, resto,

d) Representar la secuencia de salida resultante xc3 (t) cuando la función hr3 (t) es la respuesta al impulso correspondiente al sistema interpolador “first-order-hold”:

hr3 (t) =

 

t/Ts , 0 ≤ t ≤ Ts , 2 − t/Ts , Ts ≤ t ≤ 2 · Ts ,  0, resto.

e) ¿Es posible obtener xc1 (t) a partir de las señales xc2 (t) y xc3 (t)? En caso de responder afirmativamente, describir brevemente el procedimiento. Solución a) xc (t) =

∞ P

k=−∞

x [k] · hr (t − kTs )

b) La secuencia xc1 (t) es:

Muestreo y Reconstrucción de Señales.

44

xc1(t) 1

0,5

0

−0,5

-4Ts -3Ts -2Ts -Ts

Ts 2Ts 3Ts 4Ts 5Ts

0 t

c) La secuencia xc2 (t) es:

xc2(t) 1 0,8 0,6 0,4 0,2 0

-2Ts

-Ts

Ts

0 t

2Ts

3Ts

3Ts

4Ts

d) La secuencia xc3 (t) es:

xc3(t) 1 0,8 0,6 0,4 0,2 0

-2Ts

-Ts

0

e) Es posible, usando el siguiente sistema: ( Hi (ω) =

t

Ts

Ts Hr (ω) ,

0,

2Ts

|ω| < Tπs , resto.

45 Problema 2.3 Un modelo simple de canal de comunicaciones multicamino selectivo en frecuencia emplea un sistema LTI cuya respuesta al impulso viene dada por la siguiente expresión: h (t) =

L X √ pk · δ (t − τk ) k=1

Se desea simular los efectos de dicho canal empleando el sistema de la figura 2.2, en el que el convertidor D/C contiene un filtro paso bajo de ganancia T y pulsación de corte π/T .

xc(t)

C/D

xd[n] = xc(nT)

yd [n]

Hd(Ω)

yc (t) D/C

T

T Figura 2.2

Considere que la señal xc (t) está limitada en banda y que T toma un valor adecuado para evitar el solapamiento. Empleando un canal de 4 caminos (L = 4) cuyos valores de retardo τk y potencia pk vienen indicados en la tabla adjunta, se pide: a) Determinar la respuesta en frecuencia del sistema de tiempo discreto Hd (Ω) en función de T . b) Obtener la respuesta al impulso del sistema discreto que simula al canal, considerando: T = 10 ns T = 50 ns Retardo (ns) τ1 = 0 τ2 = 110 τ3 = 190 τ4 = 410

Potencia p1 = 1 p2 = 0,1072 p3 = 0,012 p4 = 0,0052

Solución a) La respuesta en frecuencia es: Ω





Hd (Ω) = 1 + 0,3274e−j T 110 + 0,1095e−j T 190 + 0,0721e−j T 410 b) La respuesta al impulso es: T = 10 ns hd [n] = δ[n] + 0,3274δ[n − 11] + 0,1095δ[n − 19] + 0,0721δ[n − 41] T = 50 ns sen (π (n − 11/5)) + π (n − 11/5) sen (π (n − 41/5)) sen (π (n − 19/5)) + 0,0721 +0,1095 π (n − 19/5) π (n − 41/5)

hd [n] = δ[n] + 0,3274

Muestreo y Reconstrucción de Señales.

46

Problema 2.4 Dado el sistema de la figura 2.3 para procesar la señal de tiempo continuo xc (t) cuyo espectro se representa en la figura 2.4. Considere que el convertidor D/C contiene un filtro paso bajo ideal, de ganancia T y de pulsación de corte π/T .

xc(t)

x[n] = xc(nT) C/D

yc (t)

y[n] Hd(Ω)

D/C

T

T Figura 2.3

XC(ω) 1

ω Se pide:

Figura 2.4

a) Obtener el valor de T , sabiendo que es la mitad del máximo periodo de muestreo posible para evitar el solapamiento. b) Representar el espectro correspondiente a las señales x [n], y [n] e yc (t). c) Obtener la expresión de la respuesta en frecuencia del sistema analógico equivalente al dado en la figura 1.  j, −π < Ω < 0 Datos: ω1 = 2π · 103 rad/s, Hd (Ω) = y es periódica con periodo 2π. −j, 0 < Ω < π

47 Solución a) T =

1 4

· 10−3 s.

b) Los espectros son:

|X(Ω)| 1/T

−π

−π/2

π/2

π Ω

|Y(Ω)|

Arg{Y(Ω)} π/2

1/T

π/2

π Ω

−π −π/2 −π −π/2

π/2

π

−π/2

Ω

Arg{Y(ω)}

|Y(ω)| π/2 1

ω1 −ω1

−ω1 c)

ω1

ω

ω −π/2

 3  j, −4π · 10 < ω < 0 Heq (ω) = −j, 0 < ω < 4π · 103  0, |ω| > 4π · 103

Muestreo y Reconstrucción de Señales.

48

Problema 2.5 El sistema de la figura 2.5 trabaja con un periodo de muestreo T y el convertidor D/C contiene un filtro paso-bajo ideal de ganancia T y frecuencia de corte 1/(2T ) Hz.

x[n] = xc(nT)

xc(t) C/D

y [n] Filtro Digital

yc (t) D/C

T

T Figura 2.5

Sabiendo que el filtro digital es LTI y causal, y viene caracterizado por la siguiente ecuación en diferencias: y [n] = x [n] + x [n − 1] , se pide: a) Obtener Hd (Ω) = Y (Ω) /X (Ω), representar gráficamente su diagrama de polos y ceros y el módulo de la respuesta en frecuencia de manera aproximada. b) Representar gráficamente el módulo de los espectros de xc (t), x [n], y [n] e yc (t) cuando se aplica como señal de entrada  π   π · t + cos ·t xc (t) = cos 2T T c) Obtener la expresión de yc (t). d) Calcular la respuesta en frecuencia del sistema analógico equivalente al de la figura. Solución a) Hd (Ω) = 1 + e−j·Ω 2

0,5

|H(Ω)|

Parte Imagianria

1

0

1

−0,5 −1 −1

−0,5

0 0,5 Parte Real

b) El módulo de los espectros es:

1

0

−π

0

Ω(rad)

π

49

|Xd(Ω)|

XC(ω)

2π π

π

−π/T −π/(2T)

ω

π/(2T) π/T

−π

−π/2

π/2 |YC(ω)|

|Y(Ω)| 2· π

−π

−π/2

π Ω

2· π

π/2

π



−π/T −π/(2T)

π/(2T) π/T

ω

√ π · t − π4 ). 2 · cos( 2T  1 + e−j·ω·T |ω| < π/T d) Heq (ω) = 0 resto c) yc (t) =

Problema 2.6 El sistema de la figura 2.6 trabaja con un periodo de muestreo T = π2 s y el convertidor D/C contiene un filtro paso-bajo ideal de ganancia T y frecuencia de corte 1/(2T ) Hz.

xc(t)

x[n] D/C

Filtro Analógico

T

yc (t)

y[n] = yc (nT) C/D T

Figura 2.6

Sabiendo que el filtro analógico es LTI y causal, y viene caracterizado por la siguiente ecuación diferencial: dyc (t) dxc (t) − xc (t) − 2 , yc (t) = 2 dt dt se pide: a) Obtener la respuesta en frecuencia del filtro analógico y representar gráficamente su módulo. b) Representar el diagrama de polos y ceros de dicho sistema analógico. c) Dibujar el módulo de los espectros de x [n], xc (t), yc (t) e y [n] cuando se aplica como señal de entrada π  ·n . x [n] = cos 4

d) Obtener las expresiones de xc (t), yc (t) e y [n].

Muestreo y Reconstrucción de Señales.

50 Solución a) Hc (ω) = −

1 − 2jω 1 + 2jω

|Hc(ω)| 1

ω

b) El diagrama de polos y ceros es: 1

Parte Imaginaria

Re{s} > -1/2

0,5 0 −0,5 −1 −1

−0,5

0

0,5

1

Parte Real

c) El módulo de los espectros es:

|X(Ω)|

|Yc(ω)|

π

−π

−π/4

π

π/4

π



-1/2

|Xc(ω)|

d) xc (t) = cos

1 2t

ω

|Y(Ω)| π

π

-1/2

1/2

1/2

 , yc (t) = sen

ω 1 2t

−π

 , y [n] = sen

−π/4  .

π 4n

π/4

π



51 Problema 2.7 Considérese un sistema LTI cuya respuesta impulsiva se puede expresar como h [n] = g [n] ∗ g [n], donde g [n] = u [n] − u [n − 3]. Este filtro se emplea para formar parte del diagrama de bloques de la figura 2.7, en el que la señal de entrada se muestrea de tal manera que no se produce solapamiento.

xc(t)

x[n]

yc(t)

y[n]

C/D

h[n]

D/C

T

T Figura 2.7

Se pide: a) Determinar el diagrama de polos y ceros de h[n] y representar de forma aproximada el módulo de su respuesta en frecuencia. b) Sabiendo que la señal de entrada y de salida al sistema de la figura 2.7 son respectivamente:   6 6 xc (t) = 1 + cos 3π 2 · 10 t + cos 2π · 10 t 6 yc (t) = A + B · cos 3π 2 · 10 t + φ

determinar los valores de T , A, B y φ. Solución

a) El diagrama de polos y ceros y el módulo de la respuesta en frecuencia es: 9

2

8

0,5

6

2π/3 |H(Ω)|

Parte Imaginaria

1

4

0 −0,5

2

−1 −1,5

2 −1

−0,5

0

0,5

1

1,5

Parte Real

b) T =

4

1 3

· 10−6 s, A = 9, B = 1, φ = −π rad.

0

−π

−2π/3

0

Ω (rad)

2π/3

π

Muestreo y Reconstrucción de Señales.

52

Problema 2.8 Se desea utilizar el sistema de la figura 2.8 para procesar xc (t), cuyo espectro se dibuja en la figura 2.9 y del que sólo resulta de interés la banda de frecuencias inferiores a 1,5 kHz. Considere que el convertidor D/C contiene un filtro paso bajo ideal, de ganancia T2 y de pulsación de corte π/T2 .

xc(t)

C/D

y [n]

x[n] = xc(nT) h[n]

yc (t) D/C

h*[-n]

T1

T2 Figura 2.8

X c (ω) 1 0,7

−9π

−6π −3π







ω (krad/s)

Figura 2.9

Se pide: a) Calcular el valor de T1 , sabiendo que debe ser lo mayor posible, y dibujar el espectro de x[n]. b) Obtener razonadamente la respuesta al impulso h[n] del sistema LTI, sabiendo que es una función real y que su transformada de Fourier (módulo y fase) viene parcialmente dada por la siguiente expresión: |H (Ω)| =



1 2,

0,

0 ≤ Ω ≤ π/2, π/2 < Ω ≤ π,

φH (Ω) = −16 · Ω, 0 ≤ Ω ≤ π.

c) Obtener la respuesta al impulso de la asociación en cascada de los dos sistemas LTI, es decir, obtener z [n] = h [n] ∗ h∗ [−n]. d) Representar los espectros de y [n] e y (t) , sabiendo que T2 = T1 /2. Solución a) T1 = 61 10−3 s.

X(Ω) 1,4/T1 1/T1 0,7/T1

−π

−π/2

π/2

π



53 b)

 sen π2 (n − 16) . h [n] = 2π (n − 16)

c) z [n] =

 sen n π2 . 4πn

d) Los espectros son:

−π

Y(Ω)

Y(ω)

1/(4T1)

1/8

0,7/(4T1)

0,7/8

π/2

−π/2

π



−6π



ω(krad/s)

Problema 2.9 Al sistema de la figura 2.10 se le introduce como entrada la secuencia cuyo espectro de periodo 2π se representa en la figura 2.11. Considerando que h0 (t) y h1 (t) son sistemas LTI estables, se pide: a) Representar los espectros de yd (t) e yp (t) , considerando que h0 (t) =



1, 0 ≤ t ≤ T, 0, resto.

b) Obtener la expresión de la respuesta en frecuencia del sistema h1 (t), sabiendo que el filtro equivalente hi (t) viene caracterizado por la siguiente expresión: hi (t) =

sen ((πt)/T ) (πt)/T

c) Representar el espectro de yc (t).

hi (t) y[n]

n

t

yd (t)

h0 (t)

T Figura 2.10

yp (t)

h1 (t)

yc (t)

Muestreo y Reconstrucción de Señales.

54

Y(Ω) 1

0,51π



0,51π

-0,5π

0,5π

π



Figura 2.11

Solución a) Los espectros son:

Yd(ω) 0,51π Τ

0,51π Τ

1

-0,5π Τ

-2π Τ

2π ω Τ

0,5π Τ

|Yp(ω)| T

-2π Τ

-0,5π Τ

2π Τ

0,5π Τ

ω

Arg{Yp(ω)}

-0,5π Τ

-2π Τ

b) H1 (ω) =

c) El espectro de yc (t) es:

(

2π Τ

0,5π Τ

Tω 2sen(ωT /2)

0

· ejωT /2

|ω| < π/T resto

ω

55

Yc(ω) 0,51π Τ

0,51π Τ

T

-0,5π Τ

ω

0,5π Τ

Problema 2.10 Se desea simular un sistema LTI causal y analógico cuya relación entrada-salida viene dada por d2 xc (t) . yc (t) = dt2 Para ello, se emplea el diagrama de bloques representado en la figura 2.12, donde el convertidor D/C contiene un filtro paso bajo ideal de pulsación de corte π/T y ganancia T .

xc(t)

xd[n] = xc(nT)

Hd(Ω)

C/D

yc (t)

yd[n] D/C

T

T Figura 2.12

Sabiendo que T = 1 ms y que el ancho de banda de las señales a procesar no alcanza los 250 Hz se pide: a) Determinar el ancho de banda analógico efectivo que es posible simular con el sistema de la figura 2.12. b) Obtener Hd (Ω) y representar de forma aproximada el módulo y la fase de su respuesta en frecuencia. c) Si a la entrada del sistema se aplica la señal xc (t) = cos (80πt), dibujar el espectro de xd [n] y calcular la respuesta del sistema discreto yd [n]. Solución a) ωc = π · 103 rad/s. b) Hd (Ω) =

  



 Ω 2 , T

  periódica 2π

|Ω| < π,

Muestreo y Reconstrucción de Señales.

56

(π/Τ)2

|Hd(Ω)| Arg{Hd(Ω)} π

−π

π

π

−π





−π

c) El espectro de xd [n] es:

Xd(Ω) π

−π

−0,08π 0,08π

 La respuesta del sistema: yd [n] = − 6400π 2 cos

π 8π 100 n

Ω



Problema 2.11 Dado el sistema de la figura 2.13, en el que el convertidor D/C contiene un filtro paso bajo de ganancia T y pulsación de corte π/T , se pide: a) Determinar la respuesta en frecuencia del sistema de tiempo discreto. b) Obtener la respuesta en frecuencia del sistema analógico equivalente al dado. c) Considerando Ω1 = 0 y que todas las señales que se aplican a su entrada están limitadas en banda a π/(4T ), se pide obtener la relación entrada-salida en el dominio del tiempo de un sistema analógico equivalente al dado.

xd[n] = xc(nT)

xc(t) C/D

Hd(Ω)

yc (t)

yd[n] D/C

T

T Figura 2.13

Dato: hd [n] = δ [n] +

sen ((π/3) (n − n0 )) jΩ1 n ·e (π/3) (n − n0 )

Solución a)

 1 + 3e−j(Ω−Ω1 )n0 , − π3 + Ω1 ≤ Ω ≤ π3 + Ω1 ,    1, −π < Ω < − π3 + Ω1 , Hd (Ω) = π  1,  3 + Ω1 < Ω < π,  periódica 2π resto de valores

57 b)

 π π + ΩT1 ≤ ω ≤ 3T + ΩT1 , 1 + 3e−j(ωT −Ω1 )n0 , − 3T    π + ΩT1 , 1, − Tπ < ω < − 3T Heq (ω) = Ω1 π π  1,  3T + T < ω < T ,  π 0, |ω| > T .

c)

yc (t) = xc (t) + 3xc (t − n0 · T ) Problema 2.12 Al sistema de la figura 2.14 se le introduce como entrada la señal cuyo espectro se representa en la figura 2.15. El filtro h[n] tiene el espectro de período 2π representado en la figura 2.15. Considerando que el convertidor ideal C/D tiene un período de muestreo T que es la mitad del período máximo para evitar el solapamiento, se pide: a) Obtener el valor de T en función de T1 . b) Representar los espectros de todas las señales que aparecen en la figura 2.14, sabiendo que el bloque Re{·} obtiene como salida p0 [n] la parte real de y0 [n].

ejπn/2 x0[n] jπn/4

x(t)

h[n]e

x[n]

p0[n]

y0[n]

2Re{}

X

e- jπn/2

C/D

h[n]e- jπn/4

T

x1[n]

X

y1[n]

p1[n]

+

Figura 2.14

H(Ω)

X(ω)

T

1

−2π/Τ1

2π/Τ1

ω

−π Figura 2.15

−π/4

π/4

π



Muestreo y Reconstrucción de Señales.

58 Solución a) T =

T1 4 .

b) Los espectros son:

X(Ω)

H(Ω+π/4)

1/T

−2π −3π/2 −π −π/2

T

π/2

π 3π/2 2π

−2π

Ω

π/2 π 3π/2 2π

−π −π/2

X1(Ω)

H(Ω−π/4)

1

T

−2π −3π/2 −π −π/2

π/2

π



−2π

Ω

−π −π/2

π/2

π/2

π



Ω

−2π

π/2 π

−π −π/2

3π/2 2π

Ω

π 3π/2 2π

Ω

P1(Ω)

Y0(Ω)

1

1

−3π/2 −π −π/2

Ω

1

1

−2π −3π/2 −π −π/2

π 3π/2 2π

Y1(Ω)

X0(Ω)

−2π

Ω

π/2

π



Ω

−2π −3π/2 −π −π/2

P0(Ω) 1

−2π −3π/2 −π −π/2

π/2

π

3π/2 2π



π/2

59 Problema 2.13 Una señal digital se transmite a través de un canal de comunicaciones LTI mediante el sistema de la figura 2.16. Los convertidores C/D y D/C son ideales, el convertidor D/C contiene un filtro ideal de ganancia T y frecuencia de corte π/T . Transmisor

Receptor

x[n] D/C

xc(t)

Canal H(ω)

yc (t)

T

y[n] C/D T

Figura 2.16

El canal de comunicaciones es modelado mediante un filtro cuya respuesta en frecuencia es: H (ω) =



1, |ω| < 20πrad/s 0, |ω| > 20πrad/s.

Se pide: (a) Asumiendo una perfecta sincronía entre el convertidor C/D y D/C, determinar razonadamente los valores de T para los cuales y[n] = x[n]. Razonar si el sistema completo es LTI. (b) Considere para este apartado T = 1/40 s y una señal de entrada con el espectro de la figura 2.17. Representar el espectro de las señales xc (t), yc (t), y[n].

1

−π −π/2

X(Ω)

π/2 π Ω

Figura 2.17

(c) Obtener el sistema digital equivalente Solución (a) La limitación en las frecuencias viene dada por el canal de comunicaciones, por lo tanto el periodo de muestreo debe cumplir: T ≥ 1/20 s. El sistema es LTI porque el canal es LTI y en ningún caso se produce solapamiento en frecuencia. (b) El espectro de las señales es:

Muestreo y Reconstrucción de Señales.

60

Τ

1

−π/2

Τ

20π 40π ω

−40π −20π

−π

Xc(ω)

Yc(ω)

−40π −20π

20π 40π ω

Y(Ω)

π/2

π Ω

(c) Si T < 1/20 s: Hef f (Ω) =



1 |Ω| < 20πT 0 |Ω| > 20πT ,

si T > 1/20 s: Hef f (Ω) = 1 Problema 2.14 Sea un sistema como el de la figura 2.18, donde el convertidor D/C contiene un filtro paso bajo ideal, de ganancia T2 y con pulsación de corte π/T2 .

xa(t)

C/D

ya (t)

y[n] = x2[n]

x[n] (.)2

T1

D/C T2

Figura 2.18

Los periodos de muestreo son T1 = 1ms, T2 = 2ms y la señal de entrada es: xa (t) =

sen(103 πt) . t

Se pide: a) Calcular la transformada de Fourier de las señales x[n], y[n], ya (t) y representarlas gráficamente. b) Discutir razonadamente la existencia de un sistema analógico equivalente, y en su caso, calcular dicho sistema equivalente.

61 Solución a) X(Ω) = 103 π, 103π

X(Ω)

π Ω

−π

Y (Ω) = 106 π 2 , 106π2

Y(Ω)

π Ω

−π

Ya (ω) =



2π 2 103 0

2π2103

|ω| < π/2 krad/s resto

Ya(ω)

π/2 ω(krad/s)

−π/2

b) No existe un sistema analógico equivalente Problema 2.15 cn Sea un sistema como el de la figura 2.19, donde la señal de entrada es x[n] = senΩ y el πn convertidor D/C contiene un filtro paso bajo ideal, de ganancia T y con pulsación de corte π/T :

x[n] D/C

xa(t)

T

y[n] C/D N .T

Figura 2.19

Se pide: a) Considerando Ωc = π4 y N = 2, calcular y representar gráficamente la transformada de Fourier de las señales x[n], xa (t), y[n]. b) En las condiciones del apartado anterior, calcular la relación entrada/salida del sistema completo en el dominio del tiempo. c) Suponer en este apartado que 0 ≤ Ωc ≤ π y N ∈ N. Establecer razonadamente la relación entre Ωc y N para que se pueda recuperar la entrada a partir de la salida del sistema.

Muestreo y Reconstrucción de Señales.

62 Solución a) Las transformadas son: X(Ω) 1

−π

−π/2 −π/4

π/4 π/2

π Ω

Xa(ω) T

−π/(4Τ)

π/(4Τ)

ω

Y(Ω) 1/2

−π

−π/2 −π/4

π/4 π/2

π Ω

b) y[n] = x[nN ] c) Ωc
25π

obtener razonadamente la expresión de la respuesta en frecuencia de las señales x[n], y[n] e ya (t). Representar gráficamente los espectros. Solución a) T < 1/50 s Hd (Ω) = 2π.

−jT Ω

+ πT δ (Ω) , cuando |Ω| ≤ π y en el resto de valores periódico con periodo

b) Las expresiones son:  1/T |Ω| < π/4 X (Ω) = y en el resto de valores periódico con periodo 2π. 0 π/4 < |Ω| < π  −j |Ω| ≤ π4 Ω + πδ (Ω) y en el resto de valores periódico con periodo Y (Ω) = 0 π/4 < |Ω| < π 2π.  −j ω + πδ (ω) |ω| ≤ 25π Ya (ω) = 0 resto

65

X(Ω) 1/Τ

−π

−π/4

π/4

π



|Y(Ω)|

π/2 −π

−π

−π/4

π/4

π

Arg{Y(Ω)}

−π/4

π/4



|Ya(ω)|

π/2

25π



Arg{Ya(ω)}

−25π −25π

π

−π/2

25π

ω

−π/2

ω

Problema 2.18 Sea un sistema como el de la figura 2.20, donde la señal de entrada tiene el espectro representado en la figura 2.21. En la figura 2.20, el convertidor C/D es ideal y trabaja a la mínima frecuencia de muestreo para que la señal de entrada pueda recuperarse a partir de las muestras, x[n]. El convertidor D/C es ideal y contiene un filtro paso bajo ideal, de ganancia T y con pulsación de corte π/T :

xa(t)

C/D

x[n]

Hd(Ω)

y[n] D/C

T

ya (t)

T Figura 2.20

Xa(ω) 1

5 7.5 10

ω (krad/s)

Figura 2.21

a) Determinar razonadamente la frecuencia de muestreo de los convertidores y representar X(Ω) en esas condiciones. b) El sistema Hd (Ω) está definido como: Hd (Ω) =



j −j

−π < Ω < 0 , 0 0,9 Solución H (z) =

1 − z −2 , |z| > 0,9372 1 + 0,8783 · z −2

y [n] = x [n] − x [n − 2] − 0,8783 · y [n − 2] Problema 4.1.2 Se desea diseñar un filtro notch causal y estable con las siguientes características: Frecuencia a eliminar: 0,05π radianes. Ancho de la banda eliminada a 3 dBs: ±0,01π radianes H (z) |z=1 = 1 Utilizando el método de ubicación de polos y ceros: a) Obtener la función del sistema del filtro de orden dos, considerando que la respuesta al impulso es real. b) Representar de manera aproximada el módulo de la respuesta en frecuencia. c) Dibujar, empleando diagramas de flujo, una realización del filtro según la forma directa I traspuesta.

Filtros Digitales.

92

d) Contabilizar en dicha estructura el número de registros de almacenamiento necesarios para coeficientes y datos, así como el de sumas (de dos en dos términos) y multiplicaciones (no considere las multiplicaciones por la unidad) que se realizan por cada muestra de salida. NOTA: suponga que la relación entre el módulo de los polos (r) y el ancho de banda a 3 dBs (∆Ω) es: r ≈ 1 − ∆Ω/2, r > 0,9 Solución a) H (z) = 1,0121 ·

1 − 1,9784 · z −1 + z −2 , |z| > 0,9686 1 − 1,9133 · z −1 + 0,9382 · z −2

b) El módulo de la respuesta en frecuencia es:

|H(Ω)| 1,4 1,2 1 0,8 0,6 0,4 0,2 0 -1

-0,8 -0,6 -0,4 -0,2

0

0,2

0,4

0,6

0,8

1 Ω/π

c) El diagrama de flujo:

k x[n] z

-1

z

a1

-1

z

z a2

donde: a1 = 1,9133 a2 = −0,9382

b1

-1

-1

y[n]

4.1. Diseño de Filtros Digitales IIR

93

b1 = −1,9784 065k = 1,0121 d)

• Registros: ◦ Coeficientes: 4

◦ Unidades de retardo: 4

• Operaciones:

◦ Sumas: 4

◦ Productos: 4 Problema 4.1.3 La función del sistema que se indica a continuación ha sido diseñada a partir de un filtro analógico con un periodo de muestreo Td = 2.

Hd (z) =

1 1−

e−0,8

·

z −1

+

2 1+

e−0,4

· z −1

Se pide: a) Obtener la función sistema del filtro analógico Hc (s) sabiendo que el filtro digital se ha obtenido mediante: El método de la respuesta al impulso invariante. La transformación bilineal. En ambos casos, ¿es única la solución? Justifique la respuesta. b) Representar los diagramas de flujo de las realizaciones del filtro digital empleando: La forma directa I Estructuras de orden uno conectadas en paralelo. Estructuras de orden uno conectadas en cascada. Solución a) • El método de la respuesta al impulso invariante: Hc (s) =

1 1 1 · + 2 s + 0,4 s + 0,2 − jπ/2

Filtros Digitales.

94 • La transformación bilineal: Hc (s) =

2 (1 + s) 1+s + 1 − e−0,8 + s (1 + e−0,8 ) 1 + e−0,4 + s (1 − e−0,4 )

La solución no es única en el caso de la respuesta al impulso invariante y sí es única en el caso de la transformación bilineal. b) • La forma directa I:

3 x[n]

z-1

y[n]

z-1

a1 b1

z-1 -a2

donde:

b1 = e−0,4 − 2e−0,8 a1 = −e−0,4 + e−0,8 a2 = −e−1,2

• Estructuras de orden uno conectadas en paralelo:

1 z e

-1

-0,8

2 x[n]

z

y[n]

-1

-0,4

-e

• Estructuras de orden uno conectadas en cascada:

1 x[n]

3 z

e- 0,8

-1

- 0,4

z - 0,8

e - 2e

-1

y[n]

-0,4

-e

Problema 4.1.4 Considere un filtro paso bajo de Chebyshev cuya función de sistema es: 4 0,001836 1 + z −1 Hlp (z) = (1 − 1,5548z −1 + 0,6493z −2) · (1 − 1,4996z −1 + 0,8482z −2)

4.1. Diseño de Filtros Digitales IIR

95

Este sistema de cuarto orden fue diseñado para que cumpliese las siguientes especificaciones: 0,89125 ≤ |Hlp (Ω)| ≤ 1, 0 ≤ Ω ≤ 0,2π |Hlp (Ω)| ≤ 0,17783, 0,3π ≤ Ω ≤ π Diseñar a partir de éste un filtro paso alto cuya frecuencia de corte de la banda de paso sea Ωp = 0,6π. Solución HHP

 0,02426 · 1 − 4z −1 + 6z −2 − 4z −3 + z −4 , |z| > 0,8750 (z) = 1 + 1,6076z −1 + 1,7571z −2 + 1,0250z −3 + 0,30773z −4

Problema 4.1.5 La siguiente función de sistema se corresponde con un filtro de Chebyshev paso alto, de orden 2, con 0,8 dBs de rizado en la banda de paso, y de pulsación de corte ωc = 2 krad/s: Hc (s) =

s2 s2 + 1967s + 3,28 · 106

a) Obtenga, a partir del filtro anterior, los coeficientes de la función de sistema Hd (z) de un filtro paso alto digital empleando la transformación que estime oportuno. Hd (z) =

b0 + b1 z −1 + b2 z −2 + ... 1 − a1 z −1 − a2 z −2 − . . .

Considere la pulsación de muestreo ωd = 8 krad/s, y opere con tres cifras decimales. b) Dibuje, empleando diagramas de flujo, una realización del filtro según la forma directa II traspuesta. c) Calcule el valor de ΩT en que se transforma la pulsación de corte del filtro analógico ωc , así como el valor del módulo de la respuesta en frecuencia (en dB’s) para dicha pulsación: 20 · log |Hd (ΩT )| Solución

a) Hd (z) =

0,439 − 0,878z −1 + 0,439z −2 , |z| > 0.567 1 − 0,434z −1 + 0,322z −2

b) La estructura es:

0,439 x[n]

y[n]

-1

z

0,434

-0,878 -1

0,439

z

- 0,322

Filtros Digitales.

96 c) ΩT = 1,3315 20 · log (|Hd (ΩT )|) ≈ 0 dB

Problema 4.1.6 La siguiente función de sistema se corresponde con un filtro paso bajo de Butterworth: Ha (s) =

Ho · ωc2 100 √ √ = 2 2 2 s + 2 · ωc s + ωc s + 10 · 2s + 100

a) Compruebe que ωc = 10 rad/s se corresponde con la pulsación de corte a 3 dBs. b) Empleando el método de la invarianza de impulso, obtenga, a partir del filtro anterior, los coeficientes de la función de sistema Hd (z) de un filtro paso bajo digital. b0 + b1 z −1 + b2 z −2 + ... 1 − a1 z −1 − a2 z −2 − ... √ Considere la pulsación de muestreo ωd = 25 · 2 rad/s. Hd (z) =

c) Dibuje, empleando diagramas de flujo, las realizaciones del filtro según las formas directas II y II traspuesta. Solución a)

1 |Ha (s = 10j)| = √ ⇒ 20 · log (|Ha (s = 10j)|) = −3 dB 2

b) Hd (z) =

2π 0,6803 · z −1 , |z| > e− 5 1 − 0,1759 · z −1 + 0,081z −2

c) Forma directa II:

x[n ] -1

z 0,1759

3,8280

y[n]

-1

z

-0,081 Forma directa II traspuesta:

y[n]

-1

z x[n]

3,8280 z-1

0,1759 -0,081

4.1. Diseño de Filtros Digitales IIR

97

Problema 4.1.7 Se desea diseñar un sistema para procesar una señal analógica xa (t) (limitada en banda a 3 kHz) con un filtro digital como se indica en la figura 4.1.

H( )

xa(t)

C/D

x[n]

Filtro paso bajo discreto

y[n]

D/C

ya(t)

HD( ) Ts= 0,125 ms

Ts= 0,125 ms Figura 4.1

Los requisitos exigidos al módulo de la respuesta en frecuencia |H(ω) | son: Debe ser mayor que 0,89125 en la banda de frecuencias comprendidas entre 0 y 800 Hz. No debe superar 0,17783 a partir de los 1400 Hz. a) Obtenga la plantilla de especificaciones del módulo de la respuesta en frecuencia, expresada en dB’s, del filtro discreto de respuesta al impulso h[n] real que cumpla lo indicado. b) El diseño del filtro anterior se decide realizar a partir de un prototipo analógico y empleando la transformación bilineal. Obtenga la plantilla de especificaciones del módulo de la respuesta en frecuencia de dicho filtro analógico. Considere Td = 2. c) En la etapa de aproximación se elige un filtro elíptico de orden 2 cuya función de sistema es Ha (s). Calcule, operando con cuatro cifras decimales, los coeficientes de la función de sistema HD (z) correspondiente. Ha (s) =

0,4386 · s2 + 0,307 2,7698 · s2 + 0,8858 · s + 0,3444

HD (z) =

b0 + b1 · z −1 + b2 · z −2 + · · · 1 − a1 · z −1 − a2 · z −2 − · · ·

d) Dibuje, empleando diagramas de flujo, las realizaciones del filtro según la formas directas II y II traspuesta. Solución

a) La plantilla es:

Filtros Digitales.

98

dB

Ap

Aa p=0,2

a=0,35

donde: Ωp = 0,2π Ωa = 0,35π Ap = −1 dB Aa = −15 dB b) La plantilla es:

dB

Ap

Aa p=0,3249

a=0,6128

donde: ωp = 0,3249 ωa = 0,6128 Ap = −1 dB Aa = −15 dB c) HD (z) =

0,1864 − 0,0658 · z −1 + 0,1864 · z −2 1 − 1,2127 · z −1 + 0,5571 · z −2

d) Forma directa II

b0 x[n]

y[n]

-1

a1

z

b1

-1

a2 z

b2

4.1. Diseño de Filtros Digitales IIR

99

Forma directa II traspuesta

b0 x[n]

b1 z

y[n]

-1

a1

-1

b2 z

a2

Problema 4.1.8 Se desea diseñar un filtro digital para aproximar la siguiente función del sistema correspondiente a un sistema LTI causal: Ha (s) =

1 s2 + 4s + 8

Considerando una pulsación de muestreo ωs = 8π , se pide obtener la función de sistema Hd (z) del filtro digital y su correspondiente diagrama polo-cero, empleando: a) el método de invarianza de impulso. b) el método de la transformación bilineal. Nota: opere con cuatro cifras decimales. Solución a) El diagrama de polos y ceros:

1 0, 8

Parte Imaginaria

0, 6 0, 4

Z P1

0, 2 ZC 1

0 -0,2

ZP 2

-0,4 -0,6 -0,8 -1 -1

• Ceros: zc1 = 0, zc2 → ∞

-0,5

0 Parte Real

• Polos: zp1 = 0,6065 · ej0,5 zp2 = 0,6065 · e−j0,5

0,5

1

Filtros Digitales.

100

Hd (z) =

0,0363 · z −1 , |z| > 0,6065 1 − 1,0646 · z −1 + 0,3679 · z −2

b) El diagrama de polos y ceros:

1 0,8 Parte Imaginaria

0,6 0,4

Z P1

0,2 2

0

Z

-0,2

C1

= Z C2 Z P2

-0,4 -0,6 -0,8 -1 -1

-0 ,5

0

0, 5

1

Parte Real • Ceros: zc1 = −1, zc2 = −1

• Polos: zp1 = 0,6202 · ej0,5191 zp2 = 0,6202 · e−j0,5191 Hd (z) =

1 + 2z −1 + z −2 1 , |z| > 0,6202 · 104 1 − 1,0769 · z −1 + 0,3846 · z −2

Problema 4.1.9 La función del sistema H(z) de la ecuación se corresponde con un filtro digital diseñado a partir de un prototipo analógico y empleando la transformación bilineal. Calcule la función del sistema de dicho filtro analógico y dibuje en ambos sistemas el diagrama polo-cero.  z −1 1 − z −1 H (z) = 1 + z −1 + z −2 Nota: Considere Td = 2.

4.1. Diseño de Filtros Digitales IIR

Solución Filtro digital:

101

 z −1 1 − z −1 H (z) = 1 + z −1 + z −2

|z| > 1

1

zp1 Parte Imaginaria

0,5

zc1

0

−0,5

zp2 −1 −1

−0,5

0

0,5

1

Parte Real Polos: zp1 = ej Ceros: zc1 = 1;

2π 3 ;

zp2 = e−j

2π 3

zc2 → ∞

Filtro analógico: H (s) = −2

s · (s − 1) s2 + 3

Re {s} > 0

s

Parte Imaginaria

1,5

p1

1 0,5

s

0

c1

s

c2

−0,5 −1 −1,5

s −2

Polos: sp1 =

√ 3j;

Ceros: sc1 = 0;

−1

√ sp2 = − 3j

sc2 = 1

p2

0 Parte Real

1

2

Filtros Digitales.

102 Problema 4.1.10 Dada la respuesta al impulso de un filtro analógico: hc (t) =

N X

k=1

Ak · espk ·t · u (t)

donde Ak y spk son constantes, y N es el orden del filtro, el método de invarianza al escalón transforma su función del sistema analógica Hc (s) en la función del sistema digital siguiente:  N X Ak · espk ·Td − 1 z −1  · Hie (z) = spk 1 − espk ·Td z −1 k=1

Del mismo modo, el método de invarianza a la rampa transforma la función del sistema analógica Hc (s) en la siguiente función del sistema N X Ak Hir (z) = spk k=1

!   1 − z −1 espk ·Td − 1  −1 · (spk · Td ) 1 − espk ·Td z −1

Dada la siguiente función del sistema estable y causal: Hc (s) =

s2

1 . + 3s + 2

Se pide: a) Obtener la función del sistema digital Hii (z) resultante de aplicar al filtro analógico dado, el método de invarianza al impulso (considere que hii [n] = Td · hc (n · Td ) ). b) Obtener la función del sistema digital Hie (z) resultante de aplicar al filtro analógico dado, el método de invarianza al escalón. c) Obtener la función del sistema digital Hir (z) resultante de aplicar al filtro analógico dado, el método de invarianza a la rampa. d) Dibuje el diagrama de polos y ceros del filtro analógico dado. Al aplicar al filtro analógico inicial los métodos de invarianza al impulso, al escalón y a la rampa, todos los sistemas resultantes son causales. ¿Son estables dichos sistemas? Razone la respuesta. Nota: considere fd = T1d = 10 y opere con cuatro cifras decimales. Exprese las funciones del sistema de la siguiente forma:

H (z) =

M P

bk k=0 N P

1−

k=1

· z −k

ak · z −k

Solución

a) Invarianza al impulso. Hii (z) =

0,0086z −1 , |z| > e−0,1 1 − 1,7235z −1 + 0,7408z −2

4.1. Diseño de Filtros Digitales IIR

103

b) Invarianza al escalón: Hie (z) =

0,0046z −1 + 0,0040z −2 , |z| > e−0,1 1 − 1,7235z −1 + 0,7408z −2

c) Invarianza a la rampa: Hir (z) =

0,0016 + 0,0057z −1 + 0,0014z −2 , |z| > e−0,1 1 − 1,7235z −1 + 0,7408z −2

d) El diagrama de polos y ceros del filtro analógico es:

Plano S

Parte Imaginaria

1

0,5

sp1

sp2

2

0

−0,5

−1 −2

−1

0 Parte Real

Polos: sp1 = −1; sp2 = −2 Ceros: sc1 = 0; sc2 = 0 ROC : Re {s} > −1 Todos los sistemas son estables.

1

Filtros Digitales.

104

Problema 4.1.11 En el diagrama de bloques de la figura 4.2 se conoce la función de transferencia del filtro discreto: √ 1 − 2z −1 + z −2 , a∈R H (z) = (1 − az −1 ) · (1 − 0,5z −1 )

x(t)

C/D

FILTRO DISCRETO H(z)

D/C

y(t)

TS

TS Ηeff(ω) Figura 4.2

Determinar el valor de a y la frecuencia de muestreo a utilizar si se desea que la respuesta del sistema ante la entrada:   √ π π + 5 2 · sen 4π103 t + x (t) = 5 · cos 2π103 t + 3 6 sea:

  2π y (t) = 8 · sen 4π103 t + 3

Solución ωs = 16π krad/s a=−

1 2

Problema 4.1.12 Un filtro continuo paso bajo cuya pulsación de corte es ωc = 10π rad/s se va a utilizar como base para el diseño de un filtro discreto ( H(Ω) ). A su vez, dicho filtro discreto se usará formando parte del sistema mostrado en la figura 4.3:

x(t)

C/D

FILTRO DISCRETO H( )

Ts=10-4 s

D/C

y(t)

Ts=10-4 s Heff( ) Figura 4.3

a) Si el filtro discreto se obtiene por aplicación del método de la respuesta al impulso invariante al filtro continuo inicial con Td = 0,01, determinar la pulsación de corte de Hef f (ω).

4.1. Diseño de Filtros Digitales IIR

105

b) Si el filtro discreto se obtiene por aplicación de la transformación bilineal al filtro continuo inicial con Td = 2, determinar la pulsación de corte de Hef f (ω). Solución

a) ωcef f = π krad/s b) ωcef f = 30,78 krad/s Problema 4.1.13 Se desea diseñar un filtro discreto paso banda que cumpla las siguientes especificaciones de respuesta de amplitud: 1 |HD (Ω)| = √ 2 para las pulsaciones Ω1 = 0, 2π Ω2 = 0, 8π El diseño del mismo, HD (z), se realizará mediante la conexión adecuada de dos filtros, H1 (z) y H2 (z), obtenidos por aplicación de la transformación bilineal a los filtros de Butterworth cuyas funciones del sistema son, respectivamente: H1 (s) =

ωa , s + ωa

H2 (s) =

s s + ωb

a) Obtener la función del sistema del filtro discreto, HD (z). b) Una vez diseñado el filtro se desea que |HD (Ω)| = 1 para la pulsación Ω = π/2. Obtener la nueva expresión de la función del sistema, HD2 (z), así como los nuevos valores de respuesta en amplitud (expresados en decibelios) para las pulsaciones Ω1 y Ω2 . c) El filtro discreto diseñado en el apartado b), HD2 (z), se utiliza para el diseño del sistema de la figura 4.4, en el que Ts = 0,125 ms. Obtener la respuesta del sistema, y(t), si la señal de entrada es:  π + 3 · cos (ω2 t + 0,72π) x (t) = 2 + 5 · sen ω1 t + 2 siendo ω1 = 4π krad/s y ω2 = 6,4π krad/s

H( )

x(t)

C/D

x[n]

Filtro paso banda discreto

y[n]

D/C

HD2( ) Ts

Ts Figura 4.4

y(t)

Filtros Digitales.

106 Solución a)

 0, 570 · 1 − z −2 , |z| > 0,510 HD (z) = H1 (z) · H2 (z) = 1 − 0, 260z −2

b)

 0, 630 · 1 − z −2 , |z| > 0,510 HD2 (z) = 1 − 0, 260z −2

En los dos casos el valor de la respuesta en amplitud es −2,143dBs   c) y (t) = 5sen ω1 t + π2 + 2, 331 cos ω2 t + π2

Problema 4.1.14 La función del sistema H(s) de un filtro de tiempo continuo cumple la plantilla de especificaciones que se muestra en la figura 4.5: H (s) =

2 s2 + 2s + 2

α (dBs) 25,12

3 0

ωp

ωa

ω (rad/s)

Figura 4.5

Dicho filtro va a ser utilizado como base para el diseño de un filtro de tiempo discreto, HD (Ω), que a su vez forma parte del sistema mostrado en la figura 4.6:

x(t)

A/D

FILTRO DISCRETO HD( )

Ts

D/A

y(t)

Ts Ha( ) Figura 4.6

a) Determinar la función del sistema del filtro de tiempo discreto, HD (z), obtenida por aplicación del método de la respuesta al impulso invariante al filtro de tiempo continuo, H(s), con Td = (1/4)s.

4.1. Diseño de Filtros Digitales IIR

107

b) Dibujar la estructura en forma directa II traspuesta del filtro digital HD (z). c) Obtener el valor de la pulsación de muestreo, ωs , que debería utilizarse si se desea que la pulsación de corte de la banda de paso del filtro analógico resultante, Ha (ω), sea de 1 krad/s. d) Obtener la plantilla de especificaciones del módulo de la respuesta en frecuencia expresada en dBs del filtro analógico resultante, Ha (ω), si para el diseño del filtro discreto se aplica el método de la transformación bilineal con Td = 0,4s. Considerar para este apartado Ts = 1ms. Solución

a) HD (z) =

−1 0,0963z −1 , |z| > e 4 1 − 1,5092z −1 + 0,6065z −2

b) Forma directa II:

0,0963 x[n]

c) ωs =

8π √ 2

z

y[n]

-1

z

1,5092 -1

-0,6065

krad/s

d) La plantilla es:

|H( ω)|a dBs

0 -3

-25,12 551,3

1752,1

ω

Problema 4.1.15 Se ha obtenido la función de transferencia H(z) de un filtro IIR por aplicación de la transformación bilineal, con Td = 2, a un filtro continuo cuya función de transferencia es:  0, 13 s2 + 1 . H (s) = 3 s + 0, 9 s2 + 1, 24 s + 0, 52

Filtros Digitales.

108

El filtro así obtenido (H(z)) forma parte del sistema de la figura 4.7. Determinar el valor de  TS para que cuando la señal de entrada sea x (t) = A0 cos 104 t + 0, 2 π , la señal de salida sea nula (y (t) = 0).

x(t)

FILTRO DISCRETO H(z)

A/D

Ts

D/A

y(t)

Ts Heff(ω) Figura 4.7

Solución Ts = 50πµs. Problema 4.1.16 La función de transferencia de un filtro de Butterworth es: H (s) =

4 √ s2 + 2 2 s + 4

A partir de él se desea obtener la función de transferencia de un filtro discreto paso bajo cuya pulsación de corte a 3 dBs sea Ωc = 0,5π rad. (a) Obtener la función del sistema del filtro discreto, H(z), utilizando la transformación bilineal. (b) Dibujar el diagrama de flujo de la estructura en Forma Directa II transpuesta correspondiente al filtro cuya función del sistema es H(z). (c) El filtro discreto H(z) se va a utilizar formando parte del diagrama de bloques de la siguiente figura, determinar el valor de Ts para que la frecuencia de corte correspondiente a Hef f (ω) sea 10 KHz.

x(t)

FILTRO DISCRETO H( )

C/D

Ts

D/C

Ts Heff( )

Solución (a) La función del sistema es: H (z) =

0, 29 + 0, 59z −1 + 0, 29z −2 1 + 0, 17z −2

|z| > 0, 41

y(t)

4.1. Diseño de Filtros Digitales IIR

109

(b) El diagrama de flujo es:

x[n]

y[n]

0,29 z -1 0,59 z -1 0,29 - 0,17

(c) Ts =

1 ms 40

Problema 4.1.17 Un filtro analógico de tercer orden, real, con función de sistema HC (s), tiene ceros en s = −1 y s = −j y polos en s = −2 y s = −1 + j, siendo su ganancia en continua de valor unidad. Al aplicar la transformación bilineal a HC (s) se observa que el filtro de tiempo discreto, HD (z), posee un cero en el origen. a) Determinar el parámetro Td utilizado en la transformación. b) Determinar si existe alguna frecuencia discreta para la cual la respuesta en frecuencia del filtro digital es nula. En caso afirmativo calcular su valor. c) Representar el diagrama polo-cero de ambos filtros. d) El filtro discreto diseñado se utiliza para la implementación de un filtro continuo tal como se muestra en la figura. Calcula el periodo de muestreo utilizado, Ts , para que al aplicarle  3 a la entrada la señal xc (t) = 3 + 2sen 5π 6 10 t + 0,234 , la señal de salida sea yc (t) = K. Determinar el valor de K.

C/D

D/C

Figura 4.8

Solución a) Td = 2 b) zc1,2 = e±jπ/2

Filtros Digitales.

110 c) zp1 = − 31 ;

zp2,3 =

−1±2j ; 5

zc1,2 = ±j;

zc3 = 0

Parte Imaginaria

1 0,5 0 −0,5 −1 −2

−1

0

1

Parte Real

Parte Imaginaria

1

0,5

0

−0,5

−1 −1

−0,5

0

0,5

1

Parte Real

d) Ts = 53 ms;

yc (t) = 3

Problema 4.1.18 Se desea diseñar un filtro discreto a partir de un filtro continuo cuya función del sistema viene dada por la siguiente expresión: H(s) =

s2 − s + s2 + s +

1 2 1 2

a) Indica de forma razonada de que tipo de filtro se trata.

4.1. Diseño de Filtros Digitales IIR

111

b) Indica de forma razonada si el diseño del filtro discreto utilizando los métodos de respuesta al impulso invariante y transformación bilineal mantienen las características del filtro analógico.

c) Diseña el filtro discreto utilizando él o los métodos para los que la respuesta del apartado anterior ha sido afirmativa. Utiliza como periodo de diseño Td = 4. d) Representa el diagrama de polos y ceros y el módulo de la respuesta en frecuencia tanto del filtro analógico como del o de los filtros discretos diseñados. Solución

El filtro continuo es un filtro paso todo.

Únicamente puede utilizarse el método de la transformación bilineal.

Aplicando la transformación bilineal: 1 2 −1 + z + z −2 H(z) = 5 5 2 1 1 + z −1 + z −2 5 5 Los polos y ceros del filtro discreto: √ zc1,2 = −1 ± 2j = √5 · e±j2,0344 5 ±j2,0344 −1 ± 2j = ·e zp1,2 = 5 5 El diagrama de polos y ceros es: Plano S

Plano z

1

2

0,5

Parte Imaginaria

Parte Imaginaria

1,5

0

−0,5

1 0,5 0 −0,5 −1 −1,5

−1

−2

−1

−0,5

0

0,5

1

Parte Real

El módulo de la respuesta en frecuencia es:

−3

−2

−1

0 Parte Real

1

2

3

Filtros Digitales.

112 3

2

2

|H(ω)|

|H( )|

3

1

0 −4

1

−3

−2

−1

0

2

1

3

4

ω (rad/s)

0 −✹

−3

−2

−1

0

1

(rad/muestra)

2

3

4

Problema 4.1.19 La figura 4.9 muestra el módulo de la respuesta en frecuencia de un filtro real de tiempo continuo. A partir del mismo se ha diseñado un filtro discreto por aplicación del método de la transformación bilineal obteniéndose la función de sistema dada por la expresión: Hd (z) = k ·

1 + az −1 + z −2 1 − 0,7z −1 + 0,1z −2

Módulo de la respuesta en frecuencia del filtro analógico

|H(ω)|

8

P

P 0

4

8

12

16 20 ω (rad/s)

24

Figura 4.9

Sabiendo que el filtro discreto responde con salida nula cuando la entrada que se le aplica es la secuencia x[n] = ej1,8546n , determinar: a) Valores de k y a. b) Valor de P y expresión de la función de sistema del filtro analógico, H(s). c) Si el filtro discreto se utiliza formando parte del esquema de la figura 4.10, determinar el valor de Ts necesario para que la salida del sistema equivalente, yc (t), sea nula cuando la entrada es xc (t) = 5 · sen(104 t + 1,8546). d) Dibujar el diagrama de flujo correspondiente a la implementación del filtro discreto utilizando las estructuras en celosía y en forma directa 2 transpuesta.

4.1. Diseño de Filtros Digitales IIR

113

Figura 4.10

Ecuaciones de la estructura en celosía: Pm−1 (z) =

Pm (z)−km ·z −m ·Pm (z −1 ) 2 1−km

Cm (z) = z −m · Pm (z −1 ) Bm−1 (z) = Bm (z) − vm · Cm (z) Solución a) a = 0,56 k = 45 b) P = 1, H(s) =

s2 +16 s2 −3s+2

c) Ts = 185,46µs d) Estructura en forma directa II transpuesta



b0 = 1 b1 = 0,56 b2 = 1 a0 = 1 a1 = 0,7 a2 = 0,1

 k1 = −0,6364 k2 = 0,1 Estructura en celosía: V0 = 1,7018 V1 = 1,26 V2 = 1

Filtros Digitales.

114

Problema 4.1.20 Se desea diseñar un filtro digital IIR utilizando la función de sistema de un filtro analógico dada por la siguiente expresión: H(s) = 2 ·

s2 + s s2 + 2s + 2

a) Indicar, de forma razonada, qué método utilizarías para el diseño: la respuesta al impulso invariante o la transformación bilineal. b) Una vez aplicado el método de diseño, se obtiene la siguiente expresión para la función de sistema del filtro digital: Hd (z) = k ·

1 − z −1 . 1 + a1 z −1 + a2 z −2

Obtener los valores de las constantes k, a1 , y a2 . c) El filtro anterior se utiliza formando parte del sistema de la figura 4.11, siendo Ts = 0,1 ms. Suponiendo que la función de sistema del filtro discreto es: Hd (z) = 4 ·

1 − z −1 , 1 + z −1 + 2z −2

obtener la señal de salida, yc (t), del sistema de la figura 4.11, cuando la señal de entrada es:  π 3 3 . xc (t) = + cos 5π · 103 t − 7 4 2

Figura 4.11

d) Dibujar el diagrama de flujo correspondiente a la implementación del filtro discreto del apartado (c) utilizando las estructuras en celosía. Ecuaciones de la estructura en celosía: Cm (z) = z −m · Pm (z −1 ) Pm−1 (z) =

Pm (z) − km · Cm (z) 2 1 − km

Bm−1 (z) = Bm (z) − vm · Cm (z) Solución a) Transformación bilineal.

4.1. Diseño de Filtros Digitales IIR

115

b) Hd (z) =

1 − z −1 4 · 5 1 + 52 z −1 + 15 z −2

c) yc (t) = 3 · cos 5π · 103 t + d) k = [ 13 v = [ 16 3

π 2



2] −4 0]

Figura 4.12

Problema 4.1.21 Se desea diseñar un sistema de respuesta en frecuencia He (ω) que elimine la componente frecuencial f1 = 4 kHz, utilizando el esquema de la figura 4.13.

Figura 4.13

Para la obtención de la función de sistema, Hd (z), del filtro discreto se partirá de un filtro analógico prototipo cuya función de sistema es: H(s) =

5 s2 + 1 · 2 2 (s + 2s + 5) (s + 2)

Si se utiliza el método de la transformación bilineal se observa que Hd (z) posee un polo en z = −1/3. Bajo estas condiciones se pide: a) Valor del periodo de muestreo, Td , que debe utilizarse al aplicar la transformación. b) Suponiendo que Td = 2, situación de los polos de Hd (z). c) Periodo de muestreo, Ts , de los convertidores C/D y D/C, necesario para que se cumplan las condiciones del enunciado.

Filtros Digitales.

116

d) ¿Qué componente frecuencial de la señal de entrada se eliminaría si se redujera a la mitad el valor de Td ? Solución

a) Td = 2 b) zp1,2 = c) Ts =

1 16



2 2

· e±j

3π 4

, zp3 = −1/3

ms

d) f = 2,36 kHz

4.2.

Diseño de Filtros Digitales FIR

Problema 4.2.1 Se pretende diseñar un filtro FIR de fase lineal tipo II (número de coeficientes par y simetría par en la respuesta al impulso). Se desea que la respuesta al impulso sea real, y que la función de sistema presente un cero en el punto del plano z: z1 =

1 1 + j 2 2

Se pide: a) Obtener la función de sistema que cumple los requisitos anteriores, sabiendo que el orden del filtro es el mínimo posible. b) Dibujar la respuesta al impulso correspondiente. c) Representar el filtro empleando diagramas de flujo mediante las conexiones: c.1) Cascada (con subestructuras de orden dos). c.2) Fase lineal. d) Contabilizar en ambas estructuras el número de registros de almacenamiento, de multiplicaciones y sumas necesarios por muestra de salida (no considerar multiplicaciones por la unidad). Solución

a) 3 3 H (z) = 1 − 2z −1 + z −2 + z −3 − 2z −4 + z −5 2 2 b) Respuesta al impulso:

4.2. Diseño de Filtros Digitales FIR

117

Respuesta al impulso h[n] 2 1,5 1

h[n]

0,5 0 −0,5 −1 −1,5 −2 −1

0

1

2

3

4

5

6

n c.1) Cascada:

x[n]

z

-1

z

z

-1 -1

-1

z

1/2

z

-2

y[n]

-1

-1

2

c.2) Fase Lineal:

z

x[n]

-1

z

-1

z

z

-1

z -2

-1

3/2

y[n] d) Cascada: • Registros de almacenamiento: Coeficientes: 4, Datos: 5 • Sumas: 5 • Multiplicaciones: 4 Fase Lineal

-1

Filtros Digitales.

118 • Registros de almacenamiento: Coeficientes: 2, Datos: 5 • Sumas: 5 • Multiplicaciones: 2

Problema 4.2.2 Para diseñar un filtro digital se emplea el método de la ventana. Observando dos ventanas diferentes (rectangular y Hamming) se obtienen los siguientes valores: Ventana a.1: Amplitud máxima del lóbulo secundario: -46 dB. Anchura del lóbulo principal: 0’098. Ventana a.2: Amplitud máxima del lóbulo secundario: -13’5 dB. Anchura del lóbulo principal: 0’056. Una vez diseñado el filtro se comprueba que la banda de transición resultante es de mayor anchura si se emplea la ventana a.1. Indique razonadamente si los datos indicados anteriormente se pueden corresponder con un diseño real y qué ventanas son las mencionadas como a.1 y a.2. Solución a.1 Hamming a.1 Rectangular Puede ser un diseño real. Problema 4.2.3 Se pretende diseñar un filtro FIR de fase lineal tipo III (número de coeficientes impar y simetría impar en la respuesta al impulso). Se desea que la respuesta al impulso sea real, y que la función de sistema presente un cero en el punto del plano z1 = 21 . Se pide: a) Obtener la función de sistema que cumple los requisitos anteriores, sabiendo que el orden del filtro es el mínimo posible. b) Dibujar la respuesta al impulso correspondiente. c) Representar el filtro empleando diagramas de flujo mediante las conexiones: c.1) Forma directa. c.1) Fase lineal. d) Contabilizar en ambas estructuras el número de registros de almacenamiento (para coeficientes y datos), de multiplicaciones y sumas necesarios por muestra de salida (no considerar multiplicaciones por la unidad).

4.2. Diseño de Filtros Digitales FIR

119

Solución a) H (z) = 1 − 52 z −1 + 52 z −3 − z −4 b) La respuesta al impulso es: Respuesta al impulso h[n] 3 2

h[n]

1 0 −1 −2 −3

0

1

2

3

4

n

c) c.1) Forma directa.

z

-1

z

-1

z

-1

-1

z

x[n] -1

5/2

-5/2

y[n] c.2) Fase lineal.

z

x[n]

-1

z

-1

-1 -1 z

-1

-5/2 y[n]

z

-1

Filtros Digitales.

120 d) Forma directa: • Registros de almacenamiento: coeficientes: 3, datos: 4 • Sumas: 3 • Multiplicaciones: 3 Fase Lineal: • Registros de almacenamiento: coeficientes: 3, datos: 4 • Sumas: 3 • Multiplicaciones: 3

Problema 4.2.4 Se pretende diseñar un filtro FIR de manera que el módulo de la respuesta en frecuencia |H(Ω) | del filtro diseñado satisfaga los siguientes requisitos: Debe ser mayor que 0,9 en la banda de frecuencias comprendidas entre 0 y 0,15π. No debe superar 0,01 a partir de los 0,25π. a) Represente la plantilla de especificaciones en escala semilogarítmica (eje de ordenadas expresado en dBs) del módulo de la respuesta en frecuencia del filtro deseado. b) Para diseñar el filtro se decide emplear el método de la ventana con las siguientes funciones: rectangular, Hanning, Hamming y Blackman. Indique qué ventanas de las anteriores satisfacen los requisitos exigidos. c) De las ventanas que cumplen las especificaciones iniciales, seleccione la que presente menor atenuación en la banda eliminada. Empleando dicha ventana, obtenga la anchura de la banda de transición y el orden del filtro resultante. d) Represente la plantilla de especificaciones, en escala lineal, que cumplirá el módulo de la respuesta en frecuencia del filtro diseñado. e) Indique, especificando los valores oportunos, la expresión que permitiría calcular los coeficientes de la respuesta al impulso del filtro diseñado. Solución a) La plantilla es:

|H(Ω)| (dB)

0 p

a

ΩP Ωa

π Ω

4.2. Diseño de Filtros Digitales FIR

121

donde: AP = −0,9151 dB Aa = −40 dB Ωp = 0,15π Ωa = 0,25π b) Todas excepto la rectangular c) La ventana de Hanning. ∆Ω = 0,25π − 0,15π = 0,1π M = 63 d) La plantilla es:

|H(Ω)|

1

1 1

ΩP Ω Ωa C donde: ΩC = 0,2π δ ≈ 0,0063 ∆Ω = 0,0987π Ωp = 0,1506π Ωa = 0,2494π e) h [n] =

(

sen(0,2·π·(n−63/2)) π·(n−63/2)

0,

·

1 2

· 1 − cos

2·π·n 63



, 0 ≤ n ≤ 63 resto

Filtros Digitales.

122

Problema 4.2.5 Se desea diseñar un sistema para procesar una señal analógica x(t) (limitada en banda a 8 kHz) con un filtro digital como se indica en la figura 4.14.

x(t)

A/D

Filtro paso bajo discreto HD( )

Ts=0,05 ms

D/A

y(t)

Ts=0,05 ms

H( ) Figura 4.14

Los requisitos exigidos al módulo de la respuesta en frecuencia |H(ω) | son: Debe ser mayor que 0,85 y menor que 1,1 en la banda de frecuencias comprendidas entre 0 y 2000 Hz. No debe superar 0,001 a partir de los 3400 Hz. a) Represente la plantilla de especificaciones en escala semilogarítmica (eje de ordenadas expresado en dBs) del módulo de la respuesta en frecuencia que debe cumplir el filtro digital HD (Ω) . b) Para diseñar el filtro se decide emplear el método de la ventana. A partir de la tabla adjunta, indique qué ventanas –considere sólo las ventanas rectangular, Hanning, Hamming y de Blackman– satisfacen los requisitos anteriores. c) Seleccione la ventana de las indicadas en el apartado anterior que presente menor atenuación en la banda eliminada. Empleando dicha ventana, obtenga la anchura de la banda de transición y el orden del filtro resultante. d) Represente la plantilla de especificaciones, en escala lineal, que cumplirá el módulo de la respuesta en frecuencia del filtro diseñado. e) Indique, especificando los valores oportunos, la expresión que permitiría calcular los coeficientes de la respuesta al impulso del filtro diseñado.

4.2. Diseño de Filtros Digitales FIR

123

Solución a) La plantilla es:

|HD( )| dBs

0,83 0 -1,41

-60

0,2

0,34

b) La ventana de Blackman. c) ∆Ω = 0,14π,

M = 80

d) La plantilla es:

HD

1,00017

1 0,99983

0,00017

0,2005

0,2005 0,27 e) h [n] =

(

sen(0,27π(n−40)) π(n−40)

0,

0,42 − 0,5 cos

2πn 80



+ 0,08 cos

4πn 80



, 0 ≤ n ≤ 80 resto

Problema 4.2.6 Dado un filtro FIR de coeficientes reales y de longitud N, que cumple que h[n] 6= 0 para 0 ≤ n ≤ M y h[n] = 0 fuera de ese intervalo. El sistema es de fase lineal si su respuesta al impulso satisface que h[n] = ±h[M − n] para 0 ≤ n ≤ (M ). Se pide: a) Deducir de la condición anterior la siguiente igualdad:

H(z) = ±z −M · H(z −1 )

Filtros Digitales.

124

b) Indicar razonadamente cómo están distribuidos los polos y los ceros si el filtro cumple que h[n] = −h[M − n], y M es un número impar. c) Indicar razonadamente si el sistema del apartado anterior puede ser un filtro paso-todo. d) Indicar razonadamente si el filtro del apartado b) es, de todos los que tienen el mismo módulo de la respuesta en frecuencia, el sistema de fase mínima. Solución

b) Es un sistema FIR tipo IV, por lo tanto tiene un cero en z = 1. El resto se distribuye en grupos de 4, por cada cero estará su recíproco y los conjugados de ambos, (z0 , z0−1 , z0∗ , (z0−1 )∗ ). Si existen ceros en el eje real y son diferentes de +1 y −1, se distribuyen por parejas con sus recíprocos. Si los ceros están en la circunferencia de radio unidad y son diferentes de +1 y −1, se distribuyen por parejas con sus conjugados. Si los ceros están en +1 ó −1 pueden aparecer aislados. En cuanto a los polos, el sistema tiene M polos en el origen. c) No es paso todo. d) No es de fase mínima a no ser que todos los ceros estén en la circunferencia unidad. Problema 4.2.7 Se desea diseñar un filtro FIR de longitud impar (M + 1) y fase cero con las siguientes especificaciones: |H(Ω) | debe ser mayor que 0,88 y menor que 1,09 en la banda de frecuencias [0, 0,2π] radianes. |H(Ω) | no debe superar 0,01 a partir de 0,3π radianes. a) Para diseñar el filtro se decide emplear el método de la ventana. Indique qué ventanas (entre rectangular, Hanning, Hamming, Blackman y Kaiser) permiten diseñar el filtro con los requisitos anteriores. b) Seleccione la ventana de las indicadas en el apartado anterior que presente menor orden –indicando su valor–. Empleando dicha ventana, obtenga la anchura de la banda de transición que tendrá el filtro diseñado. c) Represente la plantilla de especificaciones, en escala lineal, que cumplirá el módulo de la respuesta en frecuencia del filtro diseñado. d) Indique, especificando los valores oportunos, la expresión que permitiría calcular los coeficientes de la respuesta al impulso del filtro diseñado. Solución

a) Hanning, Hamming, Blackman y Kaiser. b) La ventana de Kaiser, con M = 45 y ∆Ωreal = 0,0992π

4.2. Diseño de Filtros Digitales FIR

125

c) La plantilla es:

HD

1,01 1

0,99

0,01 0,2004

d) h [n] =

  

C

0,2996

sen(0,25·π·(n−45/2)) π·(n−45/2)

0,

·

  q 2 I0 β· 1−( n−α ) α I 0 (β)

, 0 ≤ n ≤ 45 resto

donde α = 45/2, β = 3,3953. Problema 4.2.8 Se pretende diseñar un filtro FIR empleando el método de la ventana, de manera que se satisfagan los siguientes requisitos: Pulsación de corte del filtro ideal: 0,5π radianes. El filtro deseado debe tener longitud 21, con h [n] 6= 0 sólo para −10 ≤ n ≤ 10. a) Represente la plantilla de especificaciones en escala lineal del módulo de la respuesta en frecuencia que satisfará el filtro digital cuando se emplean las siguientes ventanas: a.1) Rectangular. a.2) Hanning. a.3) Kaiser, con β = 10. b) Seleccione de los anteriores, el filtro que presente la banda de transición más estrecha, y el filtro que presente mayor atenuación en la banda eliminada. Indique para ambos cómo se obtendría la respuesta al impulso del filtro final diseñado. Solución

a) la plantilla de especificaciones es:

Filtros Digitales.

126

HD

1+

1 1-

p real

π

a real C

a.1) Rectangular: Ωpreal = 0,4540π Ωareal = 0,5460π δ = 0,0902 a.2) Hanning: Ωpreal = 0,3445π Ωareal = 0,6555π δ = 0,0064 a.3) Kaiser: Ωpreal = 0,1664π Ωareal = 0,8337π δ = 1,0661 · 10−5 b) En los dos casos el filtro se obtiene de la siguiente forma: h [n] =

sen (0,5 · π · (n − 10)) · w [n] π · (n − 10)

La ventana rectangular tiene la banda de transición más estrecha: w [n] =



1, 0 ≤ n ≤ 20 0, resto

La ventana de Kaiser tiene la mayor atenuación en la banda eliminada:

w [n] =

  

  q 2 I0 β· 1−( n−10 10 ) I 0 (β)

0,

,

0 ≤ n ≤ 20 resto

Problema 4.2.9 Se pretende diseñar un filtro FIR paso banda de fase cero empleando el método de la ventana, de manera que se satisfagan los siguientes requisitos:

4.2. Diseño de Filtros Digitales FIR

127

Pulsaciones de corte de la banda de paso: 0,5π y 0,7π radianes. Pulsaciones de corte de la banda eliminada: 0,3π y 0,9π radianes. Desviación en la banda de paso: 0,001. Desviación en la banda eliminada: 0,01. a) Indique qué ventanas (entre rectangular, Hanning, Hamming y Blackman) permiten diseñar el filtro con los requisitos anteriores. b) Seleccione la ventana de las indicadas en el apartado anterior que presente mayor atenuación en la banda eliminada –indicando su valor–. Empleando dicha ventana, obtenga la longitud necesaria para cumplir las especificaciones anteriores. c) Represente la plantilla de especificaciones, en escala lineal, que cumplirá el módulo de la respuesta en frecuencia del filtro diseñado. d) Indique, especificando los valores oportunos, la expresión que permitiría calcular los coeficientes de la respuesta al impulso de un filtro causal obtenido a partir del filtro diseñado. ¿Presenta el filtro causal fase lineal? Si la respuesta es afirmativa, ¿qué tipo de filtro es? Justifique las respuestas. Solución a) La ventana de Blackman b) Longitud: M + 1 = 57. c) La plantilla es:

HD

1+ 1

1-

ar

pr

pr

ar

δ = 1,7179 · 10−4

Ωar1 = 0,3007π

Ωpr1 = 0,4993π Ωpr2 = 0,7007π Ωar2 = 0,8993π

Filtros Digitales.

128 d) hpz [n] = hI [n] · w [n] = donde w [n] =





sen (0,8 · π · (n − 28)) sen (0,4 · π · (n − 28)) − π · (n − 28) π · (n − 28)

0,42 − 0,5 · cos 0,

2πn 56



+ 0,8 · cos

4πn 56





· w [n]

, 0 ≤ n ≤ 56 resto

Es un filtro de fase lineal tipo I. Problema 4.2.10 Se pretende diseñar un filtro FIR paso bajo empleando el método de la ventana, de manera que se satisfagan los siguientes requisitos: Pulsación de corte del filtro ideal: 0,5π radianes. El filtro deseado debe tener longitud 21, con h [n] 6= 0 sólo para 0 ≤ n ≤ 20. a) Represente la plantilla de especificaciones en escala lineal del módulo de la respuesta en frecuencia que satisfará el filtro digital cuando se emplea la ventana de Kaiser, con β = 9. b) Indique cómo se obtendría la respuesta al impulso del filtro final diseñado. c) Indique qué ventanas (entre Hanning, Hamming y Blackman) podrían emplearse para obtener un filtro con la misma anchura –como máximo– para la banda de transición que la del filtro diseñado en el apartado “a”. Indique la longitud necesaria para cada una de ellas. d) Indique qué ventanas (entre Hanning, Hamming y Blackman) podrían emplearse para obtener un filtro con al menos la misma atenuación en la banda eliminada que la del filtro diseñado en el apartado “a”. Indique la longitud necesaria para cada una de ellas. Solución

a) La plantilla es:

HD

1+ 1

1-

p real

a real C

4.2. Diseño de Filtros Digitales FIR

129

donde: Ωpreal = 0,2129π Ωareal = 0,7871π δ = 3,0305 · 10−5

b) h [n] = donde: w [n] =

  

sen (0,5 · π · (n − 10)) · w [n] π · (n − 10)   q 2 I0 9· 1−( n−10 10 ) I 0 (9)

0,

, 0 ≤ n ≤ 20 resto

c) Hanning, Longitud: M + 1 = 12. Hamming, Longitud: M + 1 = 13. Blackman, Longitud: M + 1 = 21. d) Ninguna de ellas. Problema 4.2.11 Se pretende diseñar un filtro FIR de manera que se satisfagan los siguientes requisitos: Frecuencia de corte de la banda de paso: 3kHz Frecuencia de corte de la banda eliminada: 5kHz Frecuencia de muestro: 20 kHz Desviación de la banda de paso: 0,15 Desviación de la banda eliminada: 0,015 a) Represente la plantilla de especificaciones en escala semilogarítmica (eje de ordenadas en dBs) del módulo de la respuesta en frecuencia del filtro digital deseado. b) Para diseñar el filtro se decide emplear el método de las ventanas con las siguientes funciones: rectangular, Hanning, Hamming, Blackman y Kaiser. Indique qué ventanas de las anteriores satisfacen los requisitos exigidos. c) De las ventanas que cumplen las especificaciones iniciales, obtenga el orden requerido para cada uno de los filtros resultantes. d) Seleccione de los anteriores, los dos filtros que representen menor orden. Represente las plantillas de especificaciones, en escala lineal, que cumplirán los módulos de la respuesta en frecuencia.

Filtros Digitales.

130 Solución a) Plantilla:

HD

( dB )

'p p

a

0,3

0,5

Ap = 1,2140 dB A′p = −1,4116 dB Aa = −36,4782 dB b) Las ventanas de Hanning, Hamming, Blackman y Kaiser. c) Hanning, orden: M = 32. Hamming, orden: M = 34. Blackman, orden: M = 56. Kaiser, orden: M = 20. d) La ventana de Hanning y Kaiser: • Hanning:

HD

0,3028 donde δ = 0,0064.

0,4972

4.2. Diseño de Filtros Digitales FIR

131

• Kaiser:

HD

0,3007

0,4994

donde δ = 0,015. Problema 4.2.12 En el diseño de un sistema de comunicaciones se necesita un filtro capaz de realizar un desplazamiento de fase de la señal de entrada para las frecuencias comprendidas entre 1 kHz y 3 kHz, con una atenuación superior a 52 dBs para frecuencias por debajo de 0,5 kHz y por encima de 3,5 kHz. Sabiendo que la respuesta en frecuencia de un desplazador de fase ideal (transformador de Hilbert) es:

donde Ω1 < Ω2

 Ω1 < Ω < Ω2  −j, Hd (Ω) = j, −Ω2 < Ω < −Ω1  0, |Ω| < Ω1 y Ω2 < |Ω| < π

a) La respuesta al impulso del transformador de Hilbert, indicando si tiene o no simetría y en caso de tenerla de qué tipo. Para implementar el transformador real se opta por realizar el diseño de un sistema FIR utilizando el método de las ventanas. b) Si la frecuencia de muestreo utilizada es de 8 kHz, indicar las posibles ventanas que pueden utilizarse así como el orden (M ) del sistema que se obtendría para cada uno de los casos. c) Utilizando la ventana de menor orden exceptuando la de Kaiser, obtener el valor de h[2], h[M − 2] y h[2M ]. d) Si se utiliza la ventana de Kaiser obtener el valor del factor de forma necesario e indicar el tipo de sistema de fase lineal que se obtiene. ¿Podría ser un sistema de fase lineal de cualquiera de los cuatro tipos? Razona la respuesta.

Filtros Digitales.

132 Solución a) h [n] =

(

cos(Ω1 n)−cos(Ω2 n) , πn

0,

n 6= 0 n=0

cumple la simetría: h[n] = −h[−n] b) Se podrán utilizar las ventanas de Hamming, Blackman y Kaiser: • Hamming: M = 54. • Blackman: M = 89. • Kaiser: M = 50. c) hD [n = 2] = 1, 30722 · 10−3 hD [n = M − 2] = −1, 30722 · 10−3 hD [n = 2M ] = 0 d) β = 4, 772 Se trata de un sistema FIR de fase lineal, causal de tipo III. Solo pueden utilizarse sistemas tipo III ó IV. Problema 4.2.13 En el diseño de un audífono digital se necesita un sistema capaz de realizar la derivada de la señal de entrada para las frecuencias comprendidas entre cero y 3.6kHz, con una atenuación superior a 40 dBs para las frecuencias superiores a 4 kHz. Sabiendo que la respuesta en frecuencia de un diferenciador digital ideal es: H (Ω) =



jΩ, |Ω| < Ωc 0, Ωc < |Ω| < π

siendo Ωc la frecuencia de corte del derivador ideal, obtener: a) La respuesta al impulso del derivador digital ideal, indicando si tiene o no simetría y en caso de tenerla de que tipo. Para implementar el derivador se opta por realizar el diseño de un sistema FIR de fase lineal utilizando el método de las ventanas. b) Sabiendo que la frecuencia de muestreo utilizada es de 16kHz, indicar las posibles ventanas que pueden utilizarse así como el orden del sistema que se obtendría para cada uno de los casos. c) Utilizando la ventana de menor orden exceptuando la de Kaiser, obtener el valor de h[2], h[M − 2] y h[2M ]. d) Si se utiliza la ventana de Kaiser obtener el valor del factor de forma necesario e indicar el tipo de sistema de fase lineal que se obtiene. ¿Podría ser un sistema de fase lineal de cualquiera de los cuatro tipos? Razona la respuesta.

4.2. Diseño de Filtros Digitales FIR

133

Solución a) h [n] =

(

Ωc ·cos(Ωc n) πn

0,



sen(Ωc n) , πn2

n 6= 0 n=0

La simetría es: h[n] = −h[−n]. b) Se pueden utilizar las ventanas de Hanning, Hamming, Blackman y Kaiser: • Hanning: M = 125. • Hamming: M = 133. • Blackman: M = 223. • Kaiser: M = 90. c) hD [n = 2] = 1, 3615 · 10−5 hD [n = M − 2] = −1, 3615 · 10−5 hD [n = 2M ] = 0 d) β = 3, 395 Se trata de un sistema FIR lineal, causal de tipo 3. Sólo puede ser un sistema tipo 3 y 4. Problema 4.2.14 Se desea diseñar un filtro paso alto, utilizando el sistema de la figura 4.15, con las siguientes especificaciones: Banda atenuada: |Ha (ω)| ≤ 0,01, 0 ≤ |ω| ≤ 7π krad/s Banda de paso: 0,85 ≤ |Ha (ω)| ≤ 1, |ω| ≥ 8π krad/s

x(t)

A/D

FILTRO DISCRETO HD(W)

Ts

D/A

y(t)

Ts Ha(w) Figura 4.15

El filtro discreto se obtendrá por aplicación del método de la ventana y la frecuencia de muestreo empleada será de dos veces y media la frecuencia de corte de la banda de paso. a) Represente la plantilla de especificaciones en escala semilogarítmica (eje de ordenadas expresado en dB’s) del módulo de la respuesta en frecuencia que debe cumplir el filtro analógico equivalente Ha (ω). b) Represente la plantilla de especificaciones en escala semilogarítmica (eje de ordenadas expresado en dB’s) del módulo de la respuesta en frecuencia que debe cumplir el filtro digital HD (Ω).

Filtros Digitales.

134

c) Para diseñar el filtro se decide emplear el método de la ventana. A partir de la tabla adjunta, indique qué ventanas –considere sólo la rectangular, de Hanning, de Hamming y de Blackman– satisfacen los requisitos anteriores. d) Seleccione la ventana de las indicadas en el apartado anterior que presente menor atenuación en la banda eliminada. Empleando dicha ventana, obtenga la anchura de la banda de transición y el orden del filtro resultante. e) Represente la plantilla de especificaciones, en escala lineal, que cumplirá el módulo de la respuesta en frecuencia del filtro digital diseñado. f) Obtener la expresión de la respuesta al impulso del filtro digital así como el valor de los coeficientes h[0], h[M ] y h[M + 15]. Solución

a) La plantillas es:

|Ha(ω)|

|Ha(ω)| (dBs)

1 0,85

0 -1,41

0,01

-40 7π





ω(krad/s)



ω(krad/s)

b) La plantilla es: |HD(Ω)|

|HD(Ω)| (dBs)

1,081 1 0,919

0,676 0 -0,73

0,0108

-39,33 0,7π 0,8π

π Ω(rad)

0,7π

c) La ventana de Hanning, de Hamming y de Blackman. d) La de Hanning: ∆Ω = 0, 0987π, M = 63 e) La plantilla de especificaciones es:

0,8π

π Ω(rad)

4.2. Diseño de Filtros Digitales FIR

135

|HD(Ω)| 1,0064 1 0,9936

0,0064 0,70065π f) h [n] =

h

sen(π[n−31,5]) π[n−31,5]



sen(0,75π[n−31,5]) π[n−31,5]

i

0,79935π

· 0, 5 − 0, 5 · cos

h [0] = h [63] = 0 h [78] = 0

Ω(rad) 2πn 63



Problema 4.2.15 Se desea obtener la respuesta al impulso de un filtro FIR paso bajo que cumpla las siguientes especificaciones: 0,93 ≤ |H (Ω)| ≤ 1,029 ∀

0 ≤ |Ω| ≤ 0,25π

Atenuación mínima de 50 dBs para 0,45π ≤ |Ω| ≤ π Determinar: a) Ventanas que pueden utilizarse para el diseño y longitud de las mismas. b) Valor de h[9], h[25] y h[50] si se utiliza la ventana de longitud intermedia. Solución a) Utilizar las ventanas de Hamming, Blackman y Kaiser. • Hamming, Longitud (M+1)=35 • Blackman, Longitud (M+1)=57 • Kaiser, Longitud (M+1)=31 b) La ventana de Hamming, Longitud (M+1)=35 h [9] = h [34 − 9] = 0,0136 h [50] = 0

Filtros Digitales.

136

Problema 4.2.16 La figura 4.16 representa el módulo (en dB) de parte de la respuesta en frecuencia de una ventana rectangular de orden M. Determinar razonadamente el valor de M.

|W(Ω )| (dB)

10

10

10

2

0

0,4189

−2

0

0.2

0.4

0.6 Ω(rad)

0.8

1

1.2

Figura 4.16

Solución M = 14 Problema 4.2.17 Se desea obtener la respuesta al impulso de un filtro FIR paso bajo de fase lineal tipo I y retardo de grupo τ = 26 que cumpla las siguientes especificaciones:  0,89 ≤ |H (Ω)| ≤ 1,025 ; 0 ≤ |Ω| ≤ 0,27π αa ≥ 52 dBs ; 0,42π ≤ |Ω| ≤ π Determinar: a) Ventanas que pueden utilizarse para el diseño y longitud de las mismas. b) Expresión de la respuesta al impulso del filtro si se utiliza la ventana de menor longitud, exceptuando la ventana de Kaiser. Solución a) Ventanas de Hanning longitud mínima 46 y Kaiser de longitud mínima 42. b) h [n] =

(

sen(0,345π(n−26)) π(n−26)

· 0,54 − 0,46 · cos 0

2πn 52



; 0 ≤ n ≤ 52 ; resto de n

4.2. Diseño de Filtros Digitales FIR

137

Problema 4.2.18 Para procesar una señal de tiempo continuo xc (t) se va a utilizar el diagrama de bloques de la figura 4.17, donde los bloques C/D y D/C son ideales.

Figura 4.17

El sistema Hd (z) es un filtro FIR paso alto de fase lineal obtenido por aplicación del método de la ventana. Las especificaciones que debe cumplir dicho filtro son: Pulsación de corte de la banda de paso: 0,56π rad. Pulsación de corte de la banda de atenuada: 0,44π rad. Rizado en la banda de paso: r = 0,5 dB. Atenuación mínima en la banda eliminada: αa = 40 dB. Se pide: 1. Dibujar la plantilla correspondiente al módulo de la respuesta en frecuencia que debe cumplir el filtro discreto Hd (Ω). 2. Indicar qué ventanas de las indicadas en la documentación adjunta pueden utilizarse para el diseño y calcular para cada una de ellas el orden del filtro y en su caso el factor de forma. Si se utiliza para el diseño la ventana de Hamming: 3. Obtener la expresión de la respuesta al impulso del filtro discreto, hd [n]. 4. Obtener los valores de hd [26], hd [30] y hd [75]. 5. Dibujar la respuesta de fase del filtro discreto para |Ω| < π. 6. Determinar la frecuencia de muestreo a utilizar si se desea que la frecuencia de corte de la banda de paso del sistema continuo, He (ω) sea 1,68 kHz. Nota:



1 − δp 1 + δp αa = −20 log(δa )

r = −20 log



Solución

1.

δp = 0,029 ; δa = 0,01 ;

Ωa = 0,44π

; Ωp = 0,56π

Filtros Digitales.

138

Figura 4.18

2.

3.

4.

  MHanning = 52,    M Hamming = 56,  MBlackman = 93,     MKaiser = 38 ; β = 3,395      sen(π(n − 28)) sen(π/2(n − 28)) 4πn   · 0,54 − 0,46 cos , 0 ≥ n ≥ 56, −  π(n − 28) π(n − 28) 56 hd [n] =    0 , resto n. hd [26] = hd [30] = hd [75] = 0

5. φ{Hd (Ω)} = −

M Ω = −26Ω. 2

Figura 4.19

6. fs = 6 kHz

4.3. Estructuras de Filtros Digitales

4.3.

139

Estructuras de Filtros Digitales

Problema 4.3.1 Las siguientes funciones de transferencia representan dos filtros diferentes que cumplen la misma plantilla de especificaciones de respuesta en amplitud: i) H (z) =

0,4981819 + 0,9274777z −1 + 0,4981819z −2 1 − 0,6744878z −1 + 0,3633482z −2

ii) H (z) =

11 X

h(k)z −k ,

k=0

donde: h(0) = 0,54603280 · 10−2 = h(11),

h(1) = −0,45068750 · 10−1 = h(10)

h(4) = −0,63428410 · 10−1 = h(7),

h(5) = 0,57892400 = h(6)

h(2) = 0,69169420 · 10−1 = h(9),

h(3) = −0,55384370 · 10−1 = h(8)

Para cada filtro: a) Indique si se trata de un sistema FIR o IIR. b) Dibuje un diagrama de bloques para cada filtro y escriba la correspondiente ecuación en diferencias. c) Determine el número de operaciones y de registros de almacenamiento que se necesitan. Solución a) El primer filtro se trata de un filtro IIR y el segundo es un filtro FIR. b) i) y [n] = 0,6744878y [n − 1] − 0,3633482y [n − 2] + 0,4981819x [n] + + 0,9274777x [n − 1] + 0,4981819x [n − 2]

0,498 y[n]

-1

x[n] -0,363

z

0,927

-1

z

0,498

0,674 ii) y [n] =

11 X

k=0

h(k) · x [n − k]

Filtros Digitales.

140

z -1

x[n]

z -1

z -1

z -1

z -1

z

h[0]

z-1

z -1

z -1

z -1

z -1

h[1]

h[2]

h[3]

h[4]

h[5]

-1

y[n] c) Forma directa II: • Registros de almacenamiento: coeficientes: 5, datos: 2 • Sumas: 4 • Multiplicaciones: 5 Fase Lineal: • Registros de almacenamiento: coeficientes: 6, datos: 11 • Sumas: 11 • Multiplicaciones: 6 Problema 4.3.2 En la figura 4.20 se representa una realización en la forma acoplada de una función del sistema que presenta una pareja de polos conjugados.

x[n]

-1

rcosθ

z

rsenθ

-rsenθ

rcosθ

-1

y[n]

z

Figura 4.20

Se pide: a) Obtener su función del sistema. Dibujar una estructura equivalente según la forma directa IV. b) Dibujar la forma traspuesta a la dada en la figura 4.20, demostrando que tiene la misma H(z). c) Considerando la estructura de la figura 4.20, realizar un algoritmo en seudocódigo que permita el cálculo de la salida y[n] a partir de la entrada x[n].

4.3. Estructuras de Filtros Digitales

141

d) Indicar el número de operaciones y de registros de almacenamiento que se necesitan en cada una de las estructuras obtenidas. Solución a) H (z) = Forma directa IV:

rsenθ · z −1 Y (z) = X (z) 1 − 2r cos θ · z −1 + r2 · z −2

r sen(θ) x[n]

z

y[n]

-1

z

2r sen(θ) -1

-r 2

b) La forma traspuesta es:

w[n-1]

z

y[n]

-1

r cos θ

w[n] r sen θ q[n]

x[n]

r cos θ z

-1

v[n] c) y[-1]=0; v[-1]=0; Leer x[n] w[n]=x[n] - r senθ v[n]=w[n] + r cosθ y[n]=r senθ v[n-1] y[n-1]=y[n] v[n-1]=v[n] volver a leer x[n]

y[n-1] v[n-1] + r cosθ y[n-1]

y repetir proceso.

d) Estructura Original: • Registro de almacenamiento: ◦ Coeficientes: 3 ◦ Retardos: 2

-r sen θ

Filtros Digitales.

142 • Operaciones: ◦ Sumas: 3

◦ Multiplicaciones: 4 Estructura traspuesta: • Registro de almacenamiento: ◦ Coeficientes: 3 ◦ Retardos: 2 • Operaciones: ◦ Sumas: 3

◦ Multiplicaciones: 4 Estructura forma directa IV: • Registro de almacenamiento: ◦ Coeficientes: 3 ◦ Retardos: 2 • Operaciones: ◦ Sumas: 1

◦ Multiplicaciones: 3 Problema 4.3.3 Para la función de sistema indicada en la ecuación siguiente dibuje, empleando diagramas de flujo, una realización en cascada y en paralelo con sistemas de segundo orden.    10 · 1 − 21 z −1 · 1 − 23 z −1 · 1 + 2z −1       H (z) = 1 − 34 z −1 · 1 − 81 z −1 · 1 − 12 + j 21 z −1 · 1 − 21 − j 12 z −1 Solución

Realización en cascada:

10

x[n]

-1

7/8

-1

z -3/32 Realización en paralelo:

y[n]

-1

z

z -5

4/3

-1

z -1/2

-4/3

4.3. Estructuras de Filtros Digitales

143

24,75 -1

z

2,35

-1

z

x[n]

y[n]

-1/2 -14,75 -1

z

7/8

-12,89

-1

z -3/32

Problema 4.3.4 Determine la función de sistema y la respuesta al impulso de los filtros siguientes:

3 2 x[n]

z

0,5

y[n]

-1

2

Sistema 1

5

z -1

3

y[n]

x[n] -1/3

0,5

z -1 2 Sistema 2

Obtenga la forma traspuesta del primer grafo, verificando que tiene la misma función de sistema.

Filtros Digitales.

144 Solución Sistema 1 H (z) = 2 +

6 1 − 0,5z −1

h [n] = 2 · δ [n] + 6 ·

|z| > 0,5  1 n 2

· u [n]

3 w

2 x[n] 2 Sistema 2

y[n]

-1

z

v 0,5

−1 6 + 14 − 43 z −2 3 z 1 − 61 z −1 − 16 z −2 n   n −1 1 u [n] − 8 · u [n] h [n] = 8 · δ [n] + 6 · 2 3

HT (z) =

Problema 4.3.5 Determine d1 y a2 en función de b1 y b2 para que los dos sistemas siguientes sean equivalentes:

x[n]

y[n]

z-1

b1

z-1

b2

c0 x[n]

z -1 d1

a2

Solución Dos soluciones: b1 = d1 b 2 = a2 o bien: b 1 = a2 b2 = d1

z -1 c1

y[n]

4.3. Estructuras de Filtros Digitales

145

Problema 4.3.6 Para la siguiente función de sistema: H (z) =

1 + 2z −1 + z −2 1 − 34 z −1 + 18 z −2

dibuje, empleando diagramas de flujo, todas las posibles realizaciones en cascada y en paralelo empleando sistemas de primer orden. Solución 1. Conexión en cascada:

y[n]

x[n] z

-1

z 1/4

1/2

x[n]

z

-1

-1

z

-1

z

-1

z

1/2

1/4

Podrían obtenerse también las traspuestas. 2. Conexión en paralelo:

2 -1

z 1/2 -9

x[n]

-1

z 1/4 8

y[n]

-1

y[n]

Filtros Digitales.

146

Problema 4.3.7 El diagrama de flujo de la figura 4.21 muestra un sistema no computable; es decir, no es posible calcular la salida empleando las ecuaciones en diferencias representadas en el diagrama ya que presenta lazos cerrados que no contienen elementos de retardo.

x[n] b

y[n]

a z-1

a

-1

Figura 4.21

a) Escriba la ecuación en diferencias del filtro y obtenga la correspondiente función del sistema. b) De la función de sistema represente una nueva estructura que sí sea computable. Solución

a) y [n] = x [n] − by [n − 1] H (z) =

1 1 + bz −1

b) La nueva estructura es:

x[n]

z

-1

y[n]

-b Problema 4.3.8 Considere un sistema LTI causal cuya función de sistema es: H (z) =

1+

1 −1 4z



1 + 51 z −1  · 1 − 21 z −1 + 13 z −2

a) Dibuje un diagrama de flujo del sistema para cada una de las siguientes formas: i) Directa I ii) Directa II iii) Cascada empleando secciones de primer y segundo orden con la forma directa II iv) Paralelo empleando secciones de primer y segundo orden con la forma directa II v) Forma directa II traspuesta b) Obtenga la ecuación en diferencias para la estructura v) del apartado “a” y demuestre que tiene la misma función de sistema.

4.3. Estructuras de Filtros Digitales

147

Solución a)

i) Directa I

x[n]

-1

-1

z

1/4 1/5

z

y[n]

-1

-5/24

z

-1

-1/12

z

ii) Directa II

x[n]

z

1/4

z

-5/24

z

-1/12

y[n]

-1

-1

1/5

-1

iii) Cascada empleando secciones de primer y segundo orden con la forma directa II

x[n]

z -1/4

-1

1/2 1/5 -1/3

z z

-1

y[n]

-1

iv) Paralelo empleando secciones de primer y segundo orden con la forma directa II

3/125 x[n]

-1

-1/4

z

122/125 -1

1/2

z

-1

-1/3

z

-4/125

y[n]

Filtros Digitales.

148 v) Forma directa II traspuesta

x[n]

z

y[n]

-1

1/4

1/5 z

z

-1

-5/24

-1

-1/12

b) 1 5 1 1 y [n] = x [n] + x [n − 1] + y [n − 1] − y [n − 2] − y [n − 3] 5 4 24 12 Problema 4.3.9 Un sistema lineal e invariante se puede realizar empleando el siguiente diagrama de flujo:

y[n]

x[n]

z

-1

z

-1

3

z

-1

1

1

2

z

-1

a) Escriba la ecuación en diferencias que lo caracteriza. b) Calcule la función de sistema. c) En la estructura anterior, ¿cuantas multiplicaciones y sumas reales son necesarias para calcular cada muestra de la señal de salida?.(Suponga que x[n] es real y no contabilice las multiplicaciones por uno). d) La estructura anterior requiere cuatro registros de almacenamiento (unidades de retardo). ¿Es posible reducir este número empleando una estructura diferente?. Si responde afirmativamente, dibuje dicha estructura, y en caso contrario explique porqué no se puede reducir. Solución a) y [n] = x [n] + 4y [n − 1] − 7y [n − 3] − 2y [n − 4] b) H (z) = c) Sumas: 4, multiplicaciones: 2 d) No

1 Y (z) = X (z) 1 − 4z −1 + 7z −3 + 2z −4

4.3. Estructuras de Filtros Digitales

149

Problema 4.3.10 Un sistema LTI cuya función de sistema es: H (z) =

1 − 2z −1 + 78 z −2



6 0,2 1 + z −1   · 1 + z −1 + 12 z −2 · 1 − 21 z −1 + z −2

se puede realizar empleando el diagrama de flujo de la figura 4.22:

x[n]

z -1

z

-1

z

z

z

-1

z

z

-1

z

y[n]

-1

-1

-1

-1

Figura 4.22

a) Encuentre todos los coeficientes del diagrama de la figura 4.22. ¿Es única la solución? b) Defina todas las variables de nudo que crea convenientes y escriba las ecuaciones en diferencias que caracterizan a cada una de las secciones. Solución a) La solución no es única

0,2 a[n] x[n]

z

b[n]

-1

0,4

2

-1

z-1 0,2

-7/8

z

c[n] -1

2 -1

z

d[n]

z-1

2

1/2

z-1

-1/2

-1

b) Sección 1: a [n] = 0,2x [n] + 0,4x [n − 1] + 0,2x [n − 2] + 2a [n − 1] − 78 a [n − 2] Sección 2: b [n] = a [n] − b [n − 1] − 12 b [n − 2] c [n] = b [n] + 2b [n − 1] + b [n − 2] Sección 3: d [n] = c [n] + 2d [n − 1] + d [n − 2] y [n] = d [n] + 12 y [n − 1] − y [n − 2]

z

-1

z-1

y[n]

Filtros Digitales.

150

Problema 4.3.11 La estructura de la figura 4.23 es una sección LTI de dos polos y puede ser empleada en la realización de bancos de osciladores digitales. Para obtener la secuencia de salida se pueden elegir dos opciones: x1i [n] ó x2i [n], dependiendo de la función del sistema que se desee.

x[n] -1

z

x1i[n] ki -ki

x2i[n]

-1

z

Figura 4.23

Se pide: a) Obtener la función del sistema H1 (z) = X1i (z)/X (z) b) Obtener la función del sistema H2 (z) = X2i (z)/X (z) c) Representar ambos filtros empleando diagramas de flujo mediante la forma directa II traspuesta. d) Contabilizar en todas las estructuras el número de registros de almacenamiento necesarios para coeficientes y datos, así como el de multiplicaciones y sumas necesarios por muestra de salida (no considerar multiplicaciones por la unidad). Solución a)

b)

 z −1 1 − z −1 X1i (z) = H1 (z) = X (z) 1 − (2 − ki2 ) z −1 + z −2  ki z −1 1 − z −1 X2i (z) H2 (z) = = X (z) 1 − (2 − ki2 ) z −1 + z −2

c) Sistema 1:

ki

x[n]

z -1 2-k 2i z -1

-k i

-1

x 2i [n]

4.3. Estructuras de Filtros Digitales

151

Sistema 2:

x 2i [n] z-1 2

2-k i x[n]

-1

z -1

-1

d) Sistema 1: • Registros: coeficientes: 3, retardo: 2 • Multiplicaciones: 3 • Sumas: 2 Sistema 2: • Registros: coeficientes: 4, retardo: 2 • Multiplicaciones: 4 • Sumas: 2 Problema 4.3.12 La estructura de la figura 4.24 representa una realización obtenida a partir de la representación en variables de estado de un sistema con dos polos.

z

-1

s1 3

-1/2

y[n]

x[n] 1/2 z

-1

s2

Figura 4.24

Se pide: a) Obtener la función del sistema H (z) = Y (z)/X (z). b) Representar el filtro mediante la forma directa II empleando diagramas de flujo. c) Contabilizar en ambas estructuras el número de registros de almacenamiento necesarios para coeficientes y datos, así como el de sumas (de dos en dos términos) y multiplicaciones (no considere las multiplicaciones por la unidad) que se realizan por cada muestra de salida.

Filtros Digitales.

152

El filtro anterior se puede caracterizar por las siguientes ecuaciones matriciales: 

s1 [n + 1] s2 [n + 1]



=

y [n] =

 

1 1

     1 s1 [n] x [n] + · 0 s2 [n]

−1/2 0

3 1/2



·



s1 [n] s2 [n]



+ x [n]

en las que s1 [n] y s2 [n] representan el valor en el instante n de los nodos (registros) s1 y s2 respectivamente, y s1 [n + 1] y s2 [n + 1] el valor de dichos registros en el instante n + 1. Sabiendo que s1 [−1] = s2 [−1] = 0, se pide: d) Calcular las cuatro primeras muestras de la respuesta al impulso (h[0], h[1], h[2] y h[3]). e) Considerando la estructura de la figura 4.24, realizar un algoritmo en seudocódigo que permita el cálculo de la salida y[n] a partir de la entrada x[n]. Solución

a) H (z) =

1 + 2z −1 + z −2 Y (z) = X (z) 1 − z −1 + 21 z −2

b) Forma directa II:

x[n]

y[n] z

-1

2 -1/2

z

-1

c) Representación en variables de estado: • Registros: coeficientes: 3, retardo: 2 • Multiplicaciones: 3 • Sumas: 4 Representación en Forma Directa II: • Registros: coeficientes: 2, retardo: 2 • Multiplicaciones: 2 • Sumas: 4 d) h [0] = 1 h [1] = 3 h [2] = 72 h [3] = 2

4.3. Estructuras de Filtros Digitales

153

e) Inicializar registros s1 y s2 x ← leer x [n] % Calcular y [n] y ← 3 · s1 + 12 · s2 + x Escribir y [n] s2F ← s1 % registro intermedio 1 s1 ← s1 − 2 · s2 + x % actualizar s1 s2 ← s2F % actualizar s2 Volver a leer siguiente x y repetir hasta aquí Problema 4.3.13 La estructura de la figura 4.25 representa una realización obtenida a partir de la representación en variables de estado de un sistema estable con dos polos.

B

A z -1

2

1/4 x[n]

y[n] -1/4 C

z -1

D

Figura 4.25

Se pide: a) Obtener la función del sistema H (z) = Y (z)/X (z). b) Contabilizar en dicha estructura el número de multiplicaciones y sumas –realizadas de dos en dos elementos– por cada muestra de entrada. No considere las multiplicaciones por la unidad. c) Sabiendo que el filtro digital se ha obtenido empleando la transformación bilineal –con Td = 2–, se pide encontrar la función del sistema analógico Hc (s) que ha sido la base del diseño anterior. Comente la causalidad de ambos sistemas, analógico y digital. Solución a) H (z) =

2z −1 − 41 z −2 Y (z) = X (z) 1 − z −1 − 14 z −2

b) Multiplicaciones: 3, Sumas: 4 c)

 (s − 1) · s + 97 9 Hc (s) = − · , 7 (s + 1,5224) · (s − 0,0938)

Ambos sistemas son no causales.

−1,5224 < Re(s) < 0,0938

Filtros Digitales.

154 Problema 4.3.14 Dada la estructura de la figura 4.26, donde a ∈ R+ (0 < a < 1).

-1

-a

x[n]

y[n]

z-1 Figura 4.26

Se pide: a) Obtener la función del sistema Hd (z) = Y (z)/X (z). b) Representar el filtro mediante la forma directa II traspuesta empleando diagramas de flujo. c) Contabilizar en ambas estructuras el número de registros de almacenamiento necesarios para coeficientes y datos, así como el de sumas (de dos en dos términos) y multiplicaciones (no considere las multiplicaciones por la unidad) que se realizan por cada muestra de salida. d) Dibujar el módulo de la respuesta en frecuencia del filtro dado. e) Si el filtro ha sido diseñado mediante el método de la transformación bilineal, con Td = 2 , se pide encontrar la función del sistema Hc (s) de un filtro de tiempo continuo que pueda haber sido la base del diseño anterior. Dibujar el diagrama de polos y ceros de Hc (s). Solución a) Hd (z) =

a + z −1 Y (z) = X (z) 1 + az −1

b) Forma directa IV:

a x[n]

z-1 -a

c) Sistema 1: • Registros: coeficientes: 2, retardo: 1 • Multiplicaciones: 2 • Sumas: 3

y[n]

4.3. Estructuras de Filtros Digitales

155

Forma directa IV: • Registros: coeficientes: 2, retardo: 1 • Multiplicaciones: 2 • Sumas: 2 d) Módulo de la respuesta en frecuencia:

|H(Ω)| 1

−π

π Ω

siendo periódica con periodo 2π. e) Hc (s) = −

s+ s−

a+1 a−1 a+1 a−1

El diagrama de polos y ceros es:

Plano S

j

X

-(a+1)/(a-1)

(a+1)/(a-1)

Problema 4.3.15 En la estructura de la siguiente figura: -1

-1

z

x[n]

z -1

y[n]

b1 -1

z -1 -b1

Se pide:

-1

z

-b2

Filtros Digitales.

156

a) Determine la función de sistema H (z) = Y (z)/X (z) y la ecuación en diferencias que relaciona la salida (y[n]) y la entrada (x[n]). b) Dibuje el diagrama de polos y ceros del filtro anterior si b1 = b2 = 0, 5. A la vista de dicho diagrama indique de que tipo de filtro se trata, razona la respuesta. c) Realice el sistema anterior mediante la forma directa II traspuesta. Solución a) H (z) =

b2 − b1 (b2 + 1) z −1 + z −2 1 − b1 (b2 + 1) z −1 + b2 z −2

y [n] − b1 (b2 + 1) y [n − 1] + b2 y [n − 2] = b2 x [n] − b1 (b2 + 1) x [n − 1] + x [n − 2] b) Diagrama de polos y ceros:

Parte Imaginaria

1

0,5

0

−0,5

−1 −1,5

−1

−0,5

0

0,5

1

1,5

Parte Real

Filtro paso-todo con |H (Ω)| = 1,

∀ Ω.

c) Forma directa II traspuesta: x[n]

y[n]

0,5

-0,75

z-1 z-1

0,75

-0,5

Problema 4.3.16 La estructura en celosía es ampliamente utilizada en procesado de voz y en el diseño de filtros adaptativos. Un sistema en celosía presenta una serie de etapas conectadas tal como se muestra en la figura 4.27. La estructura de cada una de las etapas se representa en la figura 4.28 siendo km el parámetro de la celosía correspondiente a la etapa m-ésima conocido como coeficiente de reflexión.

4.3. Estructuras de Filtros Digitales

f1[n]

f0[n] x[n]

157 f2[n]

Etapa 1

fN-2[n]

Etapa 2

g0[n]

fN-1[n] = y[n] Etapa N-1

g1[n]

g2[n]

gN-2[n]

gN-1[n]

Figura 4.27

fm-1[n] km km

-1

gm-1[n]

z

+

fm[n]

+

gm[n]

Figura 4.28

Se pide: a) Determine la función de sistema H (z) = Y (z)/X (z) para un sistema en celosía de una única etapa en función de su coeficiente de reflexión. b) Dada H (z) = 1 + 53 z −1 + 21 z −2 , obtener los parámetros y dibujar la estructura en celosía para dicho sistema. Solución a) H (z) =

Y (z) = 1 + k1 · z −1 X (z)

b) Parámetros: k1 = k2 =

2 5 1 2

Estructura en celosía:

f1[n]

f0[n] k1

+

k2

+

f2[n] = y[n]

x[n] -1

g0[n]

z

k1

+ g1[n]

-1

z

k2

+

g2[n]

Filtros Digitales.

158 Problema 4.3.17 En el sistema cuyo diagrama de flujo se representa en la siguiente figura:

2

x[n]

y[n]

-2 z

-1

z

1/2

-1

1/2

-2

Se pide: a) Suponiendo que parte de reposo inicial, determinar la función de sistema y la ecuación en diferencias que relaciona la salida (y [n] ) y la entrada (x [n] ). b) Determinar la función de transferencia del sistema a conectar en cascada con el anterior para que el conjunto sea un filtro paso todo con ganancia en continua unidad. Solución a) H (z) =

Y (z) 2 − 3z −1 + 5z −2 , = X (z) 1 − z −1 + 14 z −2

|z| >

1 2

1 y [n] = y [n − 1] − y [n − 2] + 2x [n] − 3x [n − 1] + 5x [n − 2] 4 b) H2 (z) =

1 − 4z −1 + 4z −2 1 − 35 z −1 + 25 z −2

Problema 4.3.18 Los diagramas de flujo de la figura 4.29 representan la estructura de diseño de dos filtros digitales.

z

-1

1/2

x1[n]

z

-1

y1[n] 1/6

1/9 (a)

z

-1

-2/3 x2[n]

z

-1

y2[n] -1/2

α (b)

Figura 4.29

4.3. Estructuras de Filtros Digitales

159

Determinar: a) Función del sistema H1 (z) = Y1 (z)/X1 (z) de la figura 4.29 (a) y la ecuación en diferencias que relacionan la salida (y1 [n] ) y la entrada (x1 [n] ). b) Valor de la transmitacia “α” de la estructura de la figura 4.29 (b) para que ambos filtros presenten la misma función del sistema. Solución a) H1 (z) =

Y1 (z) X1 (z)

=

1

1 −1 2z  − 13 z −1

y1 [n] = 31 y [n − 1] + 12 x1 [n] b) α = 0 Problema 4.3.19 La respuesta al impulso de un sistema LTI es: ( an 0 ≤ n ≤ 7 h[n] = 0 resto Se pide: a) Dibuje el diagrama de flujo empleando la forma directa.

b) Demuestre que la correspondiente función de sistema se puede expresar como:  1 − a8 z −8 H (z) = |z| > |a| (1 − az −1 ) c) Dibuje el diagrama de flujo de otro sistema que tenga la misma H(z) y que sea un sistema FIR en cascada con un sistema IIR (suponga |a| < 1). d) Indique qué realización requiere: • más registros de almacenamiento (datos y coeficientes). • más operaciones (multiplicaciones y sumas por muestra de salida). Solución a) Diagrama de flujo:

z

x[n] 1

-1

z a

-1

a1

z

-1

a2

z

-1

a3

z

-1

a4

z

-1

a5

z

-1

a6

z

-1

a7 y[n]

Filtros Digitales.

160 b) Calcular la transformada y reagrupar términos. c) El diagrama de flujo es: -1

z

-1

z

-1

z

-1

z

-1

-1

z

-1

z

-1

z

z

x[n]

8

-a

y[n] -1

z a

d) La segunda realización necesita menos registros de almacenamiento y menos operaciones de suma y multiplicación. Problema 4.3.20 El diagrama de flujo de la figura representa una estructura de un sistema de tiempo discreto causal.

1/2

-2 z -1

2

-2 z -1

z -1

x[n]

z -1

y[n]

-1

2 -2 Se pide: a) Obtener la función del sistema:

Y (z) H (z) = = X (z)

M P

bk z k=0 N P

1−

−k

ak

. z −k

k=1

b) Obtener los coeficientes de reflexión y dibujar el diagrama de flujo de la estructura en N P ak z −k , celosía correspondiente a un sistema FIR cuya función del sistema, H1 (z) = 1− es igual al denominador de H(z).

k=1

c) Obtener los coeficientes y dibujar el diagrama de flujo de la estructura en celosía correspondiente al sistema definido por H(z).

4.3. Estructuras de Filtros Digitales

161

d) Razona, basándote en los coeficientes de reflexión obtenidos, si son o no estables los dos sistemas definidos. Solución a) H(z) = b) k1 = 32 ;

Y (z) X(z)

=

2+z −1 +z −2 1+3z −1 +3z −2 + 12 z −3

k2 = 2;

k3 =

1 2

y[n] x[n]

k1 k1

z-1

z

z

v1 = −1;

k3

k2 -1

-1

c) v0 = 32 ;

k3

k2

v2 = 1

x[n] k2 k2

k1 k1 z-1

k3 k3 z-1

z-1

y[n]

d) H1 (z) es causal y estable. H(z) no puede ser causal y estable puesto que los coeficientes de reflexión no son menores que la unidad. Problema 4.3.21 En la figura 4.30 se muestra el diagrama de flujo de un sistema de tiempo discreto.

Figura 4.30

Se pide: a) Obtener las ecuaciones de los nodos v[n], w[n], r[n] e y[n]. b) A partir de las ecuaciones anteriores obtener la expresión de la función de sistema, H(z) = Y (z) X(z) .

Filtros Digitales.

162

c) Determinar, razonadamente, el tipo de filtro de que se trata en función de su respuesta de amplitud. d) Obtener los coeficientes y dibujar el diagrama de flujo de la estructura en celosía correspondiente al sistema definido por H(z).

Ecuaciones de la estructura en celosía: Pm−1 (z) =

Pm (z) − km · z −m · Pm (z −1 ) 2 1 − km

Cm (z) = z −m · Pm (z −1 ) Bm−1 (z) = Bm (z) − vm · Cm (z) Solución a)

   

r [n] = w [n] − x [n] v [n] = x [n] − b · v [n − 1] − a · r [n]  w [n] = b · v [n − 1] + v [n − 2]   y [n] = w [n] − a · r [n]

b)

H ((z)) =

a + b · (a + 1) · z −1 + z −2 Y (z) = X (z) 1 + b · (a + 1) · z −1 + a · z −2

c) Puesto que se cumple que: z −2 · P z −1 H ((z)) = P (z) se trata de un sistema paso todo. d) k2 = a ; k1 = b ; V2 = 1 ; V1 = V0 = 0. El diagrama de flujo es:

V2



4.3. Estructuras de Filtros Digitales

163

Problema 4.3.22 1. En la estructura de la figura 4.31, determine la función de sistema H(z) =

Y (z) X(z) .

Figura 4.31

2. Determine la función de transferencia [H1 (z)] de un sistema de orden mínimo que conectado en cascada con el anterior haga que el conjunto tenga una respuesta de amplitud constante con la frecuencia. Nota: Al realizar la conexión de ambos sistemas no debe producirse ninguna cancelación polo-cero. 3. Se desea obtener la respuesta al impulso (h[n]) de un filtro FIR paso alto que cumpla las siguientes especificaciones: 0,93 ≤ |H (Ω)| ≤ 1,029 para 0,45π ≤ |Ω| ≤ π.

Atenuación mínima de 50 dBs para 0 ≤ |Ω| ≤ 0,25π.

Retardo de grupo constante, y su valor no debe superar las 25 muestras. Determinar: a) Ventana(s) que puede(n) utilizarse para el diseño y longitud de la(s) misma(s). b) Expresión de la respuesta al impulso si para el diseño se utiliza la ventana de mayor longitud. c) Valor de h[1], h[18], y h[36]. Solución 1. H(z) =

−2+6z −1 +2z −2 1−8z −1

2. H1 (z) =

z −1 −0,125z −2 1+3z −1 −z −2

3. a) Hamming M = 34 Kaiser M = 30 b) h[n] =

(h

sen(π(n−17)) π(n−17)



sen(0,35π(n−17)) π(n−17)

0

i

· 0,54 − 0,46 cos

2π 34



c) h[1] = 0,00166 ; h[18] = −0,0249 ; h[36] = 0

0 ≤ n ≤ 34 resto de n

Filtros Digitales.

164 Problema 4.3.23 El grafo de la siguiente figura: -k

x[n]

y[n] p

z-1 k

donde k ∈ R y 0 < k < 1, se corresponde con un filtro paso-todo. Se pide:

a) Encontrar razonadamente el valor de “p“ en función de “k“ para que cumpla los requisitos del enunciado.

b) Indicar razonadamente si el sistema puede ser un filtro FIR y si, de todos los que tienen el mismo módulo de la respuesta en frecuencia, es el sistema de fase mínima. Suponga para el resto del problema que k = 0,5. c) Dibujar el diagrama de polos y ceros del sistema. d) Encontrar la función del sistema H2 (z) de orden mínimo que conectado en cascada con el sistema anterior, permite obtener un filtro causal H1 (z) de fase lineal: H1 (z) = Hap (z) · H2 (z) e) Obtener la respuesta al impulso h1 [n]. ¿Qué tipo de filtro de fase lineal es h1 [n]? Razone la respuesta. Solución a) p = 1 − k 2 . b) No puede ser FIR, ni de fase mínima. c) El diagrama de polos y ceros es: H (z) ap

Parte Imaginaria

1

0,5

0

−0,5

−1 −1

−0,5

0

0,5

Parte Real

1

1,5

2

4.3. Estructuras de Filtros Digitales

d)

165

2  1 1 − z −1 2     1 1 −1 1 5 1 −1 H1 (z) = Hap (z) · H2 (z) = z − · 1− z = − + z −1 − z −2 2 2 2 4 2 H2 (z) =

e) La respuesta al impulso es: 5 1 1 h1 [n] = − δ [n] + δ [n − 1] − δ [n − 2] , 2 4 2 se trata de un filtro FIR de fase lineal tipo I. Problema 4.3.24 Dado un sistema FIR causal, de orden dos, con coeficientes reales y con un cero en c1 = 0,25 − 0,25j, se pide: a) Encontrar su función del sistema (H1 (z)), incluida la constante de ganancia, sabiendo que H1 (Ω)|Ω=0 = 1. b) De todos los sistemas que tienen el mismo módulo de la respuesta en frecuencia que H1 (z), ¿qué propiedad satisface el anterior sistema con respecto a la energía parcial acumulada? Razone la respuesta. c) Obtener los coeficientes y dibujar el diagrama de flujo correspondiente a la implementación del sistema, utilizando la estructura en celosía. d) Encontrar H2 (z), incluida la constante de ganancia, del sistema causal de retardo de energía máximo que tiene el mismo módulo de la respuesta en frecuencia que H1 (z). e) Razone la estabilidad de ambos sistemas. f) Se conecta ambos sistemas, H1 (z) y H2 (z), en cascada. Para el sistema resultante indicar de qué tipo de sistema se trata y dibujar el diagrama de flujo correspondiente a su implementación utilizando la estructura que utilice menor número de registros de memoria. Nota: Pm−1 (z) =

Pm (z)−km z −m Pm (z −1 ) 2 1−km

Solución a) H1 (z) =

  8 · 1 − (0,25 + 0,25j) z −1 · 1 − (0,25 − 0,25j) z −1 5

b) Como es un sistema de fase mínima, tiene la propiedad de retardo de energía mínimo. c) H2 (z) =

  1 · 1 − (2 + 2j) z −1 · 1 − (2 − 2j) z −1 5

d) Ambos sistemas son estables pues todos los polos están en el origen.

Capítulo 5

Sistemas de Tasa Múltiple Problema 5.1 Considere un sistema como el de la siguiente figura: x1[n]

x[n]

x2[n] 2

2

y0[n]

Η0(Ω)

y1[n]

Η1(Ω)

con los siguientes datos: X(Ω)

H0(Ω)

1

−π −π/2

H1(Ω)

1

π/2

π Ω

−π −π/2

1

π/2

π Ω

−π −π/2

π/2

π Ω

Calcular y representar gráficamente los espectros de las señales x1 [n], x2 [n], y0 [n], y1 [n]. Solución El espectro de las señales es: Y0(Ω)

X1(Ω)

1/2

1/2

−π −π/2

π/2

π Ω

−π −π/2

X2(Ω)

π Ω

Y1(Ω)

1/2

−π −π/2

π/2

1/2

π/2

π Ω

−π −π/2

π/2

π Ω

Sistemas de Tasa Múltiple.

168

Problema 5.2 En la figura 5.1, las dos señales reales de tiempo discreto x1 [n] y x2 [n] se inyectan en sendas ramas del diagrama de bloques. Cada uno de los bloques que componen la rama superior están definidos, mientras que la rama inferior aún no está diseñada. Las transformadas de Fourier de x1 [n], x2 [n] e y[n], periódicas de periodo 2π, se muestran en la figura 5.2.

L

M

Figura 5.1

Figura 5.2

Se pide:

a) Calcular cada uno de los parámetros de los distintos bloques que aparecen en la rama superior de la figura 5.1. Representar así mismo los espectros de las señales x3 [n], x4 [n] y x5 [n].

b) Representar el diagrama de bloques de la rama inferior de la figura 5.1 detallando el valor de los parámetros más significativos que intervienen en el mismo. Se valorará la sencillez.

Solución

a) L = 3, M = 4, G = 4, Ωc = π/6.

169 X3(Ω) 1

−π −2π/3

−π/3

π/3

4

2π/3

π Ω

X4(Ω) 2

−π −2π/3 −π/3 −π/6 π/6 π/3

1

2π/3

π Ω

2π/3

π Ω

X5(Ω) 1/2

−π −2π/3 −π/3

π/3

b) Rama inferior:

x2[n]

Η(Ω) 1

x6[n]

x7[n]

2 −π −2π/3

2π/3 π Ω

Problema 5.3 En la figura 5.3 se muestra el espectro de la señal x[n] de la que se quieren eliminar las componentes espectrales por encima de 5π/6 radianes. Sin embargo, sólo se disponen de filtros paso bajo ideales de pulsación de corte π/3 radianes. Para conseguir este fin, se emplea el diagrama de bloques de la figura 5.4, donde quedan aún por definir cuatro bloques (S1 − S4 ) de diezmado e interpolación.

Figura 5.3

Sistemas de Tasa Múltiple.

170

Figura 5.4

Se pide: a) Determinar razonadamente cada uno de los cuatro bloques que quedan por definir en la figura 5.4. b) Representar los espectros de las señales x1 [n], x2 [n], x3 [n], x4 [n] e y[n] en el inter valo [−π, π]. Definir el valor de la ganancia G del último filtro para que Y ejΩ =  X ejΩ , ∀ |Ω| ≤ 5π/6. Solución

a) Los bloques son:

S2

S1 x[n]

x1[n]

x1[n]

x5[n]

5

2

S3

S4

x2[n]

x4[n]

x3[n] 2

y[n] 5

b) G = 10.

−π

−π

−3π/5 −π/5

−π/6

X1(Ω)

X2(Ω)

1

1/2

π/5

3π/5

π



−π

−π/3

π/3

X3(Ω)

X4(Ω)

1/2

5

π/6

π



−π

−π/6 π/6

π

π





171

Y(Ω) 1

5π/6 π Ω

−π −5π/6

Problema 5.4 Al sistema de la figura 5.5 se aplica como señal de entrada x[n], cuyo espectro es el representado en la figura 5.6.

x1[n]

y1[n]

s1[n]

2

h[n]

Re{}

2 z-1

y2[n]

2

r1[n]

s2[n]

jIm{}

r2[n]

Figura 5.5

1

3/4 π/2

1/4 −π −3π/4 −π/4

π/4

3π/4 π Ω

π Ω

−π −π/2

Figura 5.6

Se pide: a) Representar el espectro de cada una de las siguientes señales: x1 [n], y1 [n], y2 [n], s1 [n], s2 [n]. b) Expresar Ri (Ω), para 1 ≤ i ≤ 2, en función de Si (Ω). c) Representar el espectro de las señales r1 [n] y r2 [n]. d) Si x1 [n] es una secuencia de longitud finita tal que:   x1 [n] = 1 + j, 2 − j, −1 − 3j, j, 3, −3j

y de valor nulo en el resto de “n”, obtenga los valores de las secuencias y1 [n], y2 [n], s1 [n], s2 [n], r1 [n] y r2 [n].

Sistemas de Tasa Múltiple.

172 Datos:

Los dos últimos bloques del diagrama de la figura 5.5 obtienen como salida ri [n] las partes real (rama superior) e imaginaria multiplicada por j (rama inferior) de la secuencia de entrada si [n]. La respuesta al impulso del primer bloque es: h [n] = ha [n] ∗ ha [n] , donde ha [n] =

√ sen (πn/4) 2π · πn

Solución a) Los espectros son: |X1(Ω)|=|Y1(Ω)|=|Y2(Ω)|

|S1(Ω)|=|S2(Ω)| π/2

π 3π/4 −π

−π/2

3π/8

π/2

π Ω

π Ω

−π

Arg{X1(Ω)}=Arg{Y1(Ω)}

Arg{S1(Ω)}

π/2

π/2 π

−π −π/2

π/2 −π/2

π Ω



−π

Arg{Y2(Ω)}

−π/2

Arg{S2(Ω)}

π

π

π/2

π/2

π −π −π/2

π/2

π

−π/2



−π

Ω −π/2

−π

−π

b) R1 (Ω) = R1 (Ω) =

1 2 1 2

(S1 (Ω) + S1∗ (−Ω)) (S2 (Ω) − S2∗ (−Ω))

c) R1 (Ω) = S1 (Ω) R2 (Ω) = 0 d) y1 [n] = x1 [n] y2 [n] = [0, 1 + j, 2 − j, −1 − 3j, j, 3, −3j] s1 [n] = [1 + j, −1 − 3j, 3] s2 [n] = [0, 2 − j, j, −3j] r1 [n] = [1, −1, 3] r2 [n] = [0, −j, j, −3j]

173 Problema 5.5 Dado el sistema de la figura 5.7, considerando la señal de entrada X(Ω) de la figura 5.8, se pide representar el espectro de cada una de las siguientes señales: r1 [n], r2 [n], y1 [n], y2 [n], y3 [n], s1 [n] y s2 [n].

x[n]

r1[n]

2

jIm{} Re{}

y1[n]

r2[n]

s1[n]

y2[n]

2

h[n]

s2[n] 2

y3[n]

Figura 5.7

1

−π

πΩ

−π

πΩ

Figura 5.8

Datos:

Los dos primeros bloques del diagrama de la figura 5.7 obtienen como salida ri [n] las partes real (rama inferior) e imaginaria multiplicada por j (rama superior) de la secuencia de entrada x[n].

La respuesta al impulso del sistema es: h [n] =

sen (πn) sen (πn/2) − πn πn

Solución R1 (Ω) = 0 R2 (Ω) = X (Ω) Y1 (Ω) = 0

Sistemas de Tasa Múltiple.

174

Αrg{Y2(Ω)} = Αrg{Y3(Ω)}

|Y2(Ω)| = |Y3(Ω)| 1

π/2 3π/4

−π/4 −π

π/2

−π/2

π Ω

−π −3π/4 −π/2

π/4 π/2

πΩ

−π/2 Αrg{S1(Ω)}

|S1(Ω)|

π/2

1 −π −3π/4 −π/2 −π

π/2

−π/2

π/2 3π/4 π Ω

π Ω −π/2

|S2(Ω)|

Αrg{S2(Ω)}

1

π/2 3π/4

−π/4 −π −3π/4 −π/2 −π/4

π/4 π/2 3π/4 π Ω

−π −3π/4 −π/2

π/4 π/2 −π/2

Problema 5.6 Dada la estructura

en la que las señales de entrada son las siguientes: x1 [n] = x3 [n] =





x1a

x1b

x1c

x3a

x3b

x3c

 

,

x2 [n] =

,

x4 [n] =

 

x2a

x2b

x2c

x4a

x4b

x4c

 

, .

πΩ

175 Se pide: a) Obtener las muestras de y2 [n], r1 [n], r2 [n], r3 [n] y r4 [n], considerando h[n] = δ[n]. b) Dibujar el espectro de la señal r4 [n], considerando que h [n] = sen (πn/4)/(πn), y que todas las señales de entrada son nulas salvo x4 [n], cuyo espectro es el siguiente:

Solución a) y2 [n] =



x1a , x2a , x1b + x3a , x2b + x4a , x1c + x3b , x2c + x4b , x3c , x4c

r1 [n] =

r3 [n] = b)





x1a , x1c + x3b

x3a + x1b , x3c





,

r2 [n] =

,

r4 [n] =





x2a , x2c + x4b

x4a + x2b , x4c







,

,

.

Problema 5.7 Dados los sistemas estables de la figura 5.9,

x[n]

H0 (z)

x

y0 [n]

x[n]

Hk (z)

y 1 [n]

p[n] Figura 5.9

Se pide: a) Obtener las expresiones de Y0 (z) e Y1 (z). b) Indicar razonadamente si ambos sistemas responden, para cualquier valor de k, con la misma salida a la misma entrada.

Sistemas de Tasa Múltiple.

176

c) En caso de haber respondido negativamente a la pregunta anterior, encontrar si existe algún conjunto de valores de k en los que ambos sistemas respondan con la misma salida a la misma entrada. Dados los sistemas estables de la figura 5.10,

y[n]

x[n] L

H0 (z)

x

y 0 [n]

x[n]

y[n] L

Hk (z)

y 1 [n]

p[n] Figura 5.10

se pide: d) Obtener las expresiones de Y0 (z) e Y1 (z). e) Indicar razonadamente si ambos sistemas responden, para cualquier valor de k, con la misma salida a la misma entrada. f) En caso de haber respondido negativamente a la pregunta anterior, encontrar si existe algún conjunto de valores de k en los que ambos sistemas respondan con la misma salida a la misma entrada.   2·π Datos: L ∈ Z+ , k, n ∈ Z, hk [n] = h0 [n] · cos 2·π L · k · n , p [n] = cos L · k · n Solución a) Y0 (z) = Y1 (z) =

  2π i   2π  2π 1 h  −j 2π k  X e L z H0 e−j L k z + X ej L k z H0 ej L k z 2  2π i   2π 1h X (z) H0 e−j L k z + X (z) H0 ej L k z 2

b) No responden igual. ˙ c) Cuando k = L. d) Y0 (z) = Y1 (z) = e) Son iguales. f) No hay caso.

 2π i     2π 1h X z L H0 e−j L k z + X z L H0 ej L k z 2   2π i    1h 2π X z L H0 e−j L k z + X z L H0 ej L k z 2

177 Problema 5.8 Se pide: a) Demostrar razonadamente si los sistemas interpolador y diezmador cumplen la propiedad de invarianza en el tiempo. b) ¿Pueden ser equivalentes (r0 [n] = r1 [n]) las dos interconexiones de la figura 5.11? Razone su respuesta con un ejemplo en el que M, L > 1.

x[n]

y0[n] M

r0[n]

?

x[n]

y1[n] L

L

M

r1[n]

Figura 5.11

Solución

a) No la cumplen. b) Si M y L son primos entre sí, por ejemplo, M = 9 y L = 7. Problema 5.9 El diagrama de la figura 5.12, se emplea para descomponer el espectro Xc (jω) de la señal de entrada de la forma siguiente: Y1 (jω) = Xc (jω) , para |ω| < 4π krad/s, Y2 (jω) = Xc (jω) , para |ω| > 4π krad/s, donde Y1 (jω) e Y2 (jω) son los espectros de y1 (t) e y2 (t) respectivamente. Para ello se puede utilizar cualquier técnica de procesado que se considere oportuna a la hora de diseñar los sistemas S1 y S2 . La única restricción a tener en cuenta es que se dispone de cuantos filtros digitales se deseen pero sólo del tipo de los que se representan en la figura 5.13, que son de ganancia G ajustable.

T x1[n]

S1 xc(t)

-

y1(t) D/C

x[n] = xc(nT) T

S2

x2[n]

D/C T

Figura 5.12

y2(t)

Sistemas de Tasa Múltiple.

178

H1(ejΩ)

−π

−π/4

H2(ejΩ)

−π

π Ω

π/4

−π/2 −π/4

π/4 π/2

π Ω

Figura 5.13

Sabiendo que T = 125µs y que el convertidor D/C contiene un filtro paso bajo ideal de ganancia T y pulsación de corte π/T rad/s, se pide: a) Determinar el diagrama de bloques S1 que permite obtener y1 (t). Represente los espectros de todas las señales intermedias, incluida x1 [n]. b) Determinar el diagrama de bloques S2 que permite obtener y2 (t). Represente los espectros de todas las señales intermedias, incluida x2 [n]. NOTA: En los diagramas de bloques S1 y S2 deben especificarse los valores de todos los parámetros que intervengan en el diseño. Se valorará la claridad y la sencillez en el diagrama. Solución a) Rama superior:

x[n]

Η1(Ω) G=2

s1[n] 2

2 −π

−π/

π/4

S1(Ω)

−π/2

π/2

π



S2(Ω) 2/T

1/T

−π

x1[n]

s2[n]

π Ω −π

−π/4

π/4

X1(Ω) 1/T

π Ω

−π

−π/2

π/2

π Ω

179 b) Rama inferior:

Η2(Ω) x[n]

G=2

s1[n]

s3[n]

2 −π

−π

−π/2 −π/4

x2[n] 2

−π/2 −π/4

π/4 π/2

πΩ

S3(Ω)

X2(Ω)

G/T

G/(2T)

π/4 π/2

π Ω

−π

−π/2

π/2

π Ω

Problema 5.10 La figura 5.14 representa una señal Xc (ω) limitada en banda (4.5 kHz.). Se desea procesar las bajas frecuencias (hasta 1.5 kHz.) del espectro de dicha señal, de manera que se modifique su ancho de banda y resulte la señal de salida Y (ω) (también representada en la figura 5.14). Para ello, se emplea el diagrama de bloques de la figura 5.15, en la que la parte de procesado digital tiene que ser diseñada.

1 0,7

Xc ( )

Y( ) 1 0,7

Figura 5.14

xc(t)

C/D

x[n] = xc(nT1)

y[n]

D/C

y(t)

T2

T1 Figura 5.15

Sabiendo que el convertidor D/C contiene un filtro paso-bajo ideal de ganancia T2 y de frecuencia de corte 1/(2T2), se pide: a) Determinar el periodo máximo de muestreo T1 que se puede utilizar en la aplicación. b) Considerando T2 = T1 , representar el diagrama de bloques de procesado digital que hay que incluir en la figura 5.15, detallando el valor de los parámetros más significativos que intervienen en el mismo. Dibujar también los espectros de salida de cada uno de los bloques que introduzca en su diseño.

Sistemas de Tasa Múltiple.

180

Sólo podrá utilizar diezmadores, interpoladores y filtros paso bajo ideales de ganancia y pulsación de corte a elegir. c) Una vez diseñado el sistema de procesado digital, se modifica el período de reconstrucción de manera que T2 = T1 /3. Se pide representar el espectro de la nueva señal analógica de salida y ′ (t). Solución a) T1 =

1 6

· 10−3 s.

b) El diagrama de bloques y los espectros a la salida son:

x[n]

Η(Ω) 4

x1[n]

y[n]

x2[n]

5

4 −π

X1(Ω)

π Ω

−π/10 π/10

X2(Ω)

0.7/T1

1/T1

−π

4/T1

−π/5 −π/10 π/10 π/5

π Ω

−π

−π/5 −π/10 π/10 π/5

Y(Ω) 1/T1 0.7/T1

−π

−4π/10

4π/10

π Ω

c) El espectro es:

Y´(ω) 1/3 0.7/3 −36π/5

36π/5 ω(krad/s)

πΩ

181 Problema 5.11 En el sistema de la figura 5.16, el convertidor D/C contiene un filtro paso-bajo ideal de ganancia T y de frecuencia de corte 1/(2T ). Se pide:

xc(t)

x[n] = xc(nT)

y0[n]

T

r0[n] 2

2

C/D

z -1

z -1

y(t) 2

D/C

2

y1[n]

r1[n]

Sistema de tiempo discreto

y[n] T

Figura 5.16

1 0,7

Xc ( )

Figura 5.17

a) Exprese Y (z) en función de X(z).   b) Si x[n] es una secuencia de longitud finita tal que x [n] = x0 x1 x2 x3 x4 x5 y de valor nulo en el resto de “n”, obtenga los valores de las secuencias y0 [n], y1 [n], r0 [n], r1 [n], y[n]. c) Si a la entrada del sistema completo se aplica la señal de la figura 5.17, represente el módulo del espectro de las señales y0 [n], y1 [n], r0 [n], r1 [n], y el espectro completo de x[n], y[n] e y(t). Considere que el periodo de muestreo T es la mitad del máximo valor que se puede utilizar sin que se produzca solapamiento. Además, en las representaciones deberá indicar los valores más significativos. Solución a) Y (z) = z −1 X (z). b)

r0 [n] =



y1 [n] =

 x0 , x2 , x4 , 0, 0   x0 , 0, x2 , 0, x4 [0, x1 , x3 , x5 ]

r1 [n] =

[0, 0, x1 , 0, x3 , 0, x5 ]

y0 [n] =

y [n] =

[0, x0 , x1 , x2 , x3 , x4 , x5 ]

Sistemas de Tasa Múltiple.

182 c) Los módulos y los espectros completos son:

−π

−2π/3 −π/3

|Y0(Ω)|=|Y1(Ω)|

|R0(Ω)|=|R1(Ω)|

1/2T

1/2T

0,7/2T

π/3

2π/3

π



−π

−π/2−π/3−π/6 π/6 π/3 π/2

X(Ω)=|Y(Ω)| 1/T

0,7/2T

πΩ

|Y(ω)| 1

0,7/2T

0,7

−π

−π/2 −π/3 −π/6 π/6 π/3 π/2

πΩ

−9π

−6π

−3π

9π ω(krad/s)

π/2

π/2

−π/2



Arg{Y(ω)}

Arg{Y(Ω)} π/2 −π



πΩ

−9π

9π ω(krad/s) −π/2

−π/2

Problema 5.12 Se desea procesar digitalmente una señal xc (t). Sin embargo, en el proceso de conversión de tiempo continuo a tiempo discreto se dispone de un convertidor C/D ideal con un período de muestro T fijo. Puesto que los recursos son limitados, se aplica un preprocesado a la secuencia x[n] con el objetivo de optimizar en el sentido de aprovechar al máximo la banda espectral disponible (figura 5.18).

xc(t)

x[n] C/D

y[n] Preprocesado

T Figura 5.18

−4π/3Τ

4π/3Τ Figura 5.19

Los requisitos para y[n] son los siguientes: Si se produce solapamiento al muestrear, la banda sin solapamiento de x[n] deberá ocupar todo el espectro entre [−π, π].

183 Si no se produce solapamiento al muestrear, la banda mínima necesaria que contenga toda la información de la señal original deberá ocupar todo el espectro entre [−π, π]. La etapa de preprocesado no debe introducir ganancia respecto a x[n]. Si el espectro de xc (t) es el representado en la figura 5.19, se pide: a) Diseñar la etapa completa de preprocesado. b) Dibujar los espectros de y[n] y de todas las señales intermedias que se produzcan. NOTA: Todos los bloques que se consideren para el diseño de la etapa de preprocesado deben ser reales, es decir, no deben dar lugar a señales complejas. Solución

a) La etapa de procesado:

x[n]

Η(Ω) 3

x1[n]

y[n]

x2[n]

2

3 −π

−π/3

π/3

π Ω

b) Las señales intermedias:

−π

−2π/3

−π/3

X1(Ω) 1/T

X2(Ω) 3/T

1/2T

3/2T

π/3

2π/3

π Ω

−π

−π/3

π/3

π Ω

Y(Ω) 1/T 1/2T −π

−π/3

π/3

π Ω

Problema 5.13 El sistema de la figura 5.20 se emplea para procesar la señal de tiempo continuo x(t) cuyo espectro se representa en la figura 5.21. Considere que el convertidor D/C contiene un filtro paso bajo ideal, de ganancia T2 y de pulsación de corte π/T2 .

Sistemas de Tasa Múltiple.

184

x(t) C/D

x[n]=x(nT1)

y[n] h[n]

2

yd[n]

2

yi[n]

y(t) D/C T2

T1 Figura 5.20

Figura 5.21

Se pide: a) Obtener la expresión de la señal x[n], y representar su espectro. b) Obtener y representar la respuesta en frecuencia del filtro LTI cuya respuesta al impulso n . es h [n] = (−1) · sen(πn/2) πn c) Representar los espectros correspondientes a las señales yd [n] e yi [n]. d) Obtener la expresión de la señal y(t) en función de T1 , considerando dos casos: d.1) T2 = T1 . d.2) T2 = 2T1 . Solución a) x [n] =

1 T1 δ [n].

X (Ω) 1/T1

−π

π Ω

b) H (Ω) =



1 0

π/2 < |Ω| < π |Ω| < π/2

Η(Ω) 1 −π −π/2

π/2 π Ω

185 c) Los espectros son:

Yd(Ω)=Yi(Ω) 1/2T1

−π −2π/3

2π/3

d) d.1) y (t) =

1 sen 2

d.2) sen y (t) =



π T1 t

πt



π 2T1 t

πt

πΩ





Problema 5.14 Dado el diagrama de bloques de la figura

donde el convertidor D/C contiene un filtro paso bajo ideal de ganancia T1 y frecuencia de corte 1/(2T1 ) Hz, se aplica una señal de entrada cuyo espectro X (Ω) se representa en la siguiente figura:

X

Y

El período de muestreo es T1 = 10−3 s, y el filtro entre ambos convertidores está definido como  G, |ω| < ωc , H (ω) = 0, |ω| > ωc . Se pide: a) Encontrar los valores de ωc , G y T2 para que la señal de salida Y (Ω) contenga la información espectral de X (Ω) en la banda de frecuencia |Ω| < 2π/3, pero ocupando una porción del espectro mayor tal y como se indica en la figura anterior. b) Diseñar un sistema completamente digital que sea equivalente al dado. Dibuje su diagrama de bloques y represente todas las señales intermedias, indicando los valores más significativos. Se valorará la sencillez.

Sistemas de Tasa Múltiple.

186 Solución

a) ωc = (2π/3) · 103 , G = 5/4, T2 = 5/4 · 10−3 . b) El diagrama de bloques del sistema digital es

donde H (Ω) =



5, |Ω| < π/6, 0, |Ω| > π/6.

Los espectros de las señales intermedias son los siguientes:

Problema 5.15 Considérese el diagrama de bloques de la figura 5.22 donde el sistema S2 queda definido en la figura 5.23. Se pide:

xc(t)

x[n]

x0[n]

x0[2n]

y0[n]

x1[n]

x1[2n]

y1[n]

h0[n]

C/D T

h1[n] Figura 5.22

187

Figura 5.23

a) Representar gráficamente las respuestas impulsivas h0 [n] y h1 [n] que caracterizan completamente a los sistemas LTI correspondientes, sabiendo que h0 [n] = w [n] ∗ w [n], siendo n w [n] = u [n] − u [n − 4] y h1 [n] = (−1) h0 [n]. b) Calcular H0 (z) y representar su diagrama de polos y ceros (dicha función presenta al menos un cero en z0 = −j). c) Considerando la relación existente entre h1 [n] y h0 [n], dibujar de forma aproximada el módulo de la respuesta en frecuencia de h1 [n]. d) Dibujar cada una de las señales que aparecen en la figura 5.22 cuando T = 4/π y xc (t) viene dada por

xc(t) 6/π

−π/4

t

π/4

Solución

a) Las funciones de respuesta al impulso son: h0[n]

5 4 3 2 1 0 −2

0

2

4 3 2 1 0 −1 −2 −3 −4 −5 0

1

2

4

n

h1[n]

3

n

4

6

5

8

6

7

Sistemas de Tasa Múltiple.

188 b) H0 (z) = 1 + z −1 1

2

1 + z −2

2

2

Parte Imaginaria

0,5

2

0

6

−0,5

2

−1 −1

−0,5

0 0,5 Parte Real

1

c) La respuesta en frecuencia es:

|H1(Ω)|

−π d) x [n] =

6 π δ [n],

x0 [n] =

6 π h0

−π/2

π/2

[n], x1 [n] =

6 π h1

π



[n], x0 [2n] = x1 [2n], y0 [n] = y1 [n].

x 0 [2n]=x1 [2n] 6 4

18/π

18/π

6/π

6/π

2 0 −1

0

1

n

2

3

4

2

3

4

y0 [n]=y1 [n] 6

18/π

18/π

4 2

0 −1

0

1

n

189 Problema 5.16 Al siguiente diagrama de bloques:

2cos (n π ) xc(t)

x[n]

x0[n]

C/D

y0[n]

p[n]

S1

X

6

T se le introduce como señal de entrada xc (t), cuyo espectro viene dado por la siguiente expresión: sen (ω · π) −j3ω Xc (ω) = ·e ω a) Sabiendo que el período de muestreo del convertidor C/D es T = 1, se pide obtener x[n] y su transformada de Fourier X(Ω). b) El sistema S1 presenta la siguiente relación entrada salida: y0 [n] =

k=−∞

sentar la secuencia y0 [n]. c) Obtener la señal de salida p[n] y su transformada de Fourier P (Ω). Solución

a) 6

x [n] =

1X δ [n − k] 2 k=0

X (Ω) =

1 −j3Ω sen (3,5 · Ω) e 2 sen (Ω/2)

b) La secuencia es:

2

y0[n] 1

0 −2

n P

0

2

4 n

6

8

10

x0 [k]. Repre-

Sistemas de Tasa Múltiple.

190 c) p [n] = P (Ω) =

u [n] ∞ X 1 πδ (Ω − 2πk) + 1 − e−jΩ k=−∞

Problema 5.17 Dos filtros de fase lineal se denominan un par complementario si: |H1 (Ω) + H2 (Ω)| = 1 Sabiendo que la respuesta en frecuencia de un filtro FIR de fase lineal de longitud N y la de su complementario se pueden expresar como: H1 (Ω) = e−j(N −1)Ω/2 R (Ω) , H2 (Ω) = e−j(N −1)Ω/2 (1 − R (Ω)) , se pide: a) Obtener la función del sistema de H2 (z) en función de H1 (z). b) Completar en el diagrama de bloques de la figura 5.24 la parte denominada S1 , utilizando únicamente un filtro H1 (z) y elementos básicos (multiplicadores, sumas y retardos). Considere N impar. c) Dibujar los módulos de los espectros de todas las señales que aparecen en la figura 5.24, sabiendo que Xc (ω) y R(ω) son los espectros representados en la figura 5.25.

Xc( ) x c (t)

Y1( )=X ( )·H1( )

X( )

C/D

x[n ]=x c( nT )

2

S1 Y2( )=X ( )·H2( )

T

2

Figura 5.24

Xc( )

R(Ω)

1

T

−0,51π

T

−π

0,51π

1

−π/2

Figura 5.25

Solución a) H2 (z) = z −(N −1)/2 − H1 (z)

π/2

π

Ω

P1( ) P2( )

191 b) El diagrama de bloques es:

X(z)

H1(z)

Y1(z) -1

z

Y2(z)

- (N-1)/2

c) Los módulos de las señales:

X(Ω)

|Y1(Ω)|

−0,51π

1/T

−π

−π

π Ω

1/T

π/2

−π/2

|P1(Ω)|

|Y2(Ω)|

1/2T

0,49/T

−π −0,98π

0,98π π Ω

−π/2

−π

0,51π

π/2

−0,51π

π Ω

π Ω

0,51π

|P2(Ω)| −0,98π

−π

−π/2

0,49/2T

π/2

0,98π

π



Problema 5.18 Dada una señal digitalizada con una frecuencia de muestreo igual a 48 kHz, se desea reducir el número de muestras para obtener una secuencia cuya frecuencia de muestreo final sea de 400 Hz. Para ello, se procede a diseñar un conjunto de filtros diezmadores y a estudiar el coste computacional de cada una de las implementaciones. Las alternativas son las siguientes: Opción 1: Implementación del filtro diezmador en una única etapa:

Opción 2: Implementación del filtro diezmador en una única etapa, utilizando las M componentes polifase tipo I:

Sistemas de Tasa Múltiple.

192

Opción 3: Implementación del filtro diezmador en dos etapas:

Opción 4: Implementación del filtro diezmador en dos etapas, utilizando las M1 y 8 componentes polifase tipo I para la primera y la segunda etapa, respectivamente. Sabiendo que los órdenes de cada uno de los filtros son iguales a 5490 para H (z), 408 para G (z) y 68 para el F (z), se pide: a) Obtener los factores de diezmado M y M1 . b) Calcular el número máximo de multiplicaciones por cada muestra de entrada que se tienen que realizar en cada una de las cuatro opciones. c) Calcular el coste computacional medido en multiplicaciones por segundo (MPS) necesarias en cada una de las cuatro opciones. Solución a) Los factores de diezmado son M = 120 y M1 = 15. b) El número máximo de multiplicaciones por cada muestra de entrada sería como sigue: Opción 1: 5491. Opción 2: ceil(5491/120)=46. Opción 3: 69+409=478. Opción 4: ceil(69/15)+ceil(409/8)=5+52=57. c) El número de multiplicaciones por segundo en cada una de las cuatro opciones sería como sigue: Opción 1: 2.196.400. Opción 2: 2.196.400. Opción 3: 220.800+163.600=384.400. Opción 4: 384.400.

193 Problema 5.19 Una señal de electrocardiograma (ECG) ha sido digitalizada empleando una tarjeta de sonido que tiene una frecuencia de muestreo igual a 44.1 kHz. Debido al número excesivo de muestras de la señal de ECG discretizada, se procede a reducir la frecuencia de muestreo empleando un diezmador, consistente en un filtro diezmador más un bloque de down-sampling. Se pide: a) Obtener el factor de diezmado necesario para que la frecuencia de muestreo resultante sea de 420 Hz. b) Representar la respuesta en frecuencia ideal del filtro diezmador, sabiendo que se permite una banda de transición con una anchura de 26.25 Hz. Indicar en dicha figura el valor de las frecuencias de corte digitales y analógicas correspondientes a los límites de las bandas de paso y eliminada. c) Si la desviación de la banda de paso del filtro es igual a la de la banda eliminada, de valor δp = δs = 0.001, obtener el orden del filtro empleando la expresión de Kaiser: p −20log10 δp δs − 13 . N= 14,6∆f Calcular el número de multiplicaciones por segundo necesarias para implementar el diezmador completo. d) Para reducir el número de operaciones, se procede a descomponer el diezmador en dos etapas, forzando a que la última etapa de down-sampling tenga un factor de diezmado igual a 7. Dibujar el diagrama de bloques del nuevo sistema, y representar las respuestas en frecuencia de los dos filtros que conforman el mismo. Indicar los valores de las frecuencias de corte analógicas y digitales de cada uno de ellos. e) Si la desviación de la banda de paso del filtro de ambos filtros es δp = 0.0005, y sus respectivas desviaciones en las bandas eliminadas vienen dadas por δs = 0.00085, se pide obtener el orden de cada uno de ellos. f) Obtener el número de multiplicaciones por segundo necesario para implementar este nuevo conjunto de sistemas. Solución

a) El factor de diezmado es M = 105. b) La respuesta en frecuencia deseado sería la siguiente:

( )

H ej

0 π/105 π/120

π

0.18375 0.210

22.05





44.1 f (kHz)

Sistemas de Tasa Múltiple.

194

c) El orden es 5409, y el número de multiplicaciones para implementar el diezmador en una sola etapa sería M P SH = 5410 × 44100 105 = 2272200 mult/s. d) El diagrama de bloques es el siguiente:

La respuesta en frecuencia de los filtros es la siguiente:

( )

G ej

0

π/8 π/7

0.275625

π

3.15

22.05

13π/7 13π/8 2π Ω

44.1 f (kHz)

40.95

|G(e j15Ω)|

|I(e jΩ)| 0

π/105 13π/105 2π/15

π/120 0.18375 0.210

2π Ω 44.1 f (kHz)

2.73

e) El orden del filtro interpolador I(z) es 61, mientras que el del filtro modelo G(z) es 390. f) El número de multiplicaciones requerido es el siguiente: M P SG = 391× 2940 7 = 164220 mult/s y M P SI = 62 × 44100 = 182280 mult/s. En total, el sistema requiere una complejidad de 15 346500 mult/s, es decir, 6.55 veces inferior al sistema original. Problema 5.20 Demostrar razonadamente que las siguientes estructuras son equivalentes, independientemente del valor de x[n]:

x n

L

L

H z

x n

y n

y n

E0 z donde E0 (z) representa la componente polifase de orden cero de H(z).

195 Solución Y (z) =

L−1

L−1

l=0

k=0

X X 1 X (z) z −l/L El (z) WL−kl = X (z) E0 (z) L

Problema 5.21 Se desea utilizar como filtro diezmador (figura 5.26) el sistema cuya estructura se representa la figura 5.27. Se pide:

x[ n]

h[n]

y[ n]

3

r [n ]

Figura 5.26

Figura 5.27

a) Encontrar la función del sistema del filtro H (z) . b) Obtener las tres componentes polifase tipo 1 H (z) del filtro, que permiten realizar el sistema como se indica en la figura: x[ n] 3

E (z)

3

E (z)

3

E2 (z)

0

z-1 1

z-1

r [n ]

c) Representar la estructura de cada una de ellas según la forma directa. Solución a) H (z) = 1 + 2,5355z −1 + 4,0547z −2 + 0,6839z −3 − 0,2377z −4 − 8,0365z −5 − 8,0365z −6 − 0,2377z −7 + 0,6839z −8 + 4,0547z −9 + 2,5355z −10 + z −11

Sistemas de Tasa Múltiple.

196 b)

E0 (z) = 1 + 0,6839z −1 − 8,0365z −2 + 4,0547z −3 E1 (z) = 2,5355 − 0,2377z −1 − 0,2377z −2 + 2,5355z −3 E2 (z) = 4,0547 − 8,0365z −1 + 0,6839z −2 + z −3 c)

Problema 5.22 De un filtro de fase lineal de un tercio de banda (Nyquist(3)), con h [n] = 0 para n < 0 y n > 8, se conocen los siguientes valores de la respuesta al impulso: h[2] = 0,0744, h[4] = 1/3, h[5] = 0,2385, h[8] = −0,0055. Dicho sistema se ha diseñado para ser incluido en un diezmador como el siguiente:

Figura 5.28 – Diezmador de orden 3.

Sin embargo, la implementación del mismo se lleva a cabo utilizando tres componentes polifase tipo I, como se indica a continuación:

197

Figura 5.29 – Implementación eficiente del diezmador de orden 3 con componentes polifase tipo I.

Se pide: a) Encontrar todos los valores de la respuesta al impulso h [n]. b) Obtener los coeficientes de las componentes polifase eℓ [n] para 0 ≤ ℓ ≤ 2. Para el siguiente apartado, considere que h[n] = n · (u [n] − u [n − 8]). c) Si se aplica como señal de entrada x[n] = n · (u [n] − u [n − 8]), obtener el valor de las muestras de las señales x0 [n], x1 [n], x2 [n], y0 [n], y1 [n], y2 [n] e y [n]. Solución a) h[0] = h[8] = −0,0055, h[1] = h[7] = 0, h[2] = h[6] = 0,0744 ,h[3] = h[5] = 0,2385 h[4] = 1/3.   b) e0 = −0,0055, 0,2385, 0,0744 . e1 = [0, 1/3, 0].   e2 = 0,0744, 0,2385, −0,0055 . c) x0 = [0, 3, 6]. x1 = [0, 2, 5]. x2 = [0, 1, 4, 7]. y0 = [0, 0, 9, 36, 36]. y1 = y2 = [0, 2, 13, 34, 35]. yn = y0 + y1 + y2 = [0, 4, 35, 104,

106].

Problema 5.23 Al sistema de la figura 5.30, donde el filtro H0 (z) es un filtro paso bajo ideal de ganancia unidad y de pulsación de corte π/2, se le introducen las señales de entrada de la figura 5.31.

x 0 [n]

x 1 [n]

2

2

y 0 [n]

y 1 [n]

H0 (z)

H1 (z)

Figura 5.30

r 0 [n]

r 1 [n]

s[n]

Sistemas de Tasa Múltiple.

198

X0(Ω)

1



1

π

X1(Ω)



π

Figura 5.31

Se pide: a) Obtener las expresiones de R0 (z) y R1 (z) en función de X0 (z), X1 (z), H0 (z), k y L. b) Representar los espectros de y0 [n] y r0 [n]. c) Encontrar un valor para k y otro para L de manera que s[n] contenga toda la información de las dos señales de entrada. d) Con los valores indicados en el apartado anterior, representar los espectros de r1 [n] y s[n].  π Datos: k, L ∈ Z+ , h1 [n] = h0 [n] · cos L ·k·n Solución

a)

b) Los espectros son:

−π

 R0 (z) = X0 z 2 Hh0 (z)   i  πk πk R1 (z) = X1 z 2 12 H0 ze−j L + H0 ze+j L Y0(Ω)

R0(Ω)

1

1

π/2

−π/2

πΩ

−π

π/2

−π/2

π Ω

c) k = L. d) Los espectros son:

−π

−π/2

R1(Ω)

S(Ω)

1

1

π/2

π Ω

−π

−π/2

π/2

π Ω

199 Problema 5.24 Considere el banco de filtros FIR de la figura 5.32:

H0 z

G0 z

X(z)

+ H1 z

Y(z)

G1 z

Figura 5.32

  donde H0 (z) = 12 1 + z −1 , H1 (z) = 12 1 − z −1 y G0 (z) = 21 (1 + z). Calcular el valor de G1 (z) para que el sistema tenga distorsión de amplitud unidad y distorsión de fase y factor de solapamiento nulos. Solución G1 (z) =

1 (1 − z) 2

Problema 5.25 En el sistema de la figura 5.33, H0 (z) es un filtro paso bajo ideal de ganancia P y de pulsación de corte π/2. A dicho sistema se le introduce la señal de entrada s(t), cuyo espectro se representa en la figura 5.34.

s(t)

C/D

s[n]

H0 (z)

x 0 [n]

M

y 0 [n]

T H1 (z)

x 1 [n]

M

y 1 [n]

Figura 5.33

S( ) 1

-3 ·106 -2 ·106

Y0 ( ) 1

2 ·10 6

6

3 ·10

Figura 5.34

Sabiendo que no se produce solapamiento, se pide: a) Encontrar valores para T , M , y P de manera que Y0 (Ω) se corresponda con el espectro (periódico de período 2π) representado en la figura 5.34. b) Con los valores obtenidos en el apartado anterior, representar los espectros de s[n], x1 [n] e y1 [n]. Dato: h1 [n] = h0 [n] · cos (π · n)

Sistemas de Tasa Múltiple.

200 Solución a) T = 41 10−6 s, M = 2, P = 12 10−6 . b) Los espectros son:

S(Ω) 1/T

X1(Ω) 2 = P/T

π/2 3π/4 π Ω

−π −3π/4 −π/2

−π −3π/4 −π/2

π/2 3π/4 π Ω

Y1(Ω) 1

π/2 3π/4 π Ω

−π −3π/4 −π/2

Problema 5.26 El diagrama de bloques de la figura 5.35 representa un banco de filtros de dos canales.

H0(z)

x0[n]

H1(z) x1[n]

p0[n]

2 2

p1[n]

r0[n] DSP

x[n]

r1[n]

Banco de Análisis

2 2

s0[n] s1[n]

F0(z)

F1(z)

y0[n]

y[n]

y1[n]

Banco de Síntesis Figura 5.35

Considere que no existe etapa de procesamiento intermedio, es decir, ri [n] = pi [n], 0 ≤ i ≤ 1, que todos los filtros tienen una respuesta al impulso finita, real y distinta de cero sólo para 0 ≤ n ≤ N. Se pide: a) Demostrar que la relación entrada-salida en el dominio z es del siguiente modo: Y (z) = T0 (z) · X (z) + T1 (z) · X (−z) Expresar la función de transferencia de distorsión total T0 (z) y la función de transferencia de solapamiento T1 (z) en función de H0 (z), H1 (z), F0 (z), F1 (z). Considere para el resto del problema que las respuestas al impulso de los filtros satisfacen las siguientes relaciones: h1 [n] = (−1)n h0 [n] , f0 [n] = h0 [n], n f1 [n] = − (−1) h0 [n]

201 b) Obtener H1 (z), F0 (z) y F1 (z) en función de H0 (z). c) Expresar T0 (z) y T1 (z) en función de H0 (z). d) Si la transformada z del filtro h0 [n] satisface la siguiente relación: H02 (z) − H02 (−z) = z −d, donde d ∈ Z+ , calcular la señal de salida y[n] en función de la entrada x[n]. Explicar razonadamente si y[n] se aproxima a x[n]. Solución a) T0 (z) = T1 (z) =

1 2 1 2

[H0 (z) F0 (z) + H1 (z) F1 (z)], [H0 (−z) F0 (z) + H1 (−z) F1 (z)].

b) H1 (z) = H0 (−z), F0 (z) = H0 (z), F1 (z) = −H0 (−z).   T1 (z) = 0. c) T0 (z) = 21 H02 (z) − H02 (−z) , d) y [n] = 21 x [n − d].

Capítulo 6

Análisis Espectral Problema 6.1 Sea x [n] , 0 ≤ n ≤ (N − 1) el conjunto de muestras disponibles de un proceso aleatorio estacionario. Se pide: a) Explicar razonadamente la definición y cálculo de la densidad espectral. b) El periodograma es un estimador no paramétrico de la densidad espectral de potencia. Explicar dos métodos para calcular el periodograma. c) Calcular el sesgo que introduce el periodograma y razonar su interpretación. d) Explicar razonadamente el procedimiento de estimación espectral paramétrica y los diferentes modelos que se pueden utilizar. Solución  a) Sx ejΩ = b)

NP −1

rx [k] e−jΩk , donde rx [k] = E {x [n] x [n − k]}.

k=−(N −1)

Método indirecto:  Sˆx ejΩ =

N −1 X

rˆx [k] e−jΩk , donde rˆx [k] =

k=−(N −1)

1 x [k] ∗ x [−k] . N

Método directo:  1  jΩ  2 1 Sˆx ejΩ = = X e N N

n  o c) Sx ejΩ − E Sˆx ejΩ =

NP −1

k=−(N −1)

|k| N rx

[k] e−jΩk .

N −1 2 X −jΩn x [n] e . n=0

d) x[n] es la respuesta de un LTI a ruido blanco. v[n], V(z)

x[n], X[z]

H(z) Sv(e jΩ ) = σv2

Sx(e jΩ ) = | H(e jΩ )|2 σv2

Análisis Espectral.

204 ARMA: x [n] = −

p X

ak x [n − k] +

k=1

B (z) H (z) = = A (z)

q P

bk z k=0 p P

q X

k=0

bk v [n − k],

−k

.

ak z −k

1+

k=1

AR: x [n] +

p X

k=1

H (z) =

ak x [n − k] = v [n] ,

1 = A (z)

1+

1 p P

. ak z −k

k=1

MA: x [n] =

q X

k=0

bk v [n − k],

H (z) = B (z) =

q X

bk z −k .

k=0

Problema 6.2 En el estudio de la estimación espectral mediante modelos estocásticos lineales, hemos obtenido las siguientes ecuaciones para el modelo AR:   Rw = r, M P  σv2 = ak rx [k], k=0

son conocidas como ecuaciones de Yule–Walker.

a) Explicar razonadamente qué representan cada una de las matrices y parámetros que aparecen en dichas ecuaciones. b) Explicar, incluyendo la formulación que sea necesaria, cuál es el sistema generador para un modelo AR. c) Explicar cómo calcularía los valores de autocorrelación necesarios para resolver las ecuaciones de Yule–Walker. Suponiendo que disponemos de la siguiente serie temporal: x[n] = T [1, 4, 5, 8, 4, 2, 6, 7, 7, 5] , calcular los valores estrictamente necesarios de autocorrelación para el caso de un modelo de orden 5.

205 Solución a) La ecuación matricial Rw = r puede desarrollarse:  rx [0] rx [1] · · · rx [M − 1]  ∗ rx [1] rx [0] · · · rx [M − 2]   rx∗ [2] rx∗ [1] · · · rx [M − 3]   .. .. .. ..  .  . . . rx∗ [M − 1] rx∗ [M − 2] · · · rx [0]

       

w1 w2 w3 .. . wM





      =      

rx∗ [1] rx∗ [2] rx∗ [3] .. . rx∗ [M ]



   .   

El modelo estocástico lineal establece que la serie temporal {x[n]} puede interpretarse como la salida de un sistema LTI al que se le inyecta ruido blanco gaussiano de media nula. Entonces, rx [k], k = 0, 1, · · · , M es la autocorrelación de x[n]. Los parámetros wk = −ak , k = 1, · · · , M son los coeficientes del sistema LTI y σv2 es la varianza del ruido a la entrada.

b) En un modelo AR (a0 = 1): M X

k=0

Puede escribirse:

a∗k x [n − k] = v [n] .

A(z) =

M X

a∗k z −k ,

k=0

A(z)X(z) = V (z). El sistema generador toma v[n] como entrada: HG (z) =

1 1 1 . = M = M P ∗ −k Q A (z) −1 ak z (1 − pk z ) k=0

k=1

c) Hay que estimar los valores de autocorrelación: rˆx [k] =

∞ 1 1 X ∗ x [k] ∗ x∗ [−k] = x [n] x [n + k]. N N n=−∞

Para un modelo de orden 5, sólo son necesarios 6 valores de autocorrelación: rx [k], k = 0, 1, · · · , 5. 1 rx [k] = [285, 242, 188, 154, 136, 137]T . 10

Análisis Espectral.

206

Problema 6.3 El modelo estocástico lineal de la señal aleatoria x[n] se corresponde con el diagrama de polos y ceros de la figura 6.1.

Diagrama polo−cero del filtro H(z) 1

Parte Imaginaria

0,8 0,6 0,4 0,2 0 −0,2 −0,4 −0,6 −0,8 −1 −1

−0,5

0 0,5 Parte Real

1

1,5

Figura 6.1

a) Indicar en qué consiste el modelo estocástico lineal de una señal aleatoria, qué tipo de modelo es el representado en la figura 6.1 y con cuál de los espectros de potencia mostrados en la figura 6.2 se corresponde.

(a)

(b) 25 Densidad espectral de Potencia

Densidad espectral de Potencia

12 10 8 6 4 2 0

0

0,1

0,2 0,3 0,4 Frecuencia (Ω/2π)

0,5

20 15 10 5 0

0

0,1

0,2 0,3 Frecuencia (Ω/2π)

0,4

0,5

207 (c)

(d) 14

15

10

5

0

0

0,1

0,2 0,3 Frecuencia (Ω/2π)

0,4

0,5

Densidad espectral de Potencia

Densidad espectral de Potencia

20

12 10 8 6 4 2 0

0

0,1

0,2 0,3 Frecuencia (Ω/2π)

0,4

0,5

Figura 6.2

b) La figura 6.3 muestra diferentes representaciones de la estimación no paramétrica del espectro de potencia de una señal aleatoria. Determinar con cuál de los sistemas de la figura 6.2 se corresponde. Indicar por qué se obtienen diferentes versiones del periodograma en la figura 6.3 y razonar cómo podría obtenerse cada una de ellas. (a)

(b)

400

120 100

300 Amplitud

Amplitud

80 200

60 40

100 20 0

0

0,1

0,2 0,3 Frecuencia (Ω/2π)

0,4

0

0,5

0

0,1

(c)

0,2 0,3 Frecuencia (Ω/2π)

0,4

0,5

0,4

0,5

(d)

120

100

100

80 Amplitud

Amplitud

80 60 40

40 20

20 0

60

0

0,1

0,2 0,3 Frecuencia (Ω/2π)

0,4

0,5

0

0

0,1

0,2 0,3 Frecuencia (Ω/2π)

Figura 6.3

Solución a) El proceso x[n] se obtiene como la salida de un LTI al que se le inyecta ruido blanco de media cero. La densidad espectral de potencia:   2 Sx ejΩ = H ejΩ σv2 .

Análisis Espectral.

208

La figura 6.1 se corresponde con un modelo ARMA. El modelo de la figura 6.1 tiene dos ceros en z = 0 y z = −1 correspondiente con los espectros de la figura 6.2 (b) y 6.2 (c). Los máximos locales se deben a la presencia de polos en esas pulsaciones. La solución es 6.2 (c), cuyo primer máximo local se corresponde con un polo situado en un ángulo inferior a π/4. b) La figura 6.2 (a) ya que es el único caso con componentes de baja frecuencia predominantes. La figura 6.3 (a) se ha obtenido mediante el cálculo del periodograma. El resto, también pero presentan una varianza menor. Por tanto, estas últimas se han obtenido mediante técnicas específicas para minimizar la varianza, por ejemplo mediante promediado de periodograma. El caso inmediato es mediante el periodograma de Bartlett, que consiste en dividir la observación en segmentos no solapados. Otra posibilidad consistiría en aplicar enventanado a cada segmento para minimizar las fugas espectrales, obteniendo una versión más suavizada del espectro. Y finalmente, se puede realizar la combinación de las anteriores (promediado y enventanado) utilizando segmentos solapados. Esta forma más genérica se denomina periodograma de Welch. Problema 6.4 El periodograma Sx [k] de la señal x[n], n = 0, · · · , N − 1, se puede calcular a partir de X[k], siendo X[k] la DFT de N puntos de x[n]. En estas condiciones, se pide: a) Demostrar la validez de la expresión siguiente:

NP −1 n=0

|x [n]|2 =

NP −1

Sx [k]

k=0

b) Demostrar que el periodograma se puede calcular como Sx [k] = DF T {s [n]}, donde s[n] es el resultado de una convolución circular. c) Comprobar el apartado b) utilizando la señal x[n] = [1, 2, 0, −1]T . Solución a) N −1 X n=0

2

N −1 X

X 1 X x [n] X ∗ [k] WNkn = N n n=0 k X 1 X X 1 = X ∗ [k] X ∗ [k] X [k] = x [n] WNkn = N N n

|x [n]| =

x [n] x∗ [n] =

k

k

N −1 X X 1 2 |X [k]| = Sx [k]. = N k=0

k

b) s [n] =

1 x [n] ⊛ x∗ [((−n))N ] . N

c) x [((−n))N ] = [1, − 1, 0, 2]T ,

X[k] = [2, (1 − 3j), 0, (1 + 3j)]T , s[n] = [3/2, 1/4, − 1, 1/4]T ,

Sx [k] = [1, 5/2, 0, 5/2]T .

209 Problema 6.5 Determinar el valor de la densidad espectral de potencia a la pulsación de Ω = 4π 3 rad para T la realización x[n] = [3, 1, 1, 2, 1, 1, 4] utilizando el periodograma y la DFT con el menor número de punto. Solución 2 Sx [k] = N1 |X [k]| donde N = 7. Para la pulsación que nos piden, bastan 3 puntos de la DFT. Se llega a Sx [k = 2] = 7. Problema 6.6 Un procedimiento válido para estimar la densidad espectral de potencia de una señal aleatoria x[n], n = 0, 1, · · · , N − 1, es a partir de la transformada de Fourier de un estimador de la autocorrelación: ∞ X  rˆx [k] e−jΩk , Sˆx ejΩ = n=−∞

donde rˆx [k] es la estima de autorrelación calculada a partir de la realización x[n].

a) Si se utiliza la expresión rˆx [k] = demostrar que es insesgado.

1 N −|k| x [k] ∗ x [−k]

como estimador de la autocorrelación,

b) Indicar la expresión del estimador de autocorrelación que se utiliza para el cálculo del periodograma y demostrar que los métodos de cálculo directo e indirecto del periodograma son equivalentes. Solución a) Para k > 0 y k < 0: rˆx [k] = rˆx [k] =

NX −1−k 1 x [n] x [n + k], N − |k| n=0 N −1 X 1 x [n] x [n + k] . N − |k| n=|k|

Para ambas expresiones: E {ˆ rx [k]} = rx [k]. b) Para el periodograma: rˆx [k] =

1 N

N −1−k P

x [n] x [n + k].

n=0

Problema 6.7 Sea x[n] una serie temporal correspondiente a un proceso estocástico de longitud igual a cuatro, y sea X(Ω) su transformada de Fourier. De la anterior serie, se conoce que el valor del espectro para las pulsaciones {−7π/2, −2π, 7π/2, 5π} es {1, 7, 1, −5}, respectivamente. Se pide: a) Determinar la secuencia x[n]. b) Obtener la estima de la autocorrelación definida como rˆ [k] =

1 N x [k]

∗ x∗ [−k].

c) Empleando el periodograma, calcular el valor de la densidad espectral de potencia estimada para Ω = π.

Análisis Espectral.

210 Solución

a) Las pulsaciones en el intervalo [0, 2π] son {π/2, 0, 3π/2, π}. Reordenando, X[k] = [7, 1, −5, 1]T . La DFT inversa: x[n] = [1, 3, 0, 3]T . b) ˆr [k] =

1 4

T

[3, 9, 3, 19, 3, 9, 3] .

c) Sˆ (π) = 25/4. Problema 6.8 Un proceso AR x[n] de orden p viene definido como la salida de un sistema LTI todo-polos de orden p excitado mediante ruido blanco de media cero. Las ecuaciones de Yule-Walker se corresponden con la solución para la estimación de los parámetros ak , k = 1, . . . , p, y la varianza:  p P   ak rx [−k], m = 0,  σe2 = rx [0] + k=1

p P   ak rx [m − k] = 0, 1 ≤ m ≤ p.  rx [m] + k=1

En adelante, considere un proceso AR de orden 1 (p = 1). Se pide de forma razonada: a) Determinar el valor de los parámetros que describen al modelo AR. Los valores estimados de la autocorrelación se proporcionan abajo. b) Conocidos los parámetros del sistema AR, obtener la expresión simplificada de la densidad espectral de potencia del proceso x[n]. Datos: rˆx [0] = 2, rˆx [1] = 1 obtenidos mediante el estimador: rˆx [k] =

1 x [k] ∗ x [−k] . N

Solución a) rx [1] 1 =− , rx [0] 2 2 2 r [0] − r [1] 3 x σe2 = x = . rx [0] 2 a1 = −

b) Sx (Ω) =

3 · 2

5 4

1 . − cos Ω

Problema 6.9 Sea x[n] un proceso aleatorio caracterizado mediante un modelo lineal AR de orden p. La solución del mismo se obtiene mediante las ecuaciones normales cuya forma matricial es la siguiente:      1 rx [0] rx [−1] rx [−2] · · · rx [−p] σe2   r [1]    rx [0] rx [−1] · · · rx [−p + 1]   x   a1   0       rx [1] rx [0] · · · rx [−p + 2]   a2  =  0  .  rx [2]   .     .. .. .. ..  .   ..   ..  .  .  .   .  . . . ap rx [p] rx [p − 1] rx [p − 2] · · · rx [0] 0

211 Para el caso particular de un proceso AR de orden 2, se pide de forma razonada: a) Obtener la expresión general simplificada de la densidad espectral de potencia del proceso x[n]. b) Calcular el valor de los parámetros que describen al modelo AR. c) Determinar el valor de los parámetros para el caso concreto en que se dispone de la realización siguiente x = [1, 3, 2, 0, −1, 0, 1, 1, 2, 3]T . Solución a) Sx (Ω) =

σe2 |A (Ω)|

2

=

1+

a21

+

a22

σe2 . + 2a1 (1 + a2 ) cos (Ω) + 2a2 cos (2Ω)

b) rx [1] rx [2] − rx [0] rx [1] , rx2 [0] − rx2 [1] r2 [1] − rx [2] rx [0] a2 = x 2 , rx [0] − rx2 [1] a1 =

σe2 = rx [0] +

rx2 [1] rx [2] − rx2 [1] rx [0] rx2 [1] rx [2] − rx2 [2] rx [0] + . rx2 [0] − rx2 [1] rx2 [0] − rx2 [1]

c) El estimador de autocorrelación es: rˆx [k] = Se obtiene:

1 N x [k]

∗ x [−k].

rˆx [0] = 3;

a1 = −0,8125;

rˆx [1] = 1,8;

a2 = 0,3542;

rˆx [2] = 0,4;

σe2 = 1,6792.

Problema 6.10 Pp El filtro FIR A (z) = 1 + k=1 ak z −k sirve como modelo para describir una señal aleatoria estacionaria x[n]. Si tomamos como entrada x[n], la salida es ruido blanco de media cero y varianza σv2 . Las ecuaciones que resuelven el sistema son las ecuaciones de Yule-Walker:     

rx [0] rx [1] .. .

rx [−1] rx [0] .. .

· · · rx [−p + 1] · · · rx [−p + 2] .. .. . . rx [p − 1] rx [p − 2] · · · rx [0] σv2 = rx [0] +

p X

      ·  

a1 a2 .. . ap





    =  

−rx [1] −rx [2] .. . −rx [p]



  . 

ak rx [−k].

k=1

Considerando x[n] = [1, 3, 2, 0, −1, 0, 1, 1, 2, 3]T , se pide: a) Explicar razonadamente de qué modelo se trata y cómo se obtiene la densidad espectral de potencia de x[n]. b) Determinar los parámetros para el caso de un modelo de orden 2 (p = 2).

Análisis Espectral.

212 Solución   2 a) Se trata de un modelo AR: Sx ejΩ = H ejΩ σv2 .

b)

a1 = −0, 8125, a2 = 0, 3542, σe2 = 1, 6792.

Problema 6.11 La serie temporal x[n], n = 0, 1, · · · , N −1 correspondiente a un proceso aleatorio estacionario viene caracterizada mediante un modelo AR definido por los parámetros siguientes: a1 = −1,4, a2 = 0,74. Además, para el cálculo de estos valores, ha sido necesario estimar los siguientes valores de autocorrelación: rx [0] = 4,97, rx [1] = 4 y rx [2] = 1,92. Se pide: a) Con los datos del enunciado, indicar cual sería el orden del modelo y determinar la expresión de la función de transferencia del sistema generador. b) Explicar razonadamente si el conocimiento de los parámetros a1 y a2 es suficiente para estimar la densidad espectral del proceso. En caso contrario, calcular el valor de los parámetros restantes. c) Representar gráficamente y de forma aproximada la función densidad espectral de potencia estimada a partir de los datos disponibles.

Solución a) Orden 2: HG (z) =

1 . 1 − 1,4z −1 + 0,74z −2

b) Se necesita conocer el valor de varianza del ruido, σv2 =

PM

k=0

ak rx [k]

σv2 = 4,97 − 1,4 × 4 + 0,74 × 1,92 = 0,79. c) jΩ

Sx(e )

6.83

0

0.5

0,62 rad

1

1.5

Frequency (Ω)

2

2.5

3

213 Problema 6.12 Se desea caracterizar la secuencia x[n] = [7, 0, 3, 0, 1]T mediante un modelo AR de orden 2. Se pide: a) Determinar los parámetros del modelo y su función de transferencia. b) Determinar la densidad espectral de potencia de x[n] y representar aproximadamente su módulo. Solución a)

581 24 2 , σv = . 59 59 1 HG (z) = 24 −2 . 1 − 59 z

a1 = 0, a2 = −

b)

 581 Sx ejΩ = 59 1 +

 24 2 59

Con el diagrama de polos y ceros : 30

1 −

48 59

cos 2Ω

.

X: 0 Y: 27.98

25

x

S (ejΩ)

20

15

10 X: 1.569 Y: 4.976

5

0 0

0.5

1

1.5

Ω (rad)

2

2.5

3

Análisis Espectral.

214

Problema 6.13 En la figura 6.4 se muestran las densidades espectrales de potencia de varias series temporales caracterizadas mediante un modelo estocástico real y lineal. 14

5 4.5

12

Densidad espectral de potencia

Densidad espectral de potencia

4 3.5 3 2.5 2 1.5

10

8

6

4

1

2 0.5 0 0

0.05

0.1

0.15

0.2

0.25

0.3

0.35

0.4

0.45

0 0

0.5

0.05

0.1

0.15

0.2

0.3

0.35

0.4

0.45

0.5

0.35

0.4

0.45

0.5

(b) 10

4

9 8

3.5

Densidad espectral de potencia

Densidad espectral de potencia

(a) 4.5

3 2.5 2 1.5 1

7 6 5 4 3 2

0.5 0 0

0.25

Frecuencia (Ω/2π)

Frecuencia (Ω/2π)

1

0.05

0.1

0.15

0.2

0.25

0.3

0.35

0.4

0.45

0.5

0 0

0.05

0.1

0.15

0.2

0.25

0.3

Frecuencia (Ω/2π)

Frecuencia (Ω/2π)

(c)

(d)

Figura 6.4 – Densidad espectral de potencia de varias series temporales.

a) Explicar razonadamente cuáles de ellas se pueden corresponder con un modelo AR. b) Explicar razonadamente cuáles de ellas se pueden corresponder con un modelo MA. c) Para un modelo AR de orden 4, se conocen los valores estimados de la autocorrelación y algunos de los parámetros que lo definen (ver sección de datos abajo). Determinar los parámetros restantes que permiten caracterizar la estimación de la densidad espectral potencia. Datos: ˆrx = [1,9793 0,8975 0,3384 − 0,0305 − 1,0300]T , a = [a0 − 0,4113 − 0,0034 a3 0,5980]T . Solución a) La figura (b) ya que el resto poseen raíces en la circunferencia de radio unidad. b) Cualquiera de ellas se podría modelar mediante un MA. c) a0 = 1, a3 = −0,1839. La varianza: σv2

=

4 X

k=0

ak rx [k] = 0,9987.

215 Problema 6.14 Las curvas de trazo continuo que se muestran en las gráficas de la figura 6.5 representan la auténtica densidad espectral de potencia de un proceso aleatorio. Se realiza la estimación de dicho espectro mediante un periodograma promediado y tres modelos AR de orden 2, 4 y 6, cuyas soluciones se muestran en trazo discontinuo en las mismas gráficas. 6

4.5 4

Densidad espectral de potencia

Densidad espectral de potencia

5 3.5 3 2.5 2 1.5 1

4

3

2

1 0.5 0 0

0.05

0.1

0.15

0.2

0.25

0.3

0.35

0.4

0.45

0 0

0.5

0.05

0.1

0.15

0.2

0.3

0.35

0.4

0.45

0.35

0.4

0.45

0.5

(b)

4.5

4.5

4

4

3.5

3.5

Densidad espectral de potencia

Densidad espectral de potencia

(a)

3 2.5 2 1.5 1 0.5 0 0

0.25

Frecuencia (Ω/2π)

Frecuencia (Ω/2π)

3 2.5 2 1.5 1 0.5

0.05

0.1

0.15

0.2

0.25

0.3

0.35

0.4

0.45

0.5

0 0

Frecuencia (Ω/2π)

0.05

0.1

0.15

0.2

0.25

0.3

0.5

Frecuencia (Ω/2π)

(c)

(d)

Figura 6.5

Indicar de forma razonada a cual se corresponde cada una de ellas. Solución El modelo AR es idóneo para representar las resonancias o componentes armónicas de un espectro. Cada par de polos conjugados se corresponde con un máximo local en la curva estimada de la DEP. Así, en 6.5(d) se aprecia un máximo, por lo que el modelo aplicado será el de orden 2. En 6.5(a) se observan 2 máximos, correspondiente a un AR de orden 4 y el de orden 6 se corresponderá con la gráfica 6.5(c). Lógicamente, la que nos queda, 6.5(b) , es la correspondiente a un periodograma promediado.

Bibliografía [1] M. Blanco Velasco, F. Cruz Roldán, R. Jiménez Martínez, and J. Sáez Landete. Tratamiento Digital de Señales. Servicio de Publicaciones de la Universidad de Alcalá, 2013. [2] A. V. Oppenheim, R. Schafer, and J. R. Buck. Discrete-Time Signal Processing. PrenticeHall, third edition, 2010. [3] J. G. Proakis and D. G. Manolakis. Tratamiento Digital de Señales. Principios, Algoritmos y Aplicaciones. Prentice-Hall, fourth edition, 2007. [4] S. K. Mitra. Digital Signal Processing. A Computer Based Approach. McGraw-Hill, third edition, 2005. [5] P. S. R. Diniz, E. A. B. da Silva, and S. L. Netto. Digital Signal Processing. System Analysis and Design. Cambridge University Press, 2011. [6] M.H. Hayes. Statistical Digital Signal Processing and Modeling. John Wiley & Sons, Inc. New York, NY, USA, 2008. [7] S. Haykin. Adaptive Filter Theory. Prentice-Hall, fifth edition, 2014. [8] J. McClellan. Educational Matlab GUIs. http://users.ece.gatech.edu/mcclella /matlabGUIs/, 2008. [9] A. Spanias. Java Digital http://jdsp.asu.edu/jdsp.html, 2008. [10] W. J. Rugh. Signal, http://www.jhu.edu/signals/, 2003.

Signal

Systems,

Processing

and

Control

(J-DSP)

Editor.

Demonstrations.

[11] C. A. Nyack. A Visual Interactive Approach to Digital Signal Processing. http://dspcan.homestead.com/files/idxpages.htm, 2008. [12] C. S. Burrus, J. H. McClellan, A. V. Oppenheim, T. W. Parks, R. W. Schafer, and R. W. Schuessler. Ejercicios de Tratamiento de la Señal Utilizando MATLAB v.4. PrenticeHall, 1998. [13] C. S. Burrus, J. H. McClellan, A. V. Oppenheim, T. W. Parks, R. W. Schafer, and R. W. Schuessler. Computer-Based Exercises for Signal Processing Using Matlab 5 (Matlab Curriculum Series). Prentice-Hall, 1998. [14] R. V. Churchill and J. W. Brown. Variable Compleja y Aplicaciones. McGraw-Hill, seventh edition, 2004.

218

BIBLIOGRAFÍA

[15] R. E. Crochiere and L. R. Rabiner. Multirate Digital Signal Processing. Prentice-Hall, New Jersey, 1983. [16] L. E. Franks. Teoría de la Señal. Reverté, 1975. [17] S. Haykin and B. Van Veen. Signals and Systems. John Wiley & Sons, second edition, 2003. [18] E. C. Ifeachor and B. W. Jervis. Digital Signal Processing: A Practical Approach. Pearson Education, 2002. [19] L. B. Jackson. Digital Filters and Signal Processing with MATLAB Exercises. Kluwer Academic Publishers, 1996. [20] L. B. Jackson. Signals, Systems, and Transforms. Addison-Wesley Publishing, 1991. [21] J. García de Jalón, J. I. Rodríguez, and A. Brazález. Aprenda MATLAB 7.0 como si estuviera en primero. http://mat21.etsii.upm.es/ayudainf/aprendainf/varios.htm, 2005. [22] D. G. Manolakis, V. K. Ingle, and S. M. Kogon. Statistical and Adaptive Signal Processing. McGraw-Hill, 2000. [23] J. B. Mariño, F. Vallverdu, J. A. Rodriguez, and A. Moreno. Tratamiento Digital de la Señal: Una Introducción Experimental. Edicions UPC, 1999. [24] J. H. McClellan, R. W. Schafer, and M. A. Yoder. Signal Processing First. Prentice-Hall, 2003. [25] A. V. Oppenheim, R. W. Schafer, and J. R. Buck. Tratamiento de Señales en Tiempo Discreto. Prentice-Hall, 2000. [26] A. V. Oppenheim, A. Willsky, and S. Hamid. Signals and Systems. Prentice-Hall, 1997. [27] A. V. Oppenheim, A. Willsky, and S. H. Nawab. Señales y Sistemas. Prentice-Hall, 1998. [28] T. W. Parks and C. S. Burrus. Digital Filter Design. Wiley-Interscience New York, NY, USA, 1987. [29] J. G. Proakis and D. G. Manolakis. Digital Signal Processing. Principles, Algorithms and Applications. Prentice-Hall, fourth edition, 2006. [30] D. Schlichthärle. Digital Filters: Basics and Design. Springer, 2000. [31] S. S. Soliman and M. D. Srinath. Señales y Sistemas Continuos y Discretos. PrenticeHall, 1999. [32] S. S. Soliman and M. D. Srinath. Continous and Discrete Signals and Systems. PrenticeHall, second edition, 1998. [33] G. Strang and T. Q. Nguyen. Wavelets and Filter Banks. Wellesley-Cambridge Press, 1997. [34] R. D. Strum and D. E. Kirk. Contemporary Linear Systems Using MATLAB. Brooks/Cole Thomson Learning, 2000. [35] P. P. Vaidyanathan. Multirate Systems and Filter Banks. Prentice-Hall, 1993.